Anda di halaman 1dari 87

1

2012

Penunjang Masa Depan


Neuropsikiatri

PDA 2010
Being
Cumlaude

DAFTAR ISI

Neuroanatomy 1 (3)
Neuroanatomy 2 (4)
Neurohistology (6)
Neurophysiology 1 (7)
2
Neurophysiology 2 (9)
MKDI Neurology 1 (11)
MKDI Neurology 2 (16)
MKDI Neurology 3 (24)
MKDI Neurology 4 (25)
MKDI Neurology 5 (30)
Neuroembriology (34)
Neuroperinatology (35)
Neuropediatry (36)
Neuroocular-microbiology (38)
Neuroemergency (41)
Neuropathology anatomy Anesthetic (42)
Neuromedical rehabilitation (45)
Neuropublichealth (48)
Neurosurgery (50)
MKDI Psychiatry 1 (53)
MKDI Psychiatry 2 (57)
Farmakopsikiatri (65)
Gabungan 1 (70)
Gabungan 2 (73)
Gabungan 3 (76)
Gabungan 4 (80)
NEUROANATOMY 1 postganglion yaitu pada autonomic ganglia

1. Pernyataan yang benar untuk axon hillock 10. Neurofibril tersusun atas kolagen
adalah :
A. Mengandung banyak Nissls 11. Axon hillock adalah
substance
3
B. Tidak mengandung neurofilament 12. Traktus piramidalis berada pad apes Pontis
C. Merupakan pangkal axon
D. Dilapisi myelin 13. Yang sifatnya sensoris selain olfactorius n
E. Meneruskan rangsang melalui konduksi
obticus adalah vestibulocochlearis
salutatory

2. Yang termasuk neurofibril adalah : 14.Nervus cranialis terkecil adalah Olfactorius


A. Serabut kolagen
B. Serabut elastic 15.Yang tidak terdapat pada brain stem selain
C. Serabut retikuler olfactorius adalah opticus
D. Microtubule
E. Dendrite 16.Yang memlihara BBB adalah astrosid

3. Myelin di PNS dibentuk oleh :


17.Myelin di pns adalah sel schwan
A. Neuron
B. Fibroblast
C. Sel Schwann 18.Myelin di cns adalah oligodendrosit
D. Oligodendrocyte
E. Sel satelit 19.Produksi CSF adalah ventrikel

4. Myelin di CNS dibentuk oleh : 20.Reabsorbsi csf adalah vili arachnoid


A. Astrocyte fibrosa
B. Astrocyte protoplasmatic 21.Nosiseptor adalah ujung saraf bebas
C. Oligodendroglia
D. Microglia
22.Sepasang saraf spinal yang mewakili tubuh
E. Sel ependym
adalah dermatom
5. Nociceptor termasuk :
a. Free nerve ending 23.proprioseptik ditemukan di muscle
b. Memiliki kapsul
c. Merespon vibrasi 24.nucleus ruber terletak di :
d. Sama dengan CORP. Meissner a. ganglia basalis
b. ponds
6. Kalo ada darah pada CSF berarti (pendarahan c. mesensefalon
subarachnoid) d. m. oblongata
e. m. spinalis
7. Efector organ somato sensory
a. Smooth mscl 25.komponen CNS
b. Gland a. nervus cranialis
c. - b. nervus spinalis
d. Skeletal muscle c. ganglion
e. Benar semua d. nucleus
e. parasimpatis
8. Ada hemiparese n kelmpuhan nervus fagus,
diduga kerusakan di m.oblongata
26.nociceptor.
a. free nerve ending
9. Pada ans, hubungan preganglion dan b. encapsulated reseptor
c. NEUROANATOMY 2
d.
1.
27.yang bukan reseptor kutaneus adalah:
a. chemoreseptor 2. Yang termasuk neurofibril adalah
b. mechanoreseptor a. Serabut kolagen
b. Serabut elastic
c. nociceptor c. Serabut retikuler 4
d. thermoreseptor d. Microtubule
e. Dendrite
28.ciri2 somaticnervous system
a. 1 neuron CNS dengan skeletal muscle 3 . Neuroglia yang membentuk blood brain
b. bisa eksitatory atau inhibitory barrier adalah:
c. cell body berada di lateral spinal cord a. Astrocyte
d. reseptornya muskarinik atau noclotinik b. Oligodendrocte
e. semua benar c. Microglia
d. Sel Satelit
e. Sel Ependyme

4. Struktur apakah yang berperan dalam pusat


emosi
a. Basal ganglia
b. Cerebrum
c. Thalamus
d. Hipotalamus
e. Hipokampus

5. Sinus Duraemater yang robek dapat


menimbulkan :
a. Epidural hematoma
b. Subdural bleeding
c. Arachnoid bleeding
d. Subarachnoid haemorrhage
e. Intracerebral haemorrhage

6. Pada membrane presinap terdapat


a. Synaptic cleft
b. Reseptor neurotransmitter
c. Synaptic vesicle
d. Post synaptic membrane
e. Ach esterase

7. Koordinasi seluruh pusat control system


saraf otonom di otak adalah
a. Lobus temporalis
b. Lobus occipitalis
c. System limbic
d. Hipofisis posterior
e. Hipotalamus

8. Struktur di bawah ini yang melekat pada


tulang adalah :
a. Periost dura
b. Meningeal dura
c. Arachnoid
d. Piameter
e. Plexus choroideus
9. Struktur di bawah ini termasuk Sistem Saraf
Pusat
a. Nervus Cranialis
b. Nervus Spinalis
c. Ganglion a. Vena emissariae
d. Nukleus b. V. ophtalmica
e. Saraf parasimpatis c. V. Cerebri magna Galeni
d. V. Jugularis Interna
e. V. Jugularis externa
10. Cairan CSF yang mengandung darah
menunjukkan : 18. Tempat pembentukan cerebrospinal fluid :
a. EDH 5
a. Canalis spinalis
b. SDH b. Ventrikel otak
c. Arachnoid bleeding c. Subarachnoid
d. SAH d. Sinus sagitalis superior
e. ICH e. Sel Ependym
11. Sistem saraf otonom bersifat 19. Pembentuk terluar saraf tepi
a. Somatic afferens a. Epineurium
b. Somatic efferens b. Endoneurium
c. Visceral afferens c. Perineurium
d. Visceral efferens d. Sel Schwann
e. Sensoric e. Mielin
12. Struktur berikut dipengaruhi Sistem Saraf 20. Clarke dorsalis terletak di
Tepi Visceral a. Pons
a. Otot skeletal b. Mesencephalon
b. Kulit c. Basal Ganglia
c. Mukosa d. Medulla spinalis
d. Tendon e. Medulla oblongata
e. Pembuluh darah
21. Sirkuit dari reseptor menuju ke pusat dan
13. Bagian dari mesensepalon yang menjadi kembali ke efektor pada segmen yang berbeda
pusat koordinasi posisi tubuh tanpa melalui pusat tertinggi system saraf,
a. Pons disebut :
b. Cerebrum a. Central Nerve System
c. Cerebelum b. Peripheral Nerve System
d. Medula Oblongata c. Autonomic Nerve System
e. Basal Ganglia d. Simple reflex
e. Complex reflex
14. Struktur pada atap Diencephalon yang
memproduksi CSF adalah 22. Neuroglia yang membentuk Blood Brain
a. Lamina terminalis Barrier
b. Hemisfer cerebri a. Astrocyte
c. Plexus Coroideus b. Oligodendrocyte
d. Ventrikel lateral c. Microglia
e. Foramen Monroe d. Sel ependyme
e. Sel schwann
15. Circulus Willisi menerima darah arterial dari :
a. A. Cerebri anterior 23. Mielin di CNS dibentuk oleh :
b. A. Cerebri Media a. Astrosit fibrosa
c. A. Cerebri Posterior b. Astrosit protoplasmic
d. A. carotis Interna c. oligodendroglia
e. A. Carotis Externa d. Microglia
e. Sel Ependym
16. Pernyataan yang benar untuk axon hillock 24. Pada ANS, preganglion dan postganglion
adalah bersinaps di
a. Mengandung banyak Nissl;s substance a. Autonomic ganglia
b. Tidak mengandung neurofilament b. Batang otak
c. Merupakan pangkal akson c. Medulla spinalis
d. Dilapisi mielin d. Ganglion dorsalis
e. Meneruskan rangsang melalui konduksi e. Sel otot skeletal
saltatory
25. -
17. Darah vena dari otak kembali ke jantung
melalui : 26. Struktur berikut dipengaruhi Sistem Saraf
Tepi Visceral NEUROHISTOLOGY
a. Cardiac Muscle
b. Smooth Muscle
c. Glands 1. a.Lapisan di korteks ini disebut sebagai
d. Pembuluh Darah LAPISAN GRANULAR
b. terletak di organ CEREBELLUM
e. Viscera organ
2. a.komposisi bercak di ujung petunjuk terdiri
atas RETIKULUM ENDOLASMA DAN RIBOSOM 6
28. Benar mengenai traktus piramidalis b. bercak-bercak ini terletak di dalam
a. Gerakan motorik kontralateral PERIKARION
b. Tidak melibatkan basal ganglia
c. Bekerja sama dengan cerebellum hanya 3. a. Menurut bentuk/polaritasnya, sel ini disebut
pada sensorik cerebellum SEL SARAF MULTIPOLAR
d. Diteruskan ke medulla spinalis melalui b. sel ini terletak di SUBSTANSIA GRISEA
spinotalamikus reseptor
4. a. Daerah pucat di ujung petunjuk adalah
e. Menerima sensoris dari kortex anterior AXON HILLOCK
b. Berbeda dengan perikaryon sekelilingnya, di
29. Hemiparesis yang disertai tanda Vagus daerah ini tidak terdapat NISSLLS BODIES
mengindikasikan lesi di
a. Medula Spinalis 5. a. Struktur berlumen di ujung petunjuk adalah
b. Medula Oblongata CANALIS CENTRALIS
c. Pons b. struktur ini dilapisi oleh EPENDYMAL CELL
d. Mesensepalon 6. a. Struktur pembungkus medulla spinalis ini
e. Subkortex cerebri adalah DURAMATER SPINALIS
b. Struktur ini dibentuk oleh JARINGAN IKAT
30. Basal plate dari myelencephalon PADAT/FIBROSA
mengandung nucleus motorik dari
a. N. IX ; N.X ; N.XI 7. a. Garis gelap di ujung petunjuk adalah AXON
b. N.XII b. Struktur terang disekitar garis tersebut
adalah MYELIN
c. N.III ; N.IV ; N.V
d. N.VII; N.VIII 8. a. Struktur pembungkus ini disebut sebagai
e. N.I PERINEURIUM
b. Struktur yang dibungkus olehnya adalah
FASCICULUS SARAF PERIFER
9. a. Celah di ujung petunjuk adalah NODUS
RANVIER
b. fungsi dari celah ini adalah MEMPERCEPAT
PENJALARAN IMPULS
10. a.Sel di ujung petunjuk adalah SEL SCHWAN
b. Fungsi sel ini adalah membentuk MYELIN DI
SARAF PERIFER
11. a. Sel di ujung petunjuk adalah SEL
PYRAMIDALIS
b. Sel ini terletak di organ CEREBRUM
12. a. Struktur berlipat-lipat di ujung
petunjuk adalah PLEXUS CHOROIDEUS
b. Fungsi struktur ini adalah
MEMPRODUKSI CSF
13. a. Glia di ujung petunjuk adalah
(spesifik) ASTROSIT PROTOPLASMIK
b. Lokasi dari glia jenis ini adalah di
SUBSTANSIA GRISEA
14. a. Glia pipih kecil di ujung petunjuk ini
adalah MIKROGLIA
b. Fungsi glia ini adalah FAGOSITOSIS
DEBRIS DI CNS
15. a. Titik hitam kecil di ujung petunjuk
adalah NUKLEOLUS
b. Sel tempat titik ini berada adalah SEL
GANGLION DI GANGSLION SPINLAIS
16. a. Sel bulat kecil yang tercat hitam ini 16. a. Sel satelit
adalah SEL SATELIT b. Sel Schwann
b. Sel ini merupakan derivate dari SEL
SCHWAN 17. a. Plexus Meissner
b. Submukosa
17. a. PREPARAT INTESTINUM. Kelompok
sel adalah PLEXUS MEISSNER 18. a. Plexus Auerbach
b. Sel-sel ini terletak di lapisan b. Mengatur peristalsis/motorik usus 7
SUBMUKOSA
19. a. Corpusculum Meissner
18. a. PREPARAT INTESTINUM. Kelompok b. papilla dermis
sel adalah PLEXUS AUERBACH
b. Sel-sel plexus ini berfungsi untuk 20. a. Serabut otot intrafusal
REGULASI PERISTALTIK USUS b. Muscle spindle
19. a. Struktur di ujung petunjuk adalah
CORPUSCULUM MEISSNER
b. Lokasi struktur ini adalah di PAPILLA DERMIS
20. a. Struktur bulat yang ditunjuk adalah
SERABUT OTOT INTRAFUSAL
NEUROPHYSIOLOGY 1
b. Seluruh struktur berkapsul disekelilingnya
adalah MUSCLE SPINDLE 1. Yang bukan tanda fight or flight respons
a.lupa
JAWABAN : b.meningkastnya gluk. Darah
1. a .Lapisan granular c. meningkatnya tek, darah
b. cerebellum d.meningkatnya denyut jantung
2. a. Endoplasmic reticulum +ribosom e. meningkt
b. perikaryon
2. Bahan dasar utk sintesis Ach
3. a. Sel neuron multipolar
b. substantia grisea a. tirosin
b. kolin
4. a. Axon hillock c. ach
b. Nissls substance
d. dopamine
5. a. Canalis centralis e. cholinesterase
b. Sel ependym
3. Integrasi asosiasi pd lobus frontalis..
6. a. Dura mater spinalis
b. jaringan ikat padat/fibrosa e. semua benar..

7. a. Axon 4. Primary motor cortex d bag. Anterior dari


b. Myelin
sulcus adalah area
8. a. perineurium a. broca
b. fasciculus saraf tepi b. wernicke
9. a. Nodus Ranvier c.pyramidal
b. mempercepat penjalaran impuls d, ekstra pir
e. somatosensory
10. a. Sel Schwann
b. membentuk myelin
5. Hipotsalamus mengontrol langsung :
11. a. Sel pyramidalis a. aktivitas jantung
b. Cerebrum
b. aktivtis sekeresi hipofise
12. a. Plexus choroideus c. release corticol dari cortex adrenal
b. menghasilkan CSF d. motilitas lambung
13. a. Astrocyte protoplasmic e. frek. Dan kedalaman nafas
b. Substantia grisea (CNS)
6. Efek stimulasi pada serat saraf kolinergik
14. a. Microglia
b. fagositosis debris di CNS lambung
a. meningkatkan motilitas dan tonus
15. a. Nucleolus b. kontraksi spinchter
b. Sel ganglion dari ganglion spnls
c. menghambat kelenjar e. hipotalamus.
d. menurunkan motilitas
e. menurunkan reflek defekasi 15. Kerusakan pada basal ganglia menyebabkan
a. Gejala Parkinson
7. Fungsi lobus occipital b. Mudah tidur
a. penglihatan
c. Tidak bs bergerak
b. warna 8
d. Wajah tidak berekspresi
c. gerak
e. Insomnia
d. lap. Pandang

8. Yang bukan reflek parasimpatis 16. Bag. Lobus otak utk pemahaman bahasa
a. barometer a. hemisfer cortex
b.menelan b. parietal
c. batuk c. temporal
d. akselerasi jantung d. occipital
e. kencing e. serebelum

9. Pada orang normal, saat berjalan terlihat 17. Skeletal muscle diatur oleh (somatic nervous
refleks System)
a. stretch
b.inverse strestc
18. Aktivitas Ach dalam digesti dan absorbsi
c. flexor
makanan dalam system pencernaan makanan
d. inverse stretch+stretch
adalah.
e.LUPA

10. Efek saraf simpatis 19. Stimulasi parasimpatis


a. meningkatkan sekresi adrenal a. peningkatan denyut jantung
b. meningkatkan kel keringat b. peningkatkan sekresi kelenjar
c. meningkatkan rennin intestine
d. meningkatkan fatty acid c. peningktan motillts
e. semua benar
20. Area yang bertanggung jawab merencanakan
11. Kerusakan pada orbitofrontal dan berfikir adalah
a. frontal
b. somatosensory
12. Yang bukan termasuk system limbic c. post central gyrus 312
a. amygdale d. premotor
b. hippocampus
c. area wernicke 21. Fgsi pusat reflek otonom utk medullary
d. bag. Thalamus dengan mengkontrol
e. bag. Hipotalamus a. fgsii jantung
b. fgsi paru
13. Pusat control pd otak manusia terletak
c. fgsi sirkulasi
a. frontal cortex d. fgsi lambung
b. supplemental premotor area e. semua benar
c. limbic system
d. brainstem
22. Area utk berfikir dan merencanakan
e. broca
a. visual cortex
b. 1,2,3 post central gyrus
14. Pusat rasa kacau dan cemas di system limbik
c. Premotor cortex
a. hippocampus
d. Frontal cortex
b. thalamus
e. Limbic cortex
c. amygdale
d. post central gyrus
23. Merupakan nervus yang berasal dari m. b. Supplemental premotor cortex area.
oblongata c. Limbic system.
a. VI d. Brainstem.
b. VII e. Broccas area.
c. VIII
d. IX 6. Nosiseptor sesuai dengan di bawah ini :
a. Rasa/pengecap dalam mulut. 9
b. Bau-bauan dari hidung.
c. Kerusakan jaringan oleh karena fisik/kimiawi.
d. Kadar O2 darah arteri dan CO2 jaringan.
e. Faktor-faktor lain yang menyusun keadaan
kimiawi tubuh.

7. Rangsangan-rangsangan yang merusak


NEUROPHYSIOLOGY 2 jaringan menyebabkan pelepasan :
a. Endorphin.
b. Histamin dan prostaglandin
1. Sekresi terminal saraf post ganglion
c. TXA.
parasimpatis :
d. Calcium.
a. NE.
e. Macrofag.
b. Dopamin.
c. Ach.
8. Pada amputasi kaki terjadi,KECUALI :
d. Serotonin.
a. Phantom Limb menurut hokum proyeksi.
e. Epinefrin.
b. Sensasi nyeri dan propioseptif pada anggota
tubuh yang sehat.
2. Bahan kimia transmisi antara pre dan post
c. Ujung saraf sehat membentuk neuron.
ganglion
d. Pelepasan histamine.
sympathetic :
e. Pelepasan Endorphin.
a. Adrenal.
b. Nor-Adrenal.
9. Yang mempengaruhi ketajaman
c. Dopamine.
diskriminatif/acuity
d. Acetylcholine.
A. Kepadatan reseptor
e. Serotonin.
B. Influx natrium.
C. Inhibisi lateral.
3. Efek stimulasi nor-adrenergik pada jantung
D. Influx kalsium.
adalah
a. Menurunkan kontraksi jantung.
10. Hal-hal tersebut di bawah ini yang berkaitan
b. Menurunkan denyut jantung.
dengan persepsi :
c. Meningkatkan kontraksi jantung.
A. Interpretasi sadar tentang dunia luar.
d. Relaksasi atrium.
B. Diciptakan otak dari pola impuls saraf ke
e. Relaksasi ventrikel.
otak oleh reseptor sensorik.
C. Korteks sering menambah/mendistorsi
4. Efek stimulasi serabut parasimpatis pada organ
informasi dan otak melengkapi gambaran.
tubuh
manusia adalah :
11. Membran potensial istirahat adalah :
a. Dilatasi pupil.
a. Selisih potensial di sebelah luar dikurangi
b. Peningkatan denyut jantung.
potensial sebelah dalam membrane.
c. Peningkatan kelenjar intestine.
b. Beda potensial antara sebelah dalam
d. Peristaltik intestine meningkat.
terha
e. Peningkatan tekanan darah.
sebelah luar membrane sel.
c. Jumlah potensial di sebelah luar dan di
5. Pusat kontraksi emosi manusia di otak berada
sebelah
di :
dalam membrane sel.
a. Frontal area cortex.
d. Potensial yang ada di sebelah dan dalam 17. Yang bukan termasuk komponen lengkung
membrane sel. reflex regang:
e. Potensial yang ada di sebelah luar a. Organ golgi.
membran sel. b. Muscle spindle.
c. Serabyut saraf tipe B.
12. Transport aktif ion K adalah : d. Synaps di medulla efferent spinalis.
a. Arahnya dari sebelah da e. Interneuron inhibisi. 10
membran.
b. Menyebabkan permeabilitas ion K 18. Yang bukan termasuk komponen lengkung
konsentrasinya lebih besar daripada luar reflex renggang berbalik :
membran. a. Organ golgi.
c. Sebagai penyebab aksi potensial. b. Muscle spindle.
d. Sebagai penyebab terjadinya depolarisasi. c. Serabut saraf tipe I B
e. Sebagai penyebab terjadinya repolarisasi. d. Synaps di medulla efferent spinalis.
e. Interneuron inhibisi.
13. Hukum ALL or NONE :
a. Berlaku untuk terjadinya depolarisasi. 19. Terjadinya aksi potensial neuron post synaptic
b. Berlaku untuk terjadinya repolarisasi. setelah :
c. Berlaku untuk terjadinya generator a. Aksi potensial pre synaps.
potensial. b. Depolarisasi neuron post synaps.
d. Berlaku untuk terjadinya potensial end c. Repolarisasi neuron post synaps
plate. d. Sumasi excitatory post synaps potensial
e. Berlaku untuk terjadinya aksi potensial. e. Hiperpolarisasi neuron post synaps.

14. Untuk terjadinya aksi potensial : 20. Inhibisi neuron post synaptic terjadi karena
a. Dibutuhkan rangsangan yang mencapai a. Adanya transimser inhibisi
firing b. Adanya peningkatan permeabilitas K di
level. neuron post synaps.
b. Dibutuhkan rangsangan yang mencapai c. Adanya peningkatan permeabilitas Na di
over neuron post synaps.
shoot. d. A dan B benar.
c. Dibutuhkan rangsangan yang mencapai e. A,B dan C benar.
depolarisasi.
d. Dibutuhkan rangsangan yang mencapai 21. Pusat bicara pada otak :
hiperpolarisasi. a. Wernickes area.
e. Dibutuhkan rangsangan yang mencapai b. Brocas area.
isoelektrik c. Visual association area.
d. Somesthetic area.
15. Fase repolarisasi pada aksi potensial serabut e. Somatosensorik area.
saraf :
a. Terjadi peningkatan permeabilitas ion Ca 22. Berikut ini tidak terlihat dalam system limbic.
b. Terjadi penurunan permeabilitas ion Na a. Hipocampus.
c. Terjadi peningkatan permeabilitas ion K++. b. Amygdale.
d. B dan C benar. c. Wernickes area.
e. Terjadi peningkatan permeabilitas CL d. Part of the thalamus.
e. Part of the hypothalamus.
16. Hantaran serabut saraf myeli
a. Saltatory conduction. 23. Fungsi hippocampus pada system limbic :
b. Hantaran meloncat-loncat. a. Menyimpan memori.
c. Propagative direction. b. Berfikir.
d. A dan B benar. c. Merencanakan.
e. A,B,dan C benar. d. Aktivitas motorik.
e. Imaginasi.
d. Pola gerakan kognitif (berbentuk huruf)
24. Peran ganglia basalis lintasan putamen benar,
kecuali :
a. Pola gerakan yang telah dipelajari
b. Menulis (membentukk huruf)
c. Menggunting
d. Membalik telapak tangan dengan cepat 11
e. Memasukkan bola basket

25. Pusat rasa kacau atau takut dalam system


limbic merupakan fungsi yang ada pada :
a. Hippocampus.
b. Thalamus.
c. Amygdala.
d. Post Centralis gyrus.
e. Hipothalamus.
MKDI NEUROLOGY 1
26. Kerusakan pada basal ganglia,dapat
menyebabkan : 1. Dari beberapa penyakit dibawah ini, yang
a. Symptoms of Parkinsons disease. bukan termasuk nyeri kepala
b. Sleepiness. primer maupun sekunder adalah:
c. Inability to moving. A. Tension headache
d. Reduce facial expression. B. Migrain
e. Insomnia. C. Cluster headache
D. Pseudo tumor cerebri
27. Reseptor -1 pada jaringan otot jantung E. Meningoenchepalitis
mampu berikatan dengan :
a. Epinephrine. 2. Penyakit di bawah ini sering menimpa wanita
b. Digoxin. obesitas dengan riwayat mengkonsumsi vitamin A
c. Acetylcholine. dosis tinggi:
d. Caffeine. A. Trigeminal neuralgia
e. Nicotine. B. Benign intracranial hypertension
C. Migrain
28. 2 reseptor yang mengikat asetilkolin adalah : D. Tension headache
a. Nicotinic dan muskarinik reseptor. E. Cluster headache
b. Alpha adrenergic dan beta adrenergic.
c. Alpha-1 adrenergic dan beta 3. Nyeri kepala yang sering disertai dengan
d. Alpha-2 adrenergic dan beta Horner Syndrome adalah:
e. Non-adrenergik dan non-cholinergic receptor. A. Tension headache
B. Migrain
29. Bila seseorang menyentuh rasa C. Cluster headache
hangat/dingin,hal ini akan mengaktivasi : D. Neurinoma acousticus
a. Proprioreceptor. E. Pseudotumor
b. Chemoreceptor.
c. Nociceptor. 4. Nyeri kepala periodik yang berkaitan dengan
d. Exteroreceptor. penyebab factor genetic adalah:
e. Mechanoreceptor. A. Classical migrain
B. Common Migrain
30. Fungsi cerebellum kecuali C. Hemiplegic migrain
a. Penggendalian perubahan gerakan yang cepat D. Cluster headache
b. Penguatan setiap awal gerakan E. Tension headache
c. Mengatur intensitas kontraksi otot pada
perubahan beban 5. Nyeri kepala separo wajah yang khas ditandai
dengan adanya trigger zone adalah:
A. Migrain E. Tiroid
B. Cluster headache
C. Tension headache 10. Pada kasus tersebut, pernyataan di
D. Trigeminal neuralgia bawah ini benar:
E. Benign intracranial hypertension A. Kadar katekolamin rendah
B. Kadar serotonin otak rendah
Kasus I. untuk soal no 6 s/d 7 C. Kadar dopamin otak rendah 12
Seorang muda 35 tahun datang ke Poliklinik saraf D. Kadar asetilkolin rendah
dengan keluhan nyeri kepala separo. Sifat nyeri E. Kadar GABA rendah
berdenyut. Nyeri berlangsung 3-4 jam dan
biasanya sesudah itu reda. Nyeri kepala tersebut 11. Letak lesi pada penderita Ballismus
sering dialami sejak usia remaja pada saat adalah:
menstruasi. Biasanya didahului dengan A. Cortex
pandangan seperti melihat kilat cahaya 40 B. Nucleus Subthalamicus
menit. Mual (+). Muntah(+). C. Thalamus
D. Cerebellum
6. Diagnosa kasus diatas mengarah pada: E. Medula Spinalis
A. Common migraine
B. Classic migraine 12. Obat yang sering menimbulkan secondary
C. Ophtalmoplegic migraine parkinsonism adalah:
D. Basiler migrain A. CDP kholine
E. Trigeminal neuralgia B. Metoclopramid
C. Trihexyphenidil
7. Jika pada anamnesa didapatkan serangan D. Propanolol
terjadi 3x per bulan dan pada pemeriksaan E. Gol. Cephalosporin
didapatkan tekanan darah 170/100, maka terapi
Kasus III untuk soal 13-15
profilaksis yang dianjurkan adalah:
Seorang laki-laki, 25 tahun, mahasiswa, mengeluh
A. Golongan ACE Inhibitor
nyeri pinggang menjalar ke betis kanan setelah
B. Golongan calcium channel blocker
mendorong mobil mogok. Pada pemeriksaan
C. Golongan antihistamin
didapatkan tes Bragard, Sicard, dan Naffziger
D. Golongan -blocker
positif, kelemahan dorsoflexi kaki kanan dan
E. Golongan -blocker
reflek tendon normal.
Kasus II untuk No. 8-10
13. Diagnosa yang paling mendekati
Seorang wanita 65 tahun datang ke praktek
adalah:
dengan keluhan tangan kanan kiri bergetar
A. Fraktur vertebrae lumbal
seperti menghitung uang. Bibirnya juaga
B. Spondylosis lumbalis
gemetar. Penderita masih bisa berjalan tetapi
C. Mialgia
sangat pelan dan dipapah. Wajah kaku seperti
D. HNP lumbalis
topeng dan air liurnya meleleh.
E. Canal stenosis
8. Diagnosa yang paling mungkin adalah:
14. Pemeriksaan penunjang yang tidak
A. Penyakit Huntington
tepat adalah :
B. Penyakit Parkinson
A. Foto lumbosacral AP/lateral
C. Stroke Ringan
B. MRI lumbal
D. Tremor essensial
C. Caudografi
E. Hipertiroid
D. CT Scan lumbal
E. Mielografi
9. Letak lesi pada kasus tersebut:
A. Korteks
15. Letak topis untuk kasus diatas adalah :
B. Sub cortex
A. Mielum segmen T12
C. Substansia Nigra
B. N. Iskiadikus kanan
D. Capsula interna
C. Cornu anterior
D. Radik lumbal C. NSAID
E. Otot lumbal D. Splint
E. Citicoline injection
16.Wanita 50 tahun datang dengan keluhan
kesemutan pada jari 1,2, dan 3 tangan kanan 21.Salah satu tanda polineuropati diabetic:
sejak 2 bulan yang lalu. Kesemutan memberat A. Proksimal lebih berat daripada distal
bila dipakai bekerja seperti mencuci baju atau B. Distal lebih berat daripada proksimal 13
pekerjaan rumah tangga lain. Pasien sering C. Gangguan sensoris sesuai dermatom tertentu
terbangun malam hari karena rasa kesemutan tu. D. Hanya terkena motorik
Kemungkinan letak lesi adalah : E. Nyeri radikular
A. N. Medianus kanan
B. N. Ulnaris kanan 22. Salah satu tanda entrapment n. Radialis:
C. N. Radialis kanan A. Claw hand
D. N. Axillaris kiri B. Policeman's tip
E. N. Radialis kiri C. Drop foot
D. Ape hand
17. Laki-laki 45 tahun mengeluh kesemutan mulai E. Drop hand
pantat kiri menjalar ke betis lateral sampai jari 1
dan 2 sejak satu minggu yang lalu. Keluhan terjadi 23. Laki-laki 40 tahun datang dengan sikap lengan
mendadak setelah mengangkut barang berat. bawah flexi ringan,tangan dorsum flexi + claw
Memberat bila pasien batuk atau bersin. hand dan jari-jari ekstensi. Kemungkinan
Kemungkinan diagnosis etiologisnya adalah : diagnosanya adalah:
A. HNP radiks L4 A. Thoracic Outlet Syndrome
B. HNP radiks L5 B. Erb's palsy
C. Tumor L5-S1 C. Klumpke Dejerine Palsy
D. Spondilosis L4-L5 D. Guyon Canal Syndrome
E. Fraktur kompresi L5 E. Carpal Tunnel Syndrome

18. Wanita 25 tahun mengalami kelemahan 24. Untuk kasus no.23 usulan pemeriksaan
mendadak pada kedua tungkai sejak satu hari apa untuk menegakkan diagnosi:
yang lalu dan diikuti dengan kedua lengan sejak A. EMG
tadi pagi. Satu sebelumnya penderita mengalami B. MRI
sakit flu dan saat ini sudah sembuh. Tidak C. CT Scan
didapatkan keluhan sensoris. Pada pemeriksaan D. Myelografi
didapatkan reflex fisiologis menghilang dan E. Foto cervical
tetraparesis ringan. Kemungkinan diagnosis
penyakitnya adalah : 25. Dari soal no.24, kemungkinan
A. Amyotrophic Lateral Sclerosis etiologinya adalah:
B. Schannoma A. Stroke ICH
C. Guillain Barre Syndrome B. Meningoensefalitis
D. HNP cervical C. Stroke SAH
E. Fraktur vertebra cervicalis D. Tumor otak
E. Stroke trombotik
19. Berikut ini salah satu gejala Bells Palsy:
A. Ptosis 26. Di bawah ini bukan karakteristik nyeri
B. Disartria kepala migren:
C. Lagoftalmus A. Berdenyut
D. Diplopia B. Mengikat
E. Rasa tebal separo wajah C. Unilateral
D. Durasi 3-72 jam
20. Berikut tidak termasuk terapi CTS: E. Disetai fotofobia
A. Kortikosteroid
B. Operasi 27. Terapi nyeri kepala kluster adalah:
A. Oksigen D. Tumor otak
B. NSAID E. Vaskulitis
C. Diazepam
D. Amitriptilin 32.Hal berikut tidak benar untuk cluster
E. Carbamazepin headache:
A. Nyeri unilateral
28. Wanita 25 tahun mengeluh nyeri kepala B. Terapi ergotamine 14
sejak 1 minggu yang lalu. Nyeri C. Didapatkan Horners syndrome
dirasakan seperti mengikat di seluruh D. Profilaxis dengans steroid
bagian kepala. Timbul terutama sore E. Durasi 3 jam
atau malam hari. Membaik setelah tidur
atau beristirahat. Pemeriksaan 33. Terapi tension type headache :
neurologis tidak didapatkan kelainan A. Ergotamin
Kemungkinan diagnosisnya adalah : B. Triptan
A. Cluster headache C. Methysergide
B. Classic migraine D. NSAID
C. Tumor otak E. -blocker
D. Tension type headache
E. Common migraine 34. Seorang laki-laki usia 67 tahun, tremor tangan
kanan waktu istirahat, rigiditas (+), jalan
29. Laki-laki 55 tahun mengeluh nyeri pada lambat dengan langkah kecil, hal ini menunjukkan
daerah rahang kanan sejak 2 bulan lesi di :
yang lalu. Nyerinya tajam seperti A. Pyramidal
ditusuk pisau, muncul sewaktu-waktu B. Extrapyramidal
dan bila penderita menggosok gigi. C. Serebelum
Kemungkinan diagnosis adalah : D. Batang otak
E. Kortex
A. Trigeminal neuralgia
B. Post herpetic neuralgia 35. Diagnosa kasus di atas cenderung :
C. Meralgia parestetika A. Tremor essential
D. Tolosa Hurt syndrome B. TIC
E. Bells palsy C. Syndrome Tourette
D. Penyakit Parkinson
30. Hal berikut benar untuk migren : E. Parkinson Akut
A. Muncul berkelompok
B. Lokasi frontal sampai oksipital 36. Kriteria diagnose penyakit Parkinson
C. Seperti dibor (PP) adalah :
D. Terapi dengan steroid A. Tremor istirahat, regiditas
E. Terapi dengan golongan triptan B. Tremor istirahat, regiditas, Bradikardi
C. Tremor istirahat, regiditas, bradikardi,
31. Laki-laki 25 tahun mengeluh nyeri kepala sejak instabilitas posturnal
1 tahun yang lalu. Awalnya dirasakan di sisi kiri D. Sekurang-kurangnya 2 dari 3 (pada jawaban C )
dan saat ini terasa di seluruh kepala. Nyeri E. Sekurang-kurangnya 2 dari 4 (jawaban C diatas)
terutama di pagi hari saat bangun tidur. 1 minggu
ini sakit kepala memberat dan penderita sering 37. Terapi Penyakit Parkinson pada usia dewasa
muntah-muntah tanpa didahului mual. Penderita muda (< 60 tahun) adalah :
juga mengeluh kelemahan pada sisi kanan tubuh. A. L Dopa
Dari pemeriksaan didapatkan GCS 456, papil B. Dopamin Agonis
edema +/+, parese N. VIII D dan XII D tipe UMN, C. Anti Kolinergik
dan hemiparese dekstra. Kemungkinan penyebab D. COMT Inhibitor
sakit kepala pada penderita ini adalah : E. MAO Inhibitor
A. Stroke
B. Meningoensefalitis 38. Terapi Penyakit Parkinson pada usia
C. Migrain komplikata tua (> 60 tahun) adalah :
A. L Dopa E. Methylprednisolon 30 mg/KgBB i.v
B. Dopamin Agonis
C. Anti Kolinergik 45. Tidak termasuk indikasi operasi pada trauma
D. COMT Inhibitor medulla spinalis adalah :
E. MAO Inhibitor A. Ada fraktur, pecahan tulang menekan medul a
spinalis.
39. Termasuk Dopamin agonis adalah : B. Gambaran nuerologis progresif 15
A. Trihexyphenidol memburuk
B. Selegiline C. Fraktur, dislokasi yang labil
C. Madopar D. Terjadi herniasi discus intervertebralis yang
D. Pramipexol menekan medulla spinalis.
E. Entacapcine E. Waktu operasi yang baik antara 24 jam sampai
dengan 3 Minggu
40. Obat untuk tremor esensial :
A. Trihexyphenidl 46. Penjelasan serebral palsi (CP) yang
B. L Dopa paling benar :
C. Pramipexol A. The progressive neurology disorder in children
D. Propanolol + alprazolam B. Non Disorder
E. COM Inhibitor C. The permanent motor disorder
D. The progressive motor disorder
41.Gejala klini lesi mielum servikal adalah: E. The progressive motor and sensor disorder
A. Hemiparese UMN + Hemi hipestasia
B. Tetraparese UMN + Tetrahipestasia 47. Obat anti spastic pada CP tipe spastic
C. B + Retensi urinea / alvi adalah:
D. C + Atropi otot A. Botox
E. C + Kejang B. Vitamin B
C. Phenitoin
42.Paresis UMN ipsilateral dibawah lesi, gangguan D. Metampiron
eksteroseptif kontralateral, gangguan E. Propofol
propioseptif ipsilateral, dikenal sebagai :
A. Sindroma Spinalis anterior 48. Klasifikasi mayor pada CP :
B. Sindroma Spinalis posterior A. Flaccid
C. Sindroma Brown Squard B. Tremor
D. Sindroma servikal C. Seizure
E. Sindroma konus medullaris D. Athetoid
E. Dysarthyc
43.Pareisi lengan > tungkai, dissosiasi sensibilitas,
disfungsi miksi, defekasi dan seksual, 49. Salah satu terapi paling tepat untuk
menunjukkan syndrome : hidrosefalus adalah :
A. Sindroma Spinalis anterior A. Antibiotics
B. Sindroma Spinalis posterior B. Analgetics
C. Sindroma Brown Squard C. Azetasolamid
D. Sindroma Spinalis sentral D. Shunting
servikal E. Corticosteroid
E. Sindroma konus medullaris
50.Berdasarkan edisi terakhir DSM-IV, kriteria
44. Bila trauma medulla spinalis kurang dari 3 jam yang harus ada pada diagnosis retardasi mental
maka terapi : adalah
A. Dexametason 5 mg/iv A. an IQ bellow 90
B. Antibiotika B. significant limitations in two or more areas of
C. Methylprednisolon 5 mg/KgBB i.v bolus selama adaptive behavior
15 menit C. evidence that the limitations became apparent
D. Methylprednisolon 10 mg/KgBB i.v bellow the age of 14
D. Depression
E. The others of mental illness
6. bila didapatkan tetraparese tipe LMN
kemungkinan penyebabnya adalah:
a. tumor mielum
b. stroke batang otak
c. meningitis
d. hiponatremia 16
e. polineuropati

7. yang termasuk susunan saraf perifer adalah:


a. kortek serebri
b. batang otak
c. serebelum
d. mielum lumbal
MKDI NEUROLOGY 2 e. radik

8. yang merupakan pusat tidur adalah:


1. Istilah untuk abnormalitas pada beberapa saraf
a. hipocampus
disebut:
b. hipotalamus
a. polineuropati
c. cerebellum
b. neuropati
d. tuberomamillary
c. mononeuropati
e. ARAS
d. poliradikulopati
e. mononeuritis multiplek
9. neurotransmitter untuk tidur adalah:
a. adenosin
2. fungsi dari lobus temporalis adalah:
b. histamin
a. pusat motorik
c. PGE2
b. pusat sensoris
d. Orexin
c. pemahaman berbahasa
e. Adrenalin
d. pusat penglihatan
e. koordinasi dan keseimbangan
10. yang benar tentang stadium REM adalah
a. empat tahap
3. ciri gangguan pada otot adalah sebagai berikut:
b. sulit dibangunkan
a. tonus otot meningkat
c. mudah diingat
b. reflek fisiologis meningkat
d. disorientasi dan bingung
c. reflek patologis positif
e. terjadi pada menit-menit pertama
d. tidak didapatkan atrofi
e. tidak ada gangguan sensoris
11. yang benar tentang insomnia adalah
a. menurun sesuai umur
4. ciri gangguan pada batang otak adlah:
b. kronik bila lebih 14 minggu
a. hemiparese satu sisi tubuh
c. akut insomnia berhubungan dengan stress
b. hemiparese alternan
d. jarang terjadi di masyarakat
c. gangguan koordinasi dan keseimbangan
e. tidak berhubungan dengan siklus sikardian
d. tetraparese
e. tidak melibatkan saraf cranial
12. yang berhubungan dengan narkolepsi adalah
a. insomnia
5. Pada SIVE, yang termasuk proses sistemik
b. katatonia
adalah:
c. OSA
a. herniasi otak
d. cardiac arrest
b. wernick ensefalopati
e. ritalin
c. tumor otak
d. stroke perdarahan
13. yang termasuk mekanisme injur otak primer
e. hidrosephalus
adalah
a. hipoperfusi a. respon membuka mata (E), Verbal (V), sensorik
b. hipoksia (S)
c. reperfusion injury b. respon membuka mata (E). Verbal (V),
d. kompresi motorik (M)
e. gangguan asam basa c. skor 4 untuk respon membuka mata spontan
d. skor 5 untuk respon pembicaraan
14. yang tidak termasuk basic neurology life membingungkan 17
support adalah e. skor 5 untuk respon sensorik normal
a. approach to status epilepticus
b. spinal cord emergency conditions
c. respiratory failure due to neuromuscular 21. Ny. XY usia 72 thn dibawa ke UGD anaknya
disease dengan penurunan kesadaran. Beberapa hari
d. miopati sebelumnya, pasien mengeluh merasa tidak
e. ischemic stroke sehat, febris, dan sesak nafas. Hasil pemeriksaan
lab: hitung leukosit 160000/mm3 (lekositosis).
15. nyeri kepala yang tergolong paling BUN 100mg/dL dan kreatinin 6,3 mg/dL
emergensi adalah (gangguan fungsi ginjal). Foto thorax: pneumonia.
a. nyeri kepala kronik progresif Diagnosis etiologis dari coma yang diderita pasien
b. nyeri kepala hilang timbul adalah:
c. nyeri kepala dengan mual a. gagal ginjal
d. nyeri kepala dengan parkinson b. gangguan elektrolit
e. nyeri kepala tipe tension c. lekositosis
d. gagal ginjal dan hypoxic askemis
16. tindakan pertama pada pasien koma adalah: encephalopathy
a. cek darah e. infeksi dan lekositosis
b. infus PZ
c. CT scan kepala 22. tipe coma dibawah ini disebabkan oleh
d. EKG hipoksia atau anoksia:
e. ABC a. coma dengan herniasi
b. coma kortikal bihemisfer
17. pemberian terapi pertama pada pasien koma c. koma neurologis
adalah: d. koma supratentorial
a. bolus fenitoin e. koma infratentorial
b. O2 lewat NGT
c. tiamin 23. tipe respirasi pada pasien com dengan lesi di
d. infus D5% pons adalah
e. manitol 10% a. cheyne-stokes respiration (diencephal)
b. central neurogenic hyperventilation (mesen)
18. coma disebabkan oleh gangguan pada: c. cluster breathing
a. cortex bihemisfer d. ataxic breathing (medulla)
b. cortex tersier e. apnea
c. ARAS
d. Pons 24. tanda herniasi otak tipe uncal dengan lesi di
e. ARAS and cortex bihemisfer mesensefalon
a. cheyne stokes respiration, small
19. penyebab koma bihemisfer adalah: reactive pupils, gegenhalten
a. encephalopathy b. CNH, pupil anisocoria, decorticate
b. ICH rigidity
c. tumor metastase c. ataxic breathing, pin point pupil,
d. head trauma hyperthermia,
e. abcess cerebri d. apneustic, midriasis, flaccid atonic
e. apnea, anisocoria, atonic
20. glasgow coma scale (GCS) meliputi
25. seorang laki2 usia 17 thn dibawa oleh ayahnya e. keadaan yang ditandai oleh seizure umum
ke Unit Gawat Darurat dengan riwayat kejang 3
kali. Pertama kejang terjadi 1 bulan lalu, kedua 3
minggu lalu dan ke-3, 1jam yang lalu. Pada 30. epilepsi syndrome ditandai oleh sekelompok
awalnya terjadi sentakan pada kedua kaki gejala dan tanda yang terjadi bersama-sama sbb
kemudian disusul kejang umum, berakhir 2-3 a. type of seizure, etiologic, anatomy, age of
menit, bingung dan tertidur. Diagnosis kasus di onset, severity, prognosis 18
atas adalah: b. type of seizure, mioklonik, tonic, clonic, atonic
a. epilepsi absans c. type of seizure, severity, conciousness,
b. epilepsi tonik-klonik hemiplegia
c. epilepsi mioklonik d. afasia, disartria, paraplegia
d. epilepsi klonik e. type of seizure, bradikinesia, tremor, rigidity
e. epilepsi tonik
31. tanda dan gejala seizure absans sbb:
26. penyebab terbanyak epilepsi pada soal di a. unaware, bengong, berhenti mendadak
atas adalah: semua aktivitas motorik, EEG : 3 Hz spike and
a. tumor otak wave
b. ensefalopati-metabolik b. unaware, tonic-clonic type, EEG, poly spike
c. idiopatik wave
d. trauma c. unconciousness, behaviour changes,
e. arterio-venous malformation EEG: spike and wave discharge
d. unconciousness, behaviour changes, EEG: spike
27. prinsip pengobatan pada penderita epilepsi and wave discharge
adalah e. unaware, bengong, berhenti mendadak semua
a. pengobatan jangka panjang, polifarmasi aktifitas motorik, EEG: polyspike wave
b. pengobatan jangka panjang, monofarmasi
c. pengobatan jangka panjang, monofarmasi, 32. tanda dari epileptic Cry terjadi pada
selama 3 thn a. simple partial seizure (temporal lobe
d. d, pengobatan jangka panjang, obat bagus epilepsy)
dan mahal b. absence epilepsy
e. pengobatan jangka pendek, monofarmasi c. general tonic-clonic epilepsy
d. myoclonic epilepsy
28. penghentia OAE yang benar: e. atonic epilepsy
a. dilakukan seketik setelah kejang berhenti
b. diantara kejang penderita tidak sadar 33. manifestasi klinis dari epileptic seizure
c. obat tambahan (bukan obat utama) terdiri dari
dihentikan pertama kali a. gangguan kesadaran, motorik, sensorik,
d. EEG abnormal autonomic, psikis
e. diagnosa meragukan b. gangguan kesadaran, sensorik dan
psikis
29. definisi epilepsi adalah sebagai berikut c. hemiparesis, kejang, dan afasia
a. kondisi yang ditandai dengan seizure berulang d. bentuk motoris, sensoris, kejang,
(2 x atau lebih) otonomik, psikis
b. kondisi neurologis kronis dengan ciri-ciri e. hanya kejang
seizure epileptic berulang
c. suatu manifestasi klinis yang dihasilkan dari 34. laki2 73 thn, dibawa ke IRD karena jatuh
cetusan abnormal dan berlebihan dari setelah dari kamar mandi, ditemukan sadar, tidak
sekelompok neuron di dalam otak dengan bisa meggerakan tubuh sebelah kanan, tensi
fenomena abnormal yang mendadak dan 180/100, nadi 80x/m dan regular. Pemeriksaan
sementara neurologi adalah sadar, hemiparese kanan
d. seizure yang mengindikasikan atau konsisten dengan parese n VII dan XII kanan, afasia
danya aktivasi dari sebagian atau satu hemisfer (mengerti tapi tidak bisa bicara), dimana letak lesi
otak (diagnose topis)nya?
a. kortex serebri (hemisfer kiri) d. polyneuritis
b. mesensafalon kiri e. myasthenia gravis
c. pons kiri
d. medula oblongota kiri 40. kepribadian, respon emosi dan fungsi social
e. medula spinalis servikal adalah fungsi dari lobus otak:
a. frontal lobe
35. bila seorang laki2 50th dengan hemiparesis b. paretal lobe 19
kiri UMN, parese VII LMN dan XII UMN,dimana c. temporal lobe
letak lesinya? d. occipital lobe
a. korteks serebri (hemisfer kanan) e. limbic
b. kapsula interna kanan
c. mesensefalon kanan 41. seorang laki2 55thn penjual bubur ayam
d. pons kanan tampak agak aneh dalam menangani
e. medula oblongota kanan keuangannya. Kadang2 sepiring buryam dihargai
Rp. 5000,- kadang Rp. 6000,-. Suatu ketika 1 porsi
36. seorang wanita 40th mengeluh 3 bulan ini hanya diberikan separuh porsi. Sepintas orang ini
parestesi atau kesemutan pada jari ke 1,2,3,4 sisi tampak baik-baik saja. Ketika ditanya apakah
radial. Atrofi otot tenar. Tinnel dan phalen positif. sering lupa,maka jawabannya polos lupa sih
Dimana letak lesi? kagak, cuman nggak inget dari kasus tersebut,
a. kornu anterior penjual buryam kemungkinan mengalami:
b. radik C6, 7 a. mild cognitive impairment
c. N.ulnaris b. demensia
d. N. Radialis c. delirium
e. N. Medianus d. amnesia
e. parknison disease
37. seorang laki2 mengeluh nyeri pinggang. Nyeri
sudah 2 hari, tidak pernah trauma, demam dan 42. bapak 60th, sarjana, beberapa thn
nyeri sendi. Nyeri dengan tanda Lasseque positif mengalami kepikunan yang progresif, selalu
kanan, reflek tendon ankle menurun, tebal pada tampak gelisah, tidak tenang, berkali2
betis luar sebelah kanan. Dimana lesinya? mengatakan sudah,sudah, sudah, keluarga
a. kornu anterior merasa kewalahan karena dia ada kecenderungan
b. radik L5 atau kemauan yang tak terkendali. Emosinya
c. pleksus lumbosacral tinggi, labil, marah kalau kehendaknya tidak
d. N. Ischiadicus dituruti. Evaluasi kognitif sangat buruk. Secara
e. otot paha dan betis fisik tidak ada kelainan. Kemungkinan yang
dialami bapak tersebut adalah:
38. seorang wanita 40 th mengeluh kadang- a. mild cognitive impairment
kadang penglihatan dobel dan kelopak cenderung b. demensia ringan
menutup. Sudah 3 bulan ini diplopia intermitten c. demensia sedang
(melihat dobel) ketika melihat tv. Kelopak mata d. demensia berat
bertambah jatuh pada malam hari, tapi normal e. gangguan jiwa
pada pagi hari. Dimana letak lesinya?
a. N. Optikus (N. II) 43. yang manakah merupakan salah 1 kriteria
b. b.batang otak mild cognitive impairment:
c. otot orbita a. lupa sementara
d. Nn. III (oculomotor), IV (trochlearis), VI b. gangguan memori jangka pendek
(abducens) ringan
e. neuromuscular junction c. hanya mengingat fakta2 penting
d. gangguan aktifitas sehari2
39. penyakit pada soal no 38 adalah e. kecemasan
a. neuritis opticus
b. stroke 44. dementia dibwah ini adalah dementia
c. myositis irreversible:
a. alzheimer dementia kata tidak bisa, tes Naming tidak bisa, maka
b. parkinson dementia jenis gangguan berbahasa:
c. dementia due to hydrocephalus a. afasia sensoris
d. dementia due to depressive state b. afasia motoris
e. A and B c. afasia global
d. afasia anomik
45. kriteri dari probable alzheimer dementia: e. afasia amnesik 20
a. defisit memori progresif dan satu
area kognisi lain 50. seorang laki2 67 thn, tremor tangan kanan
b. tidak ada gangguan kesadaran waktu istirahat, rigidiatas (+), jalan lambat
c. defisit neurologist (+) misalnya dengan langkah kecil, hal ini menunjukkan
hemiplegi or aphasia lesi di:
d. penyakit aterosklerosis (+) a. pyramidal
e. A+B b. extrapyramidal
c. serebelum
46. kriteria of AAMI (age associated memory d. batang otak
impairment): e. kortex
a. usia minimal 50th, ADL (activity of daily living)
sederhana normal, fungsi kognitif normal
b. defisit normal menetap
c. skor of MMSE < 24
d. tidak ada tanda dementia
e. gangguan ringan pada ADL kompleks atau
IADL (instrumental activity of daily living)

47. seorang wanita 58th mengalami sering


lupa,yang semakin berat secara progresif. Ia tidak
mampu berpakaian atau makan sendiri, juga tidak
mampu melakukan aktifitas sehari-hari.
Seseorang harus membantunya. Kadang2 ia
marah tanpa sebab. Secara fisik tampak normal.
Bila dokter menanyakan beberapa pertanyaan ia
selalu menjawab dengan tersenyum, tetapi
jawabannya selalu salah. Kemampuan
memberikan pertimbangan dan memberi alasan
tidak logis. Diagnosisnya adalah:
a. psikosis
b. schizophrenia
c. dementia
d. mild cognitive impairment
e. kecemasan

48. kriteria dementia menurut DSM IV:


a. gangguan memory jangka pendek
dan jangka panjang (+)
b. kemampuan untuk memberikan
pertimbangan normal
c. gangguan kesadaran ringan (+)
d. penyakit sistemik (+)
e. A+B

49. bila penderita sulit bicara, bisa mengikuti


perintah, pemahaman bahasa baik, menirukan
c. hypercholesterolemia
d. DM
e. life style salah

5. faktor resiko untuk stroke emboli adalah


a. usia
b. hypertensi berat 21
c. hypercholesterol
d. DM
e. penyakit katub jantung

6. pemeriksaan tambahan untuk menentukan


diagnosis stroke adalah
a. lab lengkap
b. ECG
c. foto thorax
MKDI NEUROLOGY 3
d. CT scan
e. Arteriografi serebral
KASUS I (NO 1-7)
Seorang laki-laki umur 62 tahun, pagi bangun 7. dosis aspirin untuk stroke trombosis
tidur tiba-tiba lemas tubuh kanan, sulit bicara, adalah
pada pemeriksaan fisik pasien sadar, parese, N.VII a. 80-300 mg / hari
& XII kanan, sakit kepala (-), muntah (-), tensi b. 400-600 mg / hari
160/90 mmHg. c. 800-1000 mg / hari
d. 1000-1200 mg / hari
1. Hemiparese kanan tipe UMN + parese N.VII di e. 1500 mg
kanan + afasia motoris menunjukan
lesi di KASUS II (NO 8-12)
a. kortex Seorang laki-laki 48 tahun, waktu aktivitas di
b. sub kortex kantor jam 10 tiba-tiba sakit kepala (+), muntah
c. batang otak (+), lumpuh kanan (+) bicara pelo, mulut merot,
d. medula spinalis parese N VII & XII kanan , tekanan darah 200/100
mmHg.
e. serebelum
8. hemiplegi dextra + parese N VII & N XII
2. pada kasus I di atas diagnosis etiologis dextra + dysartria , menunjukkan lesi di
cenderung a. kortex
a. stroke perdarahan intraserebral b. sub kortex
b. stroke trombosis c. batang otak
c. stroke emboli d. serebelum
d. stroke perdarahan subarachnoid e. medula spinalis
e. TIA 9. diagnosa pada kasus II cenderung suatu
a. stroke trombosis
3. faktor resiko utama stroke perdarahan b. stroke perdarahan intraserebral
intraserebral adalah c. stroke emboli
a. usia d. stroke perdarahan subarachnoid/SAH
b. hypertensi berat e. TIA
c. hypercholesterolemia
10. terapi pada fase akut kasus II di atas
d. DM
adalah
e. penyakit katub jantung a. tensi diturunkan sampai < 220/110
b. tensi diturunkan sampai < 200/110
4. di bawah ini bukan merupakan faktor c. tensi diturunkan sampai < 180/100
resiko untuk stroke thrombosis d. tensi diturunkan sampai < 160/90
a. Usia tua e. tensi diturunkan sampai < 120/80
b. AVM
11. penyebab kasus II di atas, terbanyak e. ibuprofen
adalah
a. TBC paru KASUS III (NO 18-22)
b. hipertensi Seorang laki-laki 45 thn datang ke dokter,
c. virus mengeluh nyeri kepala berat, dan demam selama
d. AVM atau aneurysme pembuluh darah otak 8 hari. Ia juga mengalami kejang 2 kali sehari,
e. kelainan katub jantung masing-masing selama 2 menit. Dia tidak sadar22
selama kejang. Pada pemeriksaan tensi 130/80
12. seorang wanita 26 tahun tiba-tiba sakit mmHg. Suhu tubuh 40C dan kaku kuduk.
kepala hebat, muntah +, kaku kuduk tanpa
lumpuh separuh tubuhnya. Diagnosa cenderung 18. pada pemeriksaan neurologist ditemukan
a. meningitis penurunan kesadaran (GCS 225), diagnosis yang
b. stroke emboli paling mungkin adalah
c. stroke perdarahan subarachnoid a. perdarahan subarachnoid
d. encephalitis b. abses otak
e. stroke perdarahan intraserebral c. encefalitis
d. tumor otak
13. NSAID di bawah ini dapat digunakan sebagai e. meningoencephalitis
obat antitrombotik
a. asam mefenamat 19. letak lesi pada kasus III adalah
b. sodium diklofenak a. subarachnoid
c. aspirin b. cortex cerebri
d. piroksikam c. sub cortex
e. ibuprofen d. meningen, enchepalon
e. lateral ventricle
14. berapakah dosis harian aspirin yang
dibutuhkan sebagai antitrombotik?
a. 1000 mg 20. dalam perjalanan penyakit dari pasien pada
b. 500 mg kasus III, pasien mengalami hemiplegi
c. 400 mg diagnosisnya
a. meningitis
d. 250 mg
b. cerebral abses
e. 160 mg
c. Brain tumor
d. encephalitis
15. pernyataan di bawah ini TIDAK benar
e. stroke perdarahan intraserebral
mengenai tromboksan A2 (TxA2) yaitu
a. merupakan salah satu bentuk
21. pemeriksaan berikut berguna untuk
eikosanoid
b. merupakan vasodilator yang poten menetapkan diagnosis definitive pasien pada
c. dapat menginduksi agregasi pertanyaan no 20
platelet a. pungsi lumbal
d. efek pada platelet dibatasi oleh b. foto sinar x pada kepala
prostasiklin c. CT scan dengan kontras
e. merupakan signal amplifikasi d. EEG
untuk trombin dan ADP e. EMG

16. ekskresi salisilat bebas melalui urine 22. hasil pemeriksaan likuor pada pasien dengan
tergantung pada meningitis bacterial ditemukan
a. pH urine a. glucose menurun, PMN menurun, protein
b. produksi urin menurun, keruh
c. frekuensi kencing b. glucose menurun, PMN menurun,protein
d. metabolisme salisilat meningkat, keruh
e. paruh waktu salisilat c. glucose menurun, PMN meningkat,protein
meningkat, keruh
17. salisilat berkompetensi dengan substansi d. glucose menurun, PMN meningkat,protein
yang bervariasi berikut ini untuk berikatan meningkat, jernih
dengan protein plasma, kecuali e. glucose meningkat, PMN meningkat, protein
a. tiroksin meningkat,jernih
b. penisilin
c. fenitoin KASUS IV (NO 23-24)
d. nafroksen Seorang perempuan dibawa ke Unit Gawat oleh
suaminya karena ia tidak sadar. Ia mengalami diberikan untuk meningitis supuratif pada bayi
nyeri kepala dan demam sejak 10 hari yang lalu. usia kurang dari 1 bulan
Pada pemeriksaan: tekanan darah 110/80 mmHg, a. ampicilin plus dexamethasone
temperature 40C dan kaku kuduk (+). Riwayat b. ampicilin plus cefotaxime
medis terdahulu karies beberapa gigi belum c. cefotaxime plus vancomycin
dirawat. d. chloramphenicol plus gentamicin
23. yang manakah mekanisme yang terjadi pada e. gentamicin plus ceftazidime
penyakit tersebut (kasus IV) 23
a. percontinuitatum
b. hematogen 30. seorang penderita laki-laki 30 thn datang
c. lymphogen berobat ke RS dengan keluhan sulit tidur,
d. metastatic malas bekerja, sering mendengar suara
e. direct contact orang banyak mengomentari dirinya,
padahal waktu itu tidak ada orang. Pada
24. kontraindikasi pungsi lumbal adalah pemeriksaan intrinsik, neurologik serta
a. nyeri kepala laboratorium tidak didapatkan kelainan.
b. demam Kemungkinan gangguan jiwa yang
c. papil edema diderita pasien tersebut
d. kaku kuduk a. depresi
e. coma b. gangguan afektif
c. neurosa
d. skizofrenia
25. pernyataan di bawah ini yang tidak benar e. mania
mengenai terapi meningitis bacterial adalah 31. seorang penderita laki-laki 35 thn,
a. sefalosporin generasi 1 sulit melakukan pekerjaan tani, pendidikan SD tidak tamat
penetrasi ke dalam cairan cerebrospinal datang ke Puskesmas dengan keluhan
b. vankomisin tidak boleh digunakan rutin utama gaduh gelisah. Mondar-mandir,
dalam terapi meningitis bakterial bicara melantur, kedua tangannya diikat,
c. antibiotika untuk meningitis bakterial diberikan dan diantar oleh orang banyak. Pada
secara intravena pemeriksaan lebih lanjut tidak ditemukan
d. dosis antibiotika dapat diturunkan apabila adanya kelainan organik, tetapi ditemukan
telah ada perbaikan klinis waham dan halusinasi yang cukup
e. antibiotika harus segera diberikan tanpa menonjol. Pada penderita tersebut obat apa
menunggu hasil kultur bakteri yang paling tepat diberikan
a. chlorpromazine
26. antibiotika di bawah ini dapat melalui blood b. estazolam
brain barrier yang tidak mengalami radang c. diazepam
a. amikasin d. alprazolam
b. amfoterisin B e. amitriptilin
c. gentamisin 32. seorang wanita 26 thn datang ke RS
d. isoniazid dengan keluhan marah-marah tanpa sebab,
e. klindamisin gelisah, bila ditanya jawabannya tidak
sesuai dengan pertanyaaan selama 2 hari
27. pilihan terapi untuk meningitis karena sebelum ke RS. Penderita sering kejang
metichilin-ressistant staphylococcal atau penicilin- selama 11 tahun, frequensi kejang 2-3 kali
ressistant S. Pneumoniae adalah per bulan, 1 minggu sebelum MRS,
a. kloramfenikol penderita kejang selama 15 menit, tensi
b. ciprofloksasin 110/70 mmHg, nadi 100x/menit,
c. vankomisin pernafasan 24x/menit, suhu 36,7C.
d. ampisilin Selain kelompok obat anti epileptika, pada
e. cefotaxime penderita ini diberikan obat apa?
a. alprazolam
28. pilihan terapi untuk meningitis karena Listeria b. impramine
c. haloperidol
a. ampisilin
d. diazepam
b. ciprofloxacin
c. vancomycin e. amitriptiline
d. ampicilin
e. cefotaxime 33. seorang wanita 30 thn datang ke RS dengan
keluhan marah-marah tanpa sebab, gelisah, bila
29. terapi antibiotika empiris yang dapat ditanya jawabannya inkoheren dengan
pertanyaaan selama 3 hari sebelum ke RS.
Penderita sering kejang selama 15 tahun, 38. pada psikosa karena gangguan mental organik
frequensi kejang 2-3 kali per bulan, 5 minggu yang akut dapat ditemukan semua di bawah ini,
sebelum MRS, penderita kejang selama 10 menit, kecuali
tensi 125/80 mmHg, nadi 92x/menit, pernafasan a. demensia
20x/menit, suhu 37oC. Pemeriksaan neurologik b. amnesia
yang dibutuhkan penderita ini c. delirium 24
a. CT Scan d. halusinasi
b. MRI e. kesadaran menurun
c. EEG
d. SPECT (single photon emission 39. gangguan psikotik yang mirip skizophrenia
computed tomography) paranoid diakibatkan oleh pemakaian zat
e. PET (positron emission tomograph) a. pil koplo
b. sabu
34. seorang laki-laki 45 thn datang ke RS dengan c. bir/alkohol
keluhan takut, kadang-kadang seperti melihat d. ganja
keranda mayat di depannya, kadang-kadang di e. putaw
dalam hati ada suara mati kamu, mati kamu 3
minggu ini penderita tidak masuk kerja dan 40. gangguan mental organik artinya
kadang-kadang memegangi istrinya bila rasa a. terdapat kerusakan organik pada
takutnya muncul, tensi 130/80 mmHg, nadi otak
80x/menit, pernafasan 20x/menit, suhu 36,7C. b. adanya gangguan fungsi otak
Pengobatan yang paling baik apa? c. adanya gangguan neurotransmitter
a. obat antidepresi di otak
b. obat anticemas d. gangguan mental yang diketahui
c. neuroleptika disebabkan oleh suatu penyakit
d. terapi elektro konvulsi badaniah
e. psikoterapi e. sekumpulan gejala psikiatrik
dengan kesadaran menurun
35. yang betul mengenai kesadaran pada
psikosa fungsional adalah 41. yang membedakan psikosa dengan neurosa
a. berkabut adalah
b. menurun a. keluhan psikologis
c. meninggi b. keluhan somatis
d. terganggu secara kuantitatif (rangsangan c. kehilangan sense of reality
yang kuat dengan respon lemah) d. adanya stressor
e. terganggu secara kwalitatif (rangsangan dan e. tidak satupun di atas ini
respon tidak konsisten dan tidak cocok)
42. yang benar tentang neuroleptika dosis efektif
36. yang dibawah ini merupakan gejala kecil adalah
scizophrenia, kecuali a. obat yang efek sampingnya kecil
a. hipokondria b. dapat untuk mengobati epilepsi
b. demensia c. cocok untuk anak kecil
c. respon emosi bizar d. mempunyai efek antiemetik
d. gangguan proses berpikir e. contohnya trihexifenidil
e. kemauan yang ambivalen
43. pernyataan di bawah ini benar untuk
37. penyulit yang terjadi pada pasien scizophrenia neuroleptika, kecuali:
a. bunuh diri a. termasuk golongan psikotropika
b. membunuh orang lain b. mempunyai efek anticemas
c. menelantarkan diri c. disebut juga tranquillizer minor
d. semua benar d. mempunyai efek anti agitasi
e. tidak ada yang benar e. contohnya flufenazine
44. yang tersebut dibawah ini anti psikosis atipikal 50. mekanisme kerja utama antipsikosis
, kecuali: adalah
a. sulpiride a. mengeblok reseptor dopamine di
b. clozapine mesolimbic dopamine system
c. risperidone b. mengeblok reseptor dopamin di
d. fluphenazine nigrostriatal dopamine system 25
e. olanzapine c. mengeblok reseptor dopamin di
tuberoinfundibular dopamine system
45. mekanisme kerja obat antipsikotik tipikal d. mengeblok reseptor dopamine di
a. memblok reseptor dopamin D2 post nigrostriatal and tuberoinfundibula dopamine
sinaps system
b. memblok uptake nor epinefrin e. mengeblok reseptor dopamine di seluruh
c. memblok reseptor -adrenergic receptors dopamine system di sistem saraf pusat
d. memfasilitasiefek fisiologis GABA
e. Menghambat enzim MAO
MKDI NEUROLOGY 4
46. pernyataan di bawah ini tidak benar mengenai
obat antipsikotik atipikal
1. Seorang laki-laki 23 tahun dating ke dengan
a. afinitas terhadap reseptor 5-HT2 lebih
keluhan sering tertidur. Kalau tidur sering disiang
besar dibanding dopamin D2
hari saat kuliah, mendadak tidak bisa ditahan dan
b. menyebabkan risiko efek samping
ini terjadi berulang. Pada pemeriksaan fisik semua
ekstrapiramidal yang lebih kecil
normal. Diagnosis yang paling teapat untuk kasus
c. lebih efektif untuk menangani gejala
diatas adalah :
negatif dari psikosis
a. Insomnia
d. lebih efektif untuk menangani gejala
b. Insufficient sleep
positif dari psikosis
c. Narcolepsy
e. tidak menimbulkan toleransi
d. OSA
e. Parasomnia
47. bradikinesa, rigiditas, tremor, muka topeng,
merupakan gejala dari:
2. Terapi untuk kasus diatas adalah
a. parkinsonisme
a. Clonazepam
b. distonia akut
b. Amitriptilin
c. akathisia
c. Gabapentin
d. neuroleptic malignant syndrome
d. Imipramine
e. diskinesia tardive
e. Olahraga
48. obat antipsikotik berikut sebaiknya tidak
3. Hal ini terkait insomnia kronik
diberikan apabila terjadi efek samping
a. Kortikosteroid
ekstrapiramidal
b. Hypersomnia
a. thioridazine
c. Catalepsy
b. clozapine
d. Hyanogogic paralysis
c. haloperidol
e. Methylpenictane
d. risperidone
e. olanzapine
4. Hal ini terkait dengan parasomnia
a. RLS
49. terapi parkinsonisme akibat efek samping
b. Jet Lag
obat antipsikotik
c. Circadian rhythm disorder
a. MAO inhibitor
d. Transient insomnia
b. trihexyphenidyl
e. Somnabullism
c. antihistamine
d. bromokriptin
5. Hal ini berkait dengan patofisiology bangun
e. diazepam
a. Orexin
b. VLPO e. Limbic system
c. PDG2
d. Adenosin 12. Seorang laki-laki 40 tahun tiba sehingga
e. CGS2 tidak berani membuka mata, mual, muntah
disertai rasa panic, nigtasmus (+), pendengaran
6. Seorang wanita 23 thn dating ke poli saraf normal dan 3-5 hari keluhan berkurang. Diagnosis
dengan keluhan sering berputar. Pusing berputar cenderung? 26
1 minggu, mendadak, durasi pusing hanya 1 menit a. BPPV
, lalu menghilang, disertai mual dan muntah. Satu b. Anxiety
minggu sebelumnya mengalami cedera kepala c. Meniere disorder
ringan. Pada pemeriksaan fisik tidak ada kelainan d. Vestibular Neuronitis
dan pendengaran normal. Diagnosis nya adalah e. TIA Vertebro Basilar Ischemia
a. Meniere 13. Terapi kasus di atas pada fase akut adalah :
b. Ototoksik a. Vestibular exercise
c. Vertigo post trauma kepala b. Epley maneuver
d. Neurinitis c. Bedrest, anti vertigo
e. Benign positional vertigo d. Antibiotika
e. Anti virus
7. Patofosiology yang tepat untuk kasus diatas
adalah 14. Seorang wanita 35 tahun mengeluh nyeri
a. Hidrosis endolimp kepala selama 5 bulan ini, hamper tiap hari
b. Proses autoimun terutama di kepala kanan depan dan belakang.
c. Digus aksonal injuri Hilang timbul. Berhubungan dengan insomnia,
d. Otocranial debris cemas dan nyeri epigastrik. Pada pemeriksaan
e. Infeksi virus fisik normal. Etiologi kasus di atas adalah :
a. Migraine with aura
8. Terapi untuk kasus diatas adalah b. Migraine without aura
a. Antibiotic c. Cluster headache
b. Dieresis d. Brain tumor
c. Diet tinggi protein e. TTH
d. Anti ansietas
e. Epply manufer 15. Terapi untuk kasus di atas adalah :
a. Analgetic and anti anxiety
9. Hal2 berkaitan dengan sindrom menire b. Antiserotonin and anti anxiety
a. Relaps c. Ergot alkaloids
b. Headache d. Triptans
c. Inkoordinasi e. Vasodilator
d. Gangguan nervus VII
e. Normal hearing 16. Jika nyeri kepala unilateral, berdenyut durasi 6
dan muntah. Sebelum serangan ada fotofobia,
10. Yang termasuk true vertigo osilopsia. Etiloginya adalah :
a. Central vertigo a. Classic migraine
b. Vasovagal syncope b. Common migraine
c. Disequilibrium c. TTH
d. Blackout spells d. Cluster headache
e. Light headedhness e. Stroke SAH

11. Vertigo berputar, mual, muntah,menunjukan 17. Terapi fase akut untuk kasus diatas adalah :
lesi di : a. Parasetamol
a. Cochlear system b. Topiramate
b. Vestibular system c. Ergot
c. Cardiovascular system d. Vasodilator
d. Cerebellum e. Sodium valproat.
18. Jika nyeri kepala mulai 5 bulan yang lalu, 23. Seorang wanita 40tahun mengeluh kadang
muntah proyektil 2 bulan dan buta 1 bulan yll, diplopia dan kelopak cenderung menutup. Sudah
sudah seminggu kelemahan tubuh kiri, 3 bulan ini diplopia intermitten ketika melihat tv.
diagnosanya adalah : Kelopak mata bertambah jatuh pada malam hari,
a. Migraine tetapi normal pada pagi hari. Dimana letak
b. TTH lesinya? 27
c. Stroke ICH a. N.opticus (N.II)
d. Stroke SAH b. Batang otak
e. Brain tumor c. Otot orbita
d. Nn.III, IV, VI
Laki-laki 73 tahun, dibawa ke IRD karena jatuh e. Neuromuscular junction
setelah dari kamar mandi, ditemukan sadar, tidak
bisa menggeraan tubuh sebelah kanan, tensi 24. Penyakit pada soal no 23 adalah :
80/100, nadi 80x/m dan regular. Pemeriksaan a. Neuritis opticus
neurologi adalah sadar,hemiparese kanan dengan b. Stroke
N VII dan XII kanan, afasia motoris. Diagnose c. Myositis
topisnya adalah : d. Polyneuritis
a. Kortek serebri kiri e. Myasthenia gravis
b. Mesensefalon kiri
c. Pons kiri 25. Kepribadian, respon emosi dan fungsi social
d. Medulla oblongata kiri adalah fungsi dari lobus otak :
e. Medulla spinalis servicalis a. Frontal
b. Paretal
20. Bila seorang laki-laki 50 tahun dengan c. Temporal
hemiparese kiri UMN, parase VII LMN dan XII d. Occipital
UMN dimana letak lesinya? e. Limbic system
a. Kortek serebri kanan
b. Kapsula interna kanan 26. Tanda lesi komplit medulla spinalis thorakal
c. Mesensefalon kanan 4 adalah :
d. Pons kanan a. Kelemahan betis bilateral dengan normal
e. Medulla oblongata kanan b. Kelemahan betis bilateral UMN, kehilangan
semua modalitas dibawah lesi
21. Seorang wanita 40 tahun mengeluh 3 bulan c. Kelemahan betis bilateral LMN, atrofi otot
ini parastesi atau kesemutan pada jari ke 1,2, 3,4 berat
sisi radial. Atrofi otot tenar. Tinnel dan phalen d. Kehilangan semua modalitas di bawah dada
positif. Dimanakah letak lesinya? e. Atrofi otot berat
a. Kornu anterior
b. Radiks c6.7 27. Coma disebabkan oleh gangguan pada :
c. N.ulnaris a. Cortex bihemisphere
d. N.radialis b. Cortex tersier
e. N.Medianus c. ARAS
d. Pons
22. Seorang laki-laki mengeluh nyeri pinggang e. ARAS dan cortex bihemisphere
sudah 2 hari, tidak pernah trauma, demam dan
nyeri sendi. Nyeri dengan tanda lasseque positive 28. Penyebab koma bihemisphere
kanan, reflex tendon ankle menurun, tebal pada a. Encephalopathy
betis luar sebelah kanan. Dimana letak lesinya? b. Intracerebral hemorrhage
a. Kornu anterior c. Tumormetastase
b. Radiks L5 d. Head trauma
c. Pleksus lumbosacral e. Abcess cerebri
d. N.Ischiadicus
e. Otot pada paha dan betis 29. GCS meliputi :
a. Respon membuka mata (E), Verbal (V), e. Apnea, anisocoria, atonic
Sensoris (S)
Respon membuka mata (E), Verbal (V), 35. Laki laki 20 th dibawa ke UGD dalam kondisi
Motoris tidak sadar. 3 jam sebelumnya penderita
b. Respon membuka mata (E), mengalami kejang, dengan keempat ekstremitas
(M) lurus kaku diikuti menekuk berkali kali selama
c. Skor 4 untuk Eye Spontan lebih kurang 1 menit. 28
d. Skor 5 untuk Verbal membingungkan Selama kejang penderita tidak sadar. Setelah
e. Skor 5 untuk respon sensoris normal kejang berhenti penderita masih tidak sadar
30. Bonus dan kemudian kejang berulang sampai 3 kali,
diantara kejang penderita tetap tidak sadar
31. Tn.M usia 72 tahun dibawahke UGD oleh sampai dibawa ke UGD. Pasien menderita
anaknya dengan penuruanan kesadaran. epilepsy sejak usia 2 tahun dan rutin berobat.
beberapa hari sebelumnya, pasien mengeluh Tidak ada riwayat infeksi maupun trauma. Dua
merasa tidak febris dan sesak nafas. Hasil minggu ini penderita tidak minum obat dari
pemeriksaan laboratorium : hitung lekosit hasil pemeriksaan didapatkan tekanan darah
16.000/mm3. BUN 100mg/dL dan kreatinin 6.3 150/100, nadi 90x/menit, RR 24x/. GCS 223,
mg/dL (gangguan fungsi ginjal). Foto thorax : refleks fisiologis meningkat pada kedua sisi,
pneumonia. Diagnosis etiologi dari coma yang dan refleks babinski positif pada kedua kaki.
diderita pasien adalah : Diagnosis klinik kasus V di atas adalah :
a. Gagal ginjal a. Epilepsy
b. Gagal elektrolit b. Status sepileptikus
c. Lekosistosis c. Meningoensefalitis
d. Gagal ginjal dan hypoxic askemis d. Stroke
encephalopathy e. Kejang serial
e. Infeksi dan lekosistosis
36. Untuk kasus V apa diagnosis etiologisnya :
32. Tipe coma dibawah ini disebabkan oleh a. Meningoensefalitis
hipoksia atau anoksia : b. Antiepilepsy drug withdrawal
a. Coma dengan herniasi c. Tumor otak
b. Coma bihemisfer d. Hipoalbumin
c. Coma neurologis e. Hiponatremia
d. Coma supratentorial
e. Coma infratentorial 37. Wanita usia 17 tahun dibawa ibunya
kepoliklinik karena sering bertingkah aneh.
33. Tipe respirasi pada pasien coma dengan lesi di penderita sering mendadak mengecap mulutnya
pons adalah : sambil meraba bajunya dan berjalan kesana
a. Cheyne strokes respiration kemari tanpa tujuan yang jelas. Hal itu terjadi
b. Central neurogenic hyperventilation selama lebih kurang 1 menit. Selama kejadian
c. Cluster breathing penderita tidak dapat diajak komunikasi, setelah
d. Ataxic breathing selesai penderita tampak bingung beberapa saat
e. Apnea dan kemudian kembali normal. Penderita tidak
dapt mengingat kejadian itu. Ini sudah
34. Tanda herniasi otak tipe uncal dengan lesi di berlangsung selama 3 bulan terakhir. Dalam
mesensefalon : sebulan terjadi 5 kali. Kemungkinan diagnosis
Cheyne stokrs respiration, small reactive kasus tersebut adalah:
pupils, a. Epilepsy petit mal
a. Cheyne stokrs respiration, gegenhalten b. Epilepsy grandma
b. Central neurogenic hyperventilation, pupil c. Epilepsy psikomotor
anisocoria, decorticated rigidity d. Epilepsy mioklonik
c. Ataxic breathing, pin point pupil, e. Epilepsy fokal sensoris sederhana
hyperthermia
d. Apneustic, midriasis, flaccid atonic 38. Anak usia 10 tahun mengalami kejang
pada tangan kanan. Saat kejang pasien tetap c. Frekuensi kejang
asadr. Kejang tidak berhenti sampai 5 menit d. Kesadaran diantara kejang
kemudian tungkai kiri serta penderita tidak sadar. e. Jumlah saudara sekandung
Dalam klasifikasi tipe kejang menurut ILAE (1981)
kasus tersebut termasuk dalam : 44. Dibawah ini tidak termasuk masalah terkait
a. Simple focal motor seizure dengan penderita epilepsy
b. Complex focal seizure a. Depresi 29
c. Focal seixure secondary generalized b. Restriksi ADL (activity of daily living)
d. Atonic seizure c. Harus minum obat seumur
e. Clonic seizure d. Kesulitan mendapatkan SIM
e. Resiko trauma meningkat
39. Dibawah ini diagnosis banding epilepsy kecuali
a. Syncope 45. Dibawah ini adalah factor pencetus kejang,
b. Breath holding spells kecuali
c. Transient ischemic attack a. Kurang tidur
d. Classic migraine b. Konsumsi alcohol
Tension type headache c. Stress fisik
d. Stress mental
40. Pilihan terapi lini pertama untuk kejang umum e. Tidur berlebihan
adalah :
a. Lemotrigine 46. Wanita 60thn mengeluh sering kesemutan
b. Levetiracetam pada tangan dan lengan kanan sejak 2 bulan yang
c. Vigabatrine lalu. Kesemutan dirasa menjalar darimulai ujung
d. Ethosuximide jari sampai lengan atas. Kejadiannya sekitar
e. Phenitoin beberapa detik dan mendadak hilang tanpa
tindakan apapun. Ini tidak pernah terjadi
41. Laki-laki 35 tahun dating ke poli saraf dengan sebelumnya. Hali dibawah ini tidak benar terkait
keluhan sering kejang sejak 3 bulan yang lalu. kasus sebelumnya. Hali dibawah ini tersebut,
Kejang diawali sisi kanan kemudian diikuti seluruh adalah
tubuh dan pasien tidak sadar. Frekuensi 3x dalam a. Diagnosis meliputi MRI
satu bulan. Didapatkan riwayat sakit kepala sejak b. Termasuk simple focal sensory seizure
4 bulan. Dari pemeriksaan neurologis didapatkan c. Terapi lini pertama dgn clobazzam
hemiparesis ringan sisi kanan dan refleks babinski d. DD meliputi stroke dan tumor otak
positif kaki kanan. Tidak ada riwayat panas / e. Biasanya berkembang menjadi focal sizure
trauma. Kemungkinan penyebab kejang pada secondary generalized
pasien ini adalah :
a. Epilepsy idiopatik 47. Seorang wanita 65 tahun dating ke keluhan
b. Stroke tangan kanan kiri bergetar seperti menghitung
c. Ensefalitis uang. Bibirnya juga gemetar. Penderita masih bisa
d. Uremic ensephalopathy berjalan tetapi sangat pelan dan dipapah. Wajah
e. Tumor otak kaku seperti topeng dan air liurnya meleleh.
Diagnose untuk kasus diatas adalah :
42. Dibawah ini adalah tanda pseudoseizure a. Penyakit Huntington
a. Luka pada lidah b. Penyakit Parkinson
b. Generakan tonik klonik c. Stroke ringan
c. Terjadi saat tidur d. Tremor essensial
d. Terjadisaat kondisi ramai e. Hipertiroid
e. Setelahkejang penderita tidur
48. Letak lesi pada kasus di atas tersebut :
43. Data dibawah ini tidak diperlukan dalam a. Korteks
anamnesis kejang : b. Sub cortex
a. Riwayat kejang pada saudara sekandung c. Substansia nigra
b. Tipe kejang d. Capsula interna
e. Tiroid c. hipoventilasi
d. Berikan airan isotonis
49. Pada kasus tersebut, pernyataan di bawah ini e. ABC
benar
a. Kadar katekolamin rendah 5. Mana di antara pernyataan berikut yang paling
b. Kadar asetokolin rendah berpengaruh pada beratnya cedera otak sekunder
c. Kadar serotonin otak rendah sistemik : 30
d. Kadar GABA rendah a. Cedera axonal diffuse
e. Kadar dopamine otak rendah b. Hematoma epidural
50. Letak lesi pada penderita balismus c. Hiperventilasi
a. Cortex d. Hipotermia
b. Cerebelum e. Usia
c. Nucleus subthalamus
d. Medula spinalis 6. Keadaan ini merupakan gawat darurat di
e. Thalamus bidang bedah syaraf :
a. Herniasi uncal
MKDI NEUROLOGY - 5 b. Hematoma serebri
c. Kontusio hemorrhagic
1. Karakteristik demam kejang kompleks, kecuali : d. Diffuse axonal injury
a. Kejang tonik klonik e. Benar semua
b. Lama kejang > 15 menit
c. Kejang parsial / fokal 7. Tanda fraktur basis kranii bagian medial :
d. Awal kejang parsial / fokal kemudian general a. CSF rinorre
e. Frekuensi serangan kejang >1x b. Brill hematome
pertama c. Anosmia
d. Parese N. okulomotor
2. Manifestasi klinis penderita tumor otak e. Battles sign
(intrakranial) adalah:
a. Defisit neurologis fokal merupakan gejala yang 8. Kehilangan fungsi neurologis sementara tanpa
paling sering dijumpai ada
b. Tanda-tanda peningkatan ICP dijumpai pada kelainan structural merupakan patofisiologi dari :
20% a. Diffuse axonal injury
kasus penderita tumor otak yang berobat b. Kontusio serebri
c. Kejang epileptic terjadi pada 20% c. Cerebral concussion
penderita yang tumornya berlokasi di area d. Cedera kepala ringan
hemisfer serebri e. diffuse brain injury
d. Gangguan sistem endokrin
e. Semua benar 9. Otak lecet dengan kemungkinan perdarahan
adalah patofisiologi dari :
3. Tanda-tanda peningkatan tekanan intracranial a. Diffuse axonal injury
: b. Kontusio serebri
a. Hipertensi c. Cerebral concussion
b. Hipotensi d. Cedera kepala ringan
c. Penurunan kesadaran e. Diffuse brain injury
d. Tachycardia
e. Hipoventilasi 10. Indikasi pemberian mannitol
head traima :
4. Jika anda mendaoatkan pasien trauma kepala a. Epidural hematome
dengan penurunan kesadaran dan tanda herniasi, b. Subdural hematome
untuk mengurangi edema yang harus dilakukan c. Intracranial Hemorrhage
adalah : d. Kontusio hemorrhagic
a. Berikan O2 murni dengan aliran tinggi e. Penurunan deficit neurologis yang cepat
b. Hiperventilasi
11. Korban kecelakaan dengan GCS 9, nafas 40x / b. Medulloblastoma
menit, pendarahan temporoparietal, hematoma c. Ependymoma
subdural, pupil kiri anisokor. Bagaimana d. Chordoma
patofisiologinya? e. AStrocytoma
a. Kompresi hemisfer cerebri
b. Kompresi hemisfer cerebelli 19. Pilih jawaban yang palinng benar mengenai
c. Kompresi batang otak tumor otak : 31
d. Pecahnya bridging vein a. Medulloblastoma sering terjadi pada usia
e. Pecahnya arteri meningea media lanjut
b. Proses metastase otak paling banyak lewat
12. Dari scenario di atas, apa penyebab hematogen
patofisiologi pupil anisokor? c. Pasien intracranial jarang datang dengan
a. Kompresi N. cranialis I deficit
b. Kompresi N. cranialis II neurologis, peningkatan TIK, dan kejang
c. Kompresi N. cranialis III d. Kemoterapi harus diberikan
d. Kompresi N. cranialis IV e. Pemeriksaan cerebral angiografi dapat
e. Kompresi N. cranialis V mendeteksi jenis patologi tumor otak

13. Yang merupakan tumor otak primer : 20. Chlorpromazine bukan obat yang tepat untuk
a. Adenoma pituitaria, pinealoma, teratoma manajemen :
b. Meningioma, astrocytoma a. Psikosis
c. Ca mama, Ca tiroid, Ca paru b. Schizophrenia
d. Meningioma, Ca mama, Ca tiroid c. Drug-induced psychosis
e. Adenoma hipofisis, neuroblastoma, Ca mama d. Psikotic depression
e. Insomnia
14. Yang tidak termasuk tanda-tanda yang dapat
muncul pada tumor otak adalah : 21. Pasien schizophrenia menunjukkan
a. Stroke-like syndrome bradikinesia, neck rigidity, dan tremor selama
b. Tanda-tanda peningkatan TIK terapi dengan psikotik. Di antara obat berikut
c. Epilepsi yang memiliki efek
d. Defisit neurologis fokal samping tersebut adalah :
e. Gejala-gejalanya akut a. Clorpromazine
b. Clonazepam
15. Etiologi tumor yang memiliki prognosis baik : c. Risperidone
a. Oligodendroglioma d. Diazepam
b. Ependymoma e. Ziprazidone
c. Meningioma
d. Astrocytoma 22. Seorang laki-laki muda didiagnosis
e. Meduloblastoma schizophrenia. Ia mengalami kram otot hebat dan
tortik setelah obat diganti dengan haloperidol.
16. Merupakan tanda tumor otak di regiofrontal, Terapi terbaik adalah :
kecuali : a. Tambahkan clozapin pada regimen obat
a. Gangguan kepribadian b. Stop haloperidol dan observasi pasien
b. Frontal lobe syndrome c. Berikan diphenhydramin
c. Kelemahan tangan kontralateral d. Ganti obat dengan fluphenazine
d. Gangguan berjalan e. Injeksi benztropine (antiparkinson)
e. Kelemahan kaki kontralateral
23. -
17. -
24. Pernyataan berikut ini yang sesuai dengan
18. Di antara tumor intracranial berikut ini, yang fenotiazine :
merupakan derivate neural crest adalah : a. Mengaktivasi reseptor muskarinik
a. Accoustic Neurionoma b. Merupakan antiemetik
c. Menurunkan level prolaktin serum pada :
d. Meningkatkan ambang seizure a. pH urin
e. Meningkatkan tekanan darah b. Produksi urin
c. Frekuensi uriin
25. Obat di bawah ini memiliki afinitas tinggi d. Metabolisme salisilat
terhadap e. Waktu paruh salisilat
reseptor D2 pada anak dan menyebabkan 32
disfungsi extrapyramidal : 32. Yang mana dari pernyataan berikut bukan
a. Haloperidol merupakan efek samping aspirin?
b. Olanzepine a. Ulkus peptikum
c. Risperidone b. Waktu pendarahan memanjang
d. Fluphenazin c. Menginduksi serangan asma bronkiale
e. Clozapin d. Rye syndrome
e. Agregasi platelet
26. Yang mana dari pernyataan berikut sesuai
dengan kerja monoamine oksidase inhibitor : 33. Sifat antibiotic untuk kemoterapi efektif
a. Increase NE in the brain meningitis bacterial adalah sebagai berikut,
b. Decrease Dopamine in the brain kecuali :
c. Weight neutral a. Antibiotik mampu membunuh bakteri patogen
d. Increase metabolism the other drugs b. ANtibiotik dapat menembus ke daerah CSF
yang
27. Obat di bawah ini termasuk antiplatele, ke terinfeksi
a. Acetyl salicyclic acid c. Konsentrasi antibiotic di csf anak neonates
b. Acetaminophen lebih tinggi dari orang dewasa dengan meningitis
c. Ticlopidin d. Konsentrasi antibiotic di csf sama dengan mbc
d. Dipiridamol untuk bakteri pathogen ketika diukur secara in
e. Asam mefenamat vitro
e. Bakteri pathogen harus sangat peka terhadap
28. NSAID di bawah ini dapat digunakan sebagai antibiotic yang dipilih
obat antitrombotik :
a. Asam mefenamat 34. Kesalahan umum pada penanganan meningi
b. Sodium diklofenak a. Keterlambatan lumbar pungsi
c. Aspirin b. Keterlambatan pemberian antibiotik
d. Piroksikam c. Kadar obat rendah di CSF
e. Ibuprofen d. A,B
e. A,B,C
29. Di bawah ini benar mengenai thromboxane
(TxA2) 35. Obat dengan daya penetrasi rendah untuk
a. An eicosanoid masuk ke dalam CSF pada meningitis :
b. is a potnt vasodilator a. Clindamycin
c. Induces platelet aggregation b. Penicilin
d. Action on platelets is inhibited by prostacyclin c. Vancomycin
e. as an amplification signal for thrombin & ADP d. Cefriaxon
e. Chlorprhomazine
30. Berapakah dosis efektif harian aspirin yang
dibutuhkan sebagai antitrombotik? 36. Chlorpromazine interaction :
a. 1000 mg a. Increase metabolism of phenytoin
b. 500 mg b. Increase successful therapy of parkinson
c. 400 mg c. Addictive with anticholinergic effect
d. 250 mg d. Decrease alpha-blocker effect
e. 160 mg e. Decrease antidepressant effect

31. Eskresi salisilat bebas melalui urin tergantung 37. Seseorang mahasiswa setelah diperiksa
dengan MRI, dokter menjelaskan bahwa ia e. Tensi diturnkan hingga 120/80
memiliki masalah dengan fungsi berpikir dan
merencanakan. Area manakah pada otak yang 43. Wanita 26 tahun, tiba sakit kepala hebat
bertanggung jawab terhadap berpikir dan (+),tiba-tiba sakit muntah, kaku kuduk (+), tanpa
merencanakan? lumpuh sebagian tubuhnya :
a. Visual cortex a. Meningitis
b. Area 1,2,3 post central gyrus b. Stroke emboli 33
c. Promotor cortex c. Perdarahan sub arachnoid
d. Frontal cortex d. Encephalitis
e. Limbic cortex e. Perdarahan intraserebri

38. What is the function of hypocampus in the 44. Penyebab kasus I di atas, terbanyak adalah :
limbic a. TBC paru
system? b. Hipertensi
a. Speaking c. Virus
b. Planning d. AVM
c. Imagination e. Kelainan katup jantung
d. Memory store
e. Motoric action 45. Seorang perempuan dibawa ke UGD oleh
suaminya karena tidak sadar. Ia mengalami nyeri
39. Nociceptor sesuai dengan di bawah ini kepala dengan demam sejak 10 hari yang lalu.
a. Rasa / pengecapan dalam mulut Pada pemeriksaan tekanan darah 110/80 mmHg,
b. Bau-bauan dari hidung temperature 40C, kaku kuduk (+). Riwayat medis
c. Kerusakan jaringan oleh karena fisik / kimiawi karies beberapa gigi sebelum dirawat. Mekanisme
d. Kadar O2 darah arteri dan CO2 jaringan apa yang terjadi?
e. Faktor-faktor lain yang menyusun keadaan a. Percontinuitatum
kimiawi tubuh b. Hematogen
c. Limfogen
40. Seorang laki-laki, 48 tahuhn, waktu aktivitas d. Metastatik
di kantor jam 10 tiba-tiba merasa sakit kepala (+), e. Direct contact
muntah (+), lumpuh bagian kanan (+), bicara
pelo, mulut perot,parese N VII dan XII kanan, 46. Kontraindikasi lumbal pungsi adalah :
tekanan darah 200/100 mmHg. Hemiplegi dextra a. Nyeri kepala
dan parese N VII dan N dysartria menunjukkan b. Demam
lesi di : c. Papil edema
a. Korteks d. Kaku kuduk
b. Subkorteks e. Comma
c. Batang otak
d. Cerebellum 47. Terami primer antibiotic pada meningitis
e. Spinal cord bakteri :
a. Harus dikopi dengan antibiotic segera setelah
41. Diagnosis pada kasus di atas : ada
a. Stroke thrombosis hasil kultur darah
b. Stroke ICH b. Harus diterapi dengan antibiotic segera
c. Stroke emboli setelah
d. Stroke SAH diagnosis ditegakkan
e. TIA c. Harus diterapi dengan antibiotic hanya bila
kondisi pasien terus memburuk
42. Terapi pada fase akut kasus di atas adalah : d. Antibiotik tidak selalu diperlukan
a. Tensi diturnkan hingga <220/110 e. Harus diterapi dengan antibiotic bila telah ada
b. Tensi diturnkan hingga <200/110 hasil sensitivitas terhadap antibiotik
c. Tensi diturnkan hingga <180/100
d. Tensi diturnkan hingga 160/90 48. Pemeriksaan untuk menegakkan diagnosis
pasti dari meningitis : NEUROEMBRIOLOGY
a. foto kepala
b. MRI 1. Sistem saraf pusat berasal dari lapisan
c. CT-scan
...................................... dari germ layer.
d. EEG
e. Lumbar pungsi 2. Pada minggu ke-3 bagian dorsal embryo
34
membentuk neural plate yang kemudian
49. Penyebab tersering meningitis dewasa usia mengalami invaginasi membentuk
18
tahun adalah :
a. E.coli 3. Anencephaly terjadi karena terdapat gangguan
pada penutupan craniophore anterior,yang
b. Streptococcus
terjadi pada hari ke .. dari perkembangan
c. H. influenza embryo
d. N. Meningiditis, S. pneumonia
e. Gram negative bacill 4. Medula spinalis adalah bagian dari system saraf
pusat yang berasal dari
50. Pada meningitis, terutama meningitis
bacterial, ditemukan trias klasik, yaitu : 5. Pada saat lahir, bagian metencephalon akan
menjadi organ ..
a. Demam, sakit kepala, muntah
b. Demam, sakit kepala, kaku kuduk 6. Bagian ventral neural tube akan mengalami
c. Demam, kejang, sakit kepala penebalan dan terbentuk basal plate, yang
d. demam, muntah, kaku kuduk Kemudian akan menjadi ..............................
e. Kejang, muntah, kaku kuduk
7. Ruangan tempat hemisphare otak yang disebut
ventrikel lateral akan berhubungan dengan
ventrikel III, ruang melalui ............................

8. Interaksi epitel-mesenchym pada area


frontonasal akan membentuk system pembau
yang disebut ...................................................

9. Sebutkan klasifikasi neuron berdasarkan


struktur.
a. .
b. .
c. .

10.Sebutkan sel penyokong saraf (supporting cel


yang dibentuk dari glialblast..
a. .
b. .
c. .
NEUROPERINATOLOGY 28 minggu

8. Yang bukan merupakan terapi yang dilakukan


1. Bayi cukup bulan dilahirkan secara seksio pada bayi premature yang mengalami ROP :
saesaria atas indikasi perdarahan antepartum dan A. Laser fotokoagulasi
fetal distress, setelah lahir bayi tidak langsung B. Terapi sinar
menangis dan tonus otot lemah, setelah C. Krioterapi 35
dilakukan resusitasi bayi menangis lemah dan D. Vitrektomi
nilai apgar 3 pada menit ke 5, berdasarkan
riwayat kelahirannya bayi tersebut mengalami : 9. Komplikasi yang dapat terjadi pada bayi
A. Kejang premature yang mengalami ROP :
B. Anemia A. Niktagmus
C. Asfiksia B. Konjungtivitis
D. Hipoglekemi C. Glaukoma
D. Keratitis
2. Setelah satu jam bayi mengalami kejang.
Diagnosa pada bayi tersebut ?
A. Ensefalitis 10. Pencegahan yang paling efektif kejadian
B. Meningitis ROP :
C. Hipoksi-iskemik ensefalopati A. Mencegah terjadinya sepsis pada bayi
D. Hipoksia premature
B. Mencegah kelahiran premature
3. Pada kasus di atas faktor resiko apa yang C. Mencegah kelahiran kembar
dapat menyebabkan kondisi bayi seperti di D. Menghindari pemakaian terapi sinar
atas : Ultraviolet
A. Syok hipovolemi
B. Hipoglikemi
C. Prematuritas
D. Sepsis

4. Obat pilihan pertama untuk mengatasi


kejang pada kasus di atas adalah :
A. Diazepam
B. Phenitoin
C. Ice Sprite
D. Morphine
E. Phenobarbital

5. Komplikasi yang dapat terjadi pada bayi


yang mengalami asfiksia :
A. Sepsis
B. Hipotensi
C. Anemia
D. Hiperbilirubinema

6. Yang bukan merupakan salah satu faktor


resiko terjadinya retinopati bayi prematur :
A. Prematur dengan BBL < 1500 gram
B. Prematur dengan Sepsis
C. Pemberian oksigen yang tidak berlebihan
D. Premature dengan penyakit membran
hialin

7. Skrening ROP dilakukan pada bayi


prematur :
A. Usia Gestasi > 36 minggu
B. BBL > 2000 gram
C. Usia bayi 4 6 minggu
D. BBL < 1500 gram atau usia gestasi <
NEUROPEDIATRY 7. Terapi meningitis Tuberculosa :
A. 3 Regiman Obat Anti tuberculosa
1. Manifestasi Klinik dari meningitis bacterial : (OAT) dan korticosteroid
A. Tidak berkaitan dengan umur B. 4 regimen OAT
B. Pada umumnya disertai panas tinggi 1 3 hari C. 4 regimen OAT dan acetazolamid
C. Kejang berulang atau terus-menurus D. 4 regimen OAT dan Corticosteroid
E. 4 regimen OAT, acetazolamid dan 36
D. Tidak sadar setelah kejang
E. Selalu didapatkan tanda meningeal corticosteroid.
(Meningeal Sign)
8. Komplikasi dari meningitis tuberculosa :
2. Etiologi meningitis bacterial : A. Hydrocephalus
A. Selalu disebabkan oleh S.Typhi B. Kebutaan
B. Tidak dapat disebabkan oleh E.Coli C. Epilepsi
C. Tidak dapat disebabkan oleh S.paratyphi D. Neuropathy perifer
D. Pada umumnya disebabkan oleh E. Retardasi Mental (RM)
S.pheumococcus
E. Tidak dapat disebabkan oleh H.Influenza 9. Transmisi tugas simplex virus melalui :
A. Penyebaran langsung dari focus
3. Gejala sisa (sequel) pada meningitis terdekat di otak
bacterial yang umumnya terjadi: B. Hematogenous dari focus yang jauh
A. Spastic pada extremitas C. Air ludah dan jalan napas
B. Epilepsy D. Makanan
C. Gangguan pendengaran E. Semua benar
D. Retardasi mental
E. Kelumpuhan 10. Dasar Dx Encephalitis :
A. Manifestasi Klinik
4. Manifestasi klinis dari meningitis TB, B. Hasil pemeriksaan laboratorium
kecuali : C. EEG
A. Paresis Nn. Cranialis D. Scaning
B. Kejang E. Semua benar
C. Penurunan kesadaran
D. Gerakan involunter 11. Pada Pemeriksaan cairan Cerebrospinal,
E. Hemiparesis biasannya didapatkan :
A. Jumlah sel > 1000/mm3
5. Penyebab meningitis TB : B. Kadar protein Netral
A. Penyebaran limfogenik dari Mycobacterium C. Kadang berwarna xantochrom karena
TB perdarahan
B. Tuberkel di Meningen pecah D. Kadar glucosa sangat rendah
C. Penyebaran hematogen dari mycobacterium E. Semua jawaban benar
D. Kondisi pasien immunoconpromise
E. Derajat virulensi kuman 12. Pengobatan pada encephalitis HSV :
A. Ceftriaxone for 1 week
6. Gangguan spesifik cairan cerebrospinal B. Acyclovir for 10 days
pada meningitis TB : C. Ampiciline + Chlorampenicole for 10 days
A. Protein dan glukosa meningkat D. Cefotaxime for 10 days
B. Protein dan glukosa menurun E. Gancyclovir for 10 days
C. Ribuan sel dengan dominasi
polymorphonuclear 13. Pengobatan Japanese B encephalitis :
D. Ratusan hingga ribuan sel dengan dominasi A. Ceftriaxone selama 1 minggu
linpositer B. Acyclovir selama 10 hari
E. Ratusan sel, level glukosa, dengan protein C. Ampiciline + Chloramphenicole
normal selama 10 hari
D. Pengobatan suportif
E. Kortikosteroid E. TIC

14. Gejala sisa yang mungkin timbul dan perlu 20. Salah satu dibawah ini tidak termasuk
disampaikan kepada orang tua waktu penderita dalam sindroma epilepsi :
pulang A. Absence epilepsy
A. Retardasi mental B. Infantile spasm
B. Gangguan kognitif C. Lennox Gastaut syndrome 37
C. Seizure / Kejang D. Juvenile myoclonic epilepsy
D. Gangguan perilaku E. Benign neonatal familial convulsion
E. Semua benar

15. Sequel di encephalitis yang paling umum :


A. Retardasi Mental
B. Extremity spasticity
C. Seizure
D. Blindness
E. Hearing disturbance

16. Fakta mengenai kejang demam sebagai


berikut, kecuali :
A. Kejang demam terjadi pada 2 4 %
dari populasi anak 6 bulan- 5 Tahun
B. 80% merupakan kejang demam sederhana,
sedangkan 20% kasus adalah kejang kompleks
C. 20 % berlangsung lama (lebih dari 15 menit)
D. 16 % berulang dalam waktu 24 jam
E. Kejang pertama terbanyak di antara
umur 17-23 bulan

17. Kriteria kejang demam sederhana sebagai


berikut, kecuali :
A. Kejang berlangsung singkat (kurang
dari 15 menit)
B. Kejang bersifat umum
C. Kejang tonik dan klonik
D. Kejang akan berhenti sendiri
E. Kejang Fokal atau berulang dalam waktu 24
jam

18. Faktor resiko berulangnya kejang demam


sebagai berikut, kecuali :
A. Riwayat kejang demam dalam keluarga
B. Adanya kelainan neurologis sebelum demam
C. Usia kurang dari 15 bulan
D. Temprature yang rendah saat kejang
E. Cepatnya kejang setelah demam

19. Yang termasuk dalam non-epileptic


zeizure sebagai berikut, kecuali :
A. Breath holding spell
B. Migraine
C. Masturbasi
D. Simple partial seizure
NEURO-OCULAR MICROBIOLOGY E. Formaldehid

1. Mikroorganisme dibawah ini adalah bakteri 6. Seorang anak berumur 10 Tahun tiba-tiba
yang dapat menyebabkan infeksi pada system menderita panas. Beberapa hari kemudian anak
saraf : tersebut mengalami paralisa pada kaki kanannya.
A. Rabies Kemungkinan besar anak tersebut terserang
penyakit : 38
B. Listeria Monocytogenes
C. Cryptococcus neoformans A. Meningitis
D. Prion B. Poliomyelitis
E. Japanese B encephalitis. C. Cryptococcosis
D. Botulism
Sekenario berikut digunakan untuk menjawab E. Rabies
pertanyaan No. 2 5
7. Bacterial Meningitis bisa disebabkan oleh
Seseorang jatuh di tempat yang banyak kotoran mikroorganisme yang mengkontaminasi makanan
kuda. Beberapa hari, yang lalu terjadi kekakuan (susu), yaitu :
pada rahangnya, sehingga orang tersebut oleh A. Mycobacterium leprae
keluarganya segera dilarikan ke rumah sakit yang B. Clostridium tetani
terdekat. C. Poliovirus
D. Diplococcus pneumonieae
2. Dilihat dari gejala penyakit, orang tersebut E. Listeria monocytogenes
kemungkinan terserang infeksi oleh:
A. Staphylococcus aureus 8. Bahan pemeriksaan yang dianjurkan
B. Clostridium perfringens untuk menegakkan penyakit polio adalah :
C. Clostridium tetani A. Sputum
D. Listeria Monocytogenes B. Darah
E. Cryptococcus neoformans C. Apusan luka
D. Apusan rectum/feses
3. Untuk memastikan bahwa infeksi disebabkan E. Bahan muntahan
oleh kuman tersebut diatas; maka perlu dilakukan
pemeriksaan secara mikrobiologis dengan 9. Kelebihan imunisasi polio menggunakan
menggunakan jenis pewarnaan : vaksin Sabin dibanding vaksin Salk adalah :
a . Pewarnaan Gram A. Menginduksi imunitas humoral
B. Pewarnaan Flagela B. Dapat memberikan proteksi terhadap infeksi
C. Pewarnaan tahan asam oleh virus polio
D. Pewarnaan Kapsul C. Secara tidak langsung dapat memberikan
E. Pewarnaan metakromasi imunisasi kepada komunitas
D. Berbahaya bagi seseorang yang mengalami
4. Gejala kekakuan rahang tersebut immunocompromized
disebabkan oleh metabolit kuman berupa : E. Perlu diberikan boster
A. Endotoksin
B. Eksotoksin 10. Hal dibawah ini BUKAN karakter dari toksin
C. Hemolisin botulinum :
D. Leukosidin A. Endotoksin
E. Kolagenase B. Tidak tahan panas
C. Menyebabkan paralisis
5. Pencegahan terhadap terjadinya infeksi D. Mudah diabsorbsi di traktus gastrointestinal
tersebut adalah dengan cara mencuci luka E. Mempengaruhi sistem saraf otonom
menggunakan :
A. Alkohol 70% 11. Untuk membuktikan infeksi pada mata
B. Rivanol disebabkan oleh virus, spesimen yang diambil
C. Hidrogen peroksida (H202) dapat ditanam pada:
D. Lisol A. Thioglycolate broth
B. Blood agar trachoma :
C. Chocolate agar A. Disebabkan oleh C.trachomatis serotype A, B,
D. Kultur Sel Ba, dan C
E. Anaerobic jar B. Merupakan masalah yang memerlukan
pendekatan kesehatan masyarakat.
12. Hordeolum adalah infeksi pada kelenjar C. Terapi antibiotika saja tidak
di kelopak mata yang sering disebabkan menyelesaikan masalah 39
oleh : D. Terapi pembedahan dapat mencegah
A. Adenovirus kebutaan
B. HSV-1 E. Infeksi Penglihatan
C. N.gonorrhoeae
D. Staphylococcus 18. Infeksi herpes simpleks pada mata:
E. Pseudomonas aeruginoasa A. Dapat berupa blefaroconjungtivitis atau
keratitis
13. Bakteri berikut ini dapat merusak epitel B. Keratitis epitel sulit diidentifikasi
kornea yang intak : C. Untuk diagnosis diperlukan pemeriksaan
A. H. Influenzae, N. Gonorrhoeae laboratoris rutin
B. Streptococcus sp., Staphylococcus aureus D. Terapi dengan tetes mata antibiotik dan
C. Chlamydia tracomatis, Pseudomonas steroid
aeruginosa E. Jarang rekuren
D. Achamtamoeba
E. Bacillus sp. , P. Acnes 19. Pernyataan berikut yang benar adalah :
A. Keratitis bakterial sering terjadi pada kornea
14. Pernyataan berikut benar mengenai yang intak epitelnya
pharyngoconjungtiaval fever : B. Bakteri dapat dengan mudah melekat pada
A. Salep mata chloramphenicol adalah epitel kornea
terapi pilihan C. Pemakaian lensa kontak menimbulkanlesi
B. Antibiotik sistemik lebih efektif yang dapat mempermudah infeksi kornea
C. Sekret mata purulen D. Terapi ulkus kornea perlu menunggu hasil tes
D. Dapat sembuh sendiri sensitivitas antibiotika yang sesuai
E. Sering terjadi setelah operasi mata E. S. Aureus adalah penyebab tersering infeksi
kornea pada pemakai lensa kontak
15. Conjungtivitis pada bayi :
A. Hanya disebabkan oleh N.gonorrhoe 20. Bakteri berikut yang sering menyebabkan
B. Harus dirawat inap endoftalmitis pasca operasi katarak:
C. Bila purulen, perlu dilakukan A. P.Acnes
pemeriksaan mikrobiologis B. Salmonella
D. Tidak dapat menimbulkan komplikasi C. Bacillus cereus
E. Tidak perlu diberi pengobatan D. Clostridium
E. Candida albicans
16. Pernyataan berikut tidak benar mengenai
infeksi Chlamydia trachomatis pada mata : 21. Komplikasi lokal dari infeksi step throat
A. Serotype D-K menyebabkan inclusion adalah :
conjungtivitis A. Perikarditis
B. Inclusion conjungtivitis bersifat sub B. Abses peritonsiler
akut C. Gagal ginjal akut
C. Diagnosis pasti dapat dilakukan D. Meningitis
dengan PCR E. Ensefalitis
D. Penularan dapat terjad melalui air
kolam renang 22. Yang tersebut dibawah ini benar untuk C.
E. Terapi antibiotika tidak efektif Diphtheriae yang lisogenik. :
A. Kuman difteri yang mengalami lisis
17. Pernyataan berikut TIDAK BENAR mengenai B. Kuman difteri yang tidak memproduksi toksin
C. Kuman difteri yang diinfeksi bakteriofaga D. Bekerja pada sel otot jantung
D. Kuman difteri yang resisten terhadap E. Tidak dapat dinetralkan dengan antitoksin
antimikroba
E. Bakteri non pathogen 29. Virus penyebab common cold tersering
adalah :
23. Komplikasi dini dan yang mengancam jiwa A. Coronavirus
pasien difteri adalah : B. Rhinovirus 40
A. Trismus C. Enterovirus
B. Paralisis palatum mole D. Influenza virus
C. Obstruksi saluran pernafasan E. Adenovirus
D. Paralisis otot-otot pernafasan
E. Endokarditis 30. Medium diperkaya untuk H. Influenzae
adalah :
24. Yang tidak termasuk mekanisme A. Pai
pertahanan tubuh pada daerah THT B. Loeffler
adalah : C. isoVitaleX
A. Tonsila palatine D. Blood Agar
B. Tonsila lingualis E. Blood tellurite Agar
C. Adenoid
D. Sel goblet membran mukosa
E. Patches of Peyer

25. Hemophilus influenzae tipe b


menyebabkan infeksi pada daerah THT
berupa :
A. Faringitis
B. Laringitis
C. Otitis media
D. Epiglotitis
E. Sinusitis

26. Yang tersebut di bawah ini adalah sifat


C.diptheriae :
A. Berbentuk kokobasil gram negatif.
B. Mempunyai volutin granules
C. Membentuk spora
D. Obligat anaerob
E. Tumbuh subur pada medium sederhana

27. Antigen C. Diphtheriae yang bereaksi


silang dengan Mycobacterium adalah :
A. Antigen K
B. Antigen O
C. Neuraminidase
D. Glikolipid
E. Polipeptida

28. Yang tersebut dibawah ini benar untuk


eksotoksin kuman difteri :
A. Merupakan bagian dari dinding sel bakteri
B. Sub unit B adalah bagian aktif
C. Keluar bila bakteri lisis
NEUROEMERGENCY c. bersamaan pada saat memeriksa jalan
nafas
1. Manakah tanda spesifik untuk spinal shock: d. menstabilkan tulang belakang hanya bila
a. tachycardia yakin terdapat luka kepala & leher
b. bradycardia
c. diaphoresis 8. di bawah ini mana yang menjadi prioritas
d. hypertension pertama pada korban trauma:
a. periksa jalan nafas 41
2. penyebab tersering apa yang menyebabkan b. periksa tingkat kesadaran
cardiopulmonary arrest pada penderita trauma c. mencari tahu apakah korban shock
a. ventricular arrhytmia d. mengeluarkan korban terlebih dahulu
b. hypoxia
c. myocardial contusion 9. cedera spinal dengan spinal shock sering
d. cedera otak memberi tanda:
a. kuliat basah dan dingin, penurunan tensi,
3. cedera kepala dengan peningkatan tekanan peningkatan denyut nadi
intrakranial biasanya dikenali dengan tanda- b. kuliat hangat dan kering, penurunan tensi,
denyut nadi normal
tanda:
c. kuliat hangat dan kering, penurunan tensi,
a. penurunan LOC, penurunan tekanan darah, peningkatan denyut nadi
peningkatan denyut nadi d. kuliat basah dan dingin, penurunan tensi,
b. penurunan LOC, peningkatan tekanan darah, denyut nadi normal
peningkatan denyut nadi
c. penurunan LOC, peningkatan tekanan darah, 10. golden hour dimulai pada
penurunan denyut nadi a. saat terjadi trauma
d. penurunan LOC, penurunan tekanan darah, b. saat keberangkatan ambulans
penurunan denyut nadi. c. saat ambulans tiba di tempat kejadian
d. saat ambulans meninggalkan tempat
4. pada penderita trauma dengan tanda kejadian menujur rumah sakit.
racoon eyes keluar darah dengan telinga kiri
kemungkinan besar ia mengalami: 11. pria 28 th dipukul mata kirinya, jatuh dan
a. epidural hematoma kepala sisi kanan cedera. Pada pemeriksaan
b. subdural hematoma dijumpai : penderita disorientasi (orang,tempat,
c. fraktur basis kranii waktu), pupil anisocor midriasis yang kanan, tensi
d. pendarahan intracerebral 110/60 , pulse 70/min, respirasi 16/min dan
tercium bau alkohol. Penderita dipikirkan dalam
5. indikator manakah yang menandakan adanya situasi Load and Go
cedera otak atau peningkatan tekanan a. benar
intrakranial b. salah
a. hypothermi
b. tachycardia 12. indikator manakah yang menandakan adanya
c. hyperventilasi cedera otak atau peningkatan tekanan cranial?
d. perubahan tingkat kesadaran a. hipotensi
b. tachycardia
6. jika anda mendapatkan korban dengan trauma d. perubahan tingkat kesadaran
kepala, dengan penurunan kesadaran dan disertai
tanda herniasi otak hal yang penting dalam 13. keadaan-keadaan di bawah ini manakah yang
mengurangi atau mencegah edema otak adalah sesuai dengan isolated cedera kepala dengan
a. beri O2 murni dan aliran tinggi peningkatan intra cranial?
b. hyperventilasi a. tensi 90/60 mm Hg, nadi 50/ min
c. hypotermia b. tensi 90/60 mm Hg, nadi 120/ min
d. cairan isotonis c. tensi 160/100 mm Hg, nadi 50/ min
d. tensi 160/100 mm Hg, nadi 120/ min
7. saat anda melakukan primary survey pada
korban trauma, kapan anda menstabilkan 14. pria 35 th mengalami kecelakaan lalu lintas,
cervical spine: sedang duduk dan bertanya : apa yang terjadi?
a. segera setelah anda yakin bahwa jalan napas Pertanyaan itu diulang-ulang,meskipun sudah
telah terbuka dijawab ia tidak sadar 1-2 menit sesudah
b. segera setelah anda selesai melakukan kejadian, menderita memar di dahi dan mengeluh
primary survey
mual, nyeri kepala ringan. Diagnosa kerja apakah
yang paling sesuai? NEUROPATHOLOGY ANATOMY
a. intoksikasi alkohol atau obat
b. comotio cerebri NEUROANESTHETIC
c. hematom epidural
d. shock kejiwaan NEURORADIOLOGY
15. situasi trauma bagaimanakah yang bukan 1. Neoplasma yang sering terjadi di otak 42
Load and Go? adalah
a. penderita trauma capitis, sadar, salah satu a. tumor metastase
telinga mengeluarkan darah b. tumor primer
b. penderita trauma capitis yang hanya c. tumor pembuluh darah
berespon dengan rasa sakit d. neurofibromatosis
c. penderita dengan nyeri abdomen , kulit e. osteo sarcoma
basah, tensi 100/60, nadi 120/J.
d. penderita dengan nyeri, deformitas pelvis 2. Neoplasma CNS pada anak-anak sering
terjadi di daerah
a. hemisphere cerebri
b. fossa posterior
c. corda spinalis
d. lobus frontalis
e. lobus parietalis

3. Tumor-tumor yang berasal dari sel glia


a. astrocytoma
b. oligodendroglioma
c. ependymoma
d. glioblastoma
e. semua benar

4. Gambaran nekrosis dengan kelompok


sel pseudopalisade di sekitarnya
a. meningioma
b. pilocytic astrocytoma
c. ependymoma
d. glioblastoma multiforme
e. oligodendroglioma

5. Pilocytic astrocytoma tergolong klasifikasi WHO


grade
a. I
b. II
c. III
d. IV

6. Tergolong neoplasma jinak


a. astrocytoma
b. oligodendroglioma
c. malignant peripheral nervesheat
tumor
d. fibroblastik meningioma
e. meduloblastoma

7. Psammomma bodies didapatkan pada


a. astrocytoma
b. oligodendroglioma
c. pilocytic astrocytoma
d. meningioma
e. meduloblastoma

8. meningioma dengan klasifikasi WHO grade III


a. synctitial meningioma Untuk
b. fibroblastik meningioma a. mempertahankan cerebral perfusion pressure
c. anaplastic meningioma b. mempertahankan TIK
d. transtional meningioma
c. mempertahankan suplai oksigen
e. psamommatous meningioma
d. jawaban A dan C
9. Verocay bodies didapatkan pada e. jawaban A dan B
a. meninigoma 43
b. schwannoma 16. kondisi yang menyebabkan vasodilatasi
c. neurofibroma cerebral adalah
d. astrocytoma a. penurunan PaCO2
e. meduloblastoma b. peningkatan PaO2
c. peningkatan PaCO2
10. Tumor jaringan saraf subcutan multiple
a. meningioma d. penurunan CMRO2
b. neurofibbroma e. pemberian barbiturate
c. schwannoma
d. dermatofibroma 17. peningkatan konsentrasi anestesi inhalasi
e. fibroushistiocytoma akan menimbulkan efek sebagai berikut
a. meningkatkan CBF
b. meningkatkan ICP
11.obat-obat anestesi mempengaruhi sistem saraf
c. meningkatkan CMR
pusat dengan mekanisme
d. jawaban A,B,C
a. menurunkan cerebral blood flow
e. jawaban A dan B
b. menurunkan cerebral metabolic rate.
c. meningkatkan cerebral metabolic rate
18. di antara agent di bawah ini yang menurunkan
d. meningkatkan aktivitas depolarisasi neuron
cerebral metabolic rate dan itntracranial pressure
korteks
a. halothane
e. meningkatkan excitatory neurotransmitter
b. isoflurane
c. sevoflurane
12. komponen otak yang paling banyak menerima
d. barbiturate
cerebral blood flow adalah
e. semua salah
a. kortikal
b. subkortikal
19. di antara obat di bawah ini, yang paling besar
c. basal ganglia
menurunkan cerebral metabolic rate
d. mesensefalon
a. propofol
e. cerebellum
b. etomidate
c. thiopental
13. bila terjadi peningkatan tekanan intracranial
d. diazepam
karena edema cerebri, kompensasi awal yang
e. midazolam
dilakukan adalah mengurangi volume
a. vaskular
20.obat anestesi yang meningkatkan absorbs
b. arteri
liquor cerebrospinal
c. vena
a. halothane
d. liquor cerebrospinal
b. enfluran
e. plexus choroideus
c. desfluran
d. etomidate
14.dalam kondisi normal, tekanan intrakranial
e. isoflurane
dipengaruhi oleh
a. produksi liquor serebrospinalis
21. Neuroimaging choice for stroke
b. absorbsi liquor serebrospinalis
a. angiography
c. intracranial volume
b. MRI & MRA
d. jawaban A dan B
c. Carotid doppler
e. jawaban A,B,C
d. CT scan
e. plain radiograph
15. regulasi cerebral blood flow bertujuan
22.neuroimaging choice for head trauma 28.in the brain tumor patients, we must
a. brain MRI evaluated:
b. brain CT scan a. CT & MR characteristic, history of sickness
c. brain angiography b. age of the patients, metastasis
d. brain sonography c. solitary/multiple mass, age of the patients
e. skull X Ray d. pseudotumor , kind of mass 44
e. metastasis, calfication
23. imaging modality in neuroradiology:
a. MRI, ductulography 29. possible location of the intracranial
b. angiography, scintigraphy hemorrhage:
c. USG, MRI a. intraparenchymal, intraventricular, orbita
d. CT scan, barium enema b. mastoid, epidural, subarachnoid
e. gamma knife, barium follow through c. subdural, intraparenchymal, paranasal sinus
d. intraventricular, intrasisterna, subdural
24. meningitis is the inflammatin of the e. epidural, subdural, subarachnoid
meningitis, neuroimaging choice for evaluated
meningens: 30. A blow to the skul results in compression
a. skull X Ray injury to coup and countercoup, this may result:
b. angiography a. shearing injury
c. MRA b. hematosinus
d. MRI c. hematomastoid
e. head sonography d. skull fracture
e. pneumoencephalus
25. the goal of imaging in acute stroke:
a. differentiate between infark &
hemorrhage
b. differentiate luxury perfusion
c. differentiate between brain tumor & CVA
d. differentiate between brain infection & CVA
e. differentiate between hemorrhage infark and
brain tumor with hemorrhage

26. neuroradiology is a subspeciality of radiology


focusing on the
a. prevention, diagnosis and follow up of
abnormalities of the CNS, PNS, spine, head & neck
b. diagnosis and characteriszation of
abnormalities of the CNS, PNS, Spine, head &
neck
c. diagnosis and treatment of abnormalities of
the CNS, PNS, spine, head and neck
d. treatment and management of abnormalities
of the CNS, PNS, Spine, head and neck
e. follow up of abnormalities of the CNS, PNS,
spine, head and neck

27. definition of encephalitis


a. inflammation of brain parenchym
b. inflammation of cerebral vascular
c. inflammation of cerebrospinal fluid
d. inflammation of brain & bone
e. inflammation of membranes of the brain
NEUROMEDICAL 6. Tujuan utama Program Rehabilitasi Medik
REHABILITATION pada penderita stroke adalah :
A. Mencegah terjadinya kelainan pada
Seorang bayi dilahirkan pada letak sungsang organ
(presentasi bokong), berat badan lahirnya 4.1 Kg B. Mampu ambulasi / jalan normal
dengan kesulitan (macet) pada bahu. Bayi baru C. Mencegah komplikasi
lahir tersebut mengalami kelemahan pada lengan D. Perbaikan fungsional 45
kirinya. Pada pemeriksaan fisik didapatkan nilai E. Mencegah ulkus decubitus
motorik otot fleksi siku 2, otot ekstensi siku 2,
otot ekstensi pergelangan tangan 1, sedangkan 7. Metoda tradisioal/unilateral untuk program
otot tangan (hand) nya masih normal. Didapatkan penderita stroke bermanfaat :
juga penurunan refleks biceps dan triceps A. Untuk meningkatkan kontrol pada
motorik dan sensori
1. Diagnosa yang tepat untuk pasien ini B. Untuk meningkatkan kontrol pada
adalah : motor dan autonom
A. Duchennes Muscular Distrophy C. Untuk meningkatkan kontrol pada
B. Spinal Muscular Atrophy motorik dan koordinasi
C. HNP Cervical D. Untuk meningkatkan kontrol pada
D. ERB Palsy motor dan ketahanan fisik
E. Klumpke Palsy E. Untuk meningkatkan kontrol pada
motor dan penguatan
2. Deformitas yang terjadi pada pasien ini
adalah : 8. Seorang pasien laki-laki umur 21 tahun
A. Abduksi bahu dengan lesi plexus brachialis kanan sejak 1
B. Eksternal rotasi bahu bulan yang lalu, pada pemeriksaan MMT
C. Fleksi siku didapatkan kekuatan otot anggota gerak
D. Supinasi siku atas kanan rata-rata 2, maka tindakan
E. Ekstensi pergelangan Tangan rehabilitasi yang dianjurkan adalah :
A. Electrical stimulation
3. Penyebab deformitas pada pasien ini B. Electrical stimulation, diathermy
karena kelainan pada : dan exercise
A. C5 C6 C. Electrical stimulation, diathermy
B. C5 C6 C7 dan latihan penguatan otot
C. C5 C6 C7 C8 D. Electrical stimulation, diathermy
D. C7 C8 T1 dan traksi leher
E. C8 T1 E. Electrical stimulation, diathermy
dan traksi massage
4. Penatalaksanaan pada pasien ini adalah :
A. Memposisikan bahu pada 9. Seorang pasien anak dengan stiffness pada
hiperabduksi (patung liberty) sendi siku setekah di gips dapat dilakukan :
B. Kmuckle bander A. Latihan ROM aktif
C. Latihan luas gerak sendi B. Latihan peregangan dilakukan
D. Latihan peregangan. dengan secara aktif
E. Latihan ketahanan C. Dilakukan gentle stretching dan
short lever arm
5. Kelemahan otot-otot ekstensor siku pada D. Pada sendi kontraktur perlu disertai
pasien ini diberikan terapi : dengan diathermy
A. Shor wave diathermy E. Peregangan dilakukan hanya sekali
B. Microwave Diathermy
C. Ultra Sound Diathermy 10. Seorang pasien stroke MRS di RSSA dengan
D. Electro Stimulation anggota gerak atas dan bawah kanan mengalami
E. Traksi kelumpuhan dengan otot dengan nilai 3, maka
pada pemeriksaan
kekuatan otot (MMT) didapatkan : A. Dapat melawan tahanan kuat dan
A. Anggota gerak atas dan bawah sebagian ROM
kanan tidak mampu melawan B. Dapat melawan tahanan ringan
tahanan ringan dengan full ROM
B. Anggota gerak atas dan bawah C. Dapat melawan gravitasi dan full
kanan tidak mampu melawan ROM
gravitasi D. Dapat melawan tahanan kuat 46
C. Anggota gerak atas dan bawah dibidang horizontal
kanan mampu melawan gravitasi E. Dapat melawan tahanan kuat dan
dengan full ROM full ROM
D. Anggota gerak atas dan bawah
kanan mampu melawan gravitas 15. Pernyataan yang benar untuk nilai kekuatan
dengan sebagian ROM otot sama dengan 2 :
E. Anggota gerak atas dan bawah A. Dapat melawan tahanan ringan
kanan mampu melawan gravitasi B. Dapat melawan gravitasi
dengan mendekati full ROM C. Dapat melawan tahanan ringan
dibidang horizontal
11. Untuk stimulasi callus, mencegah atropi dan D. Dapat bergerak dibidang horizontal
edema pada pasien Fracture Antebrachii sampai full ROM
dengan pemasangan gips (Plaster of Paris), E. Dapat bergerak dibidang horizontal
maka latihan yang dianjurkan adalah : tidak full ROM
A. Latihan isometrik pada lengan
B. Latihan isometrik pada lengan dan 16. Seorang pasien spinal cord injury dengan
ROM aktif pada jari-jari tangan level T 10 paraplegia ASIA (frankle)D,
C. Latihan isometrik pada lengan dan maka pasien mampu :
ROM pasif pada jari-jari tangan A. Berjalan
D. Latihan isometrik pada lengan dan B. Berdiri
ROM aktif asistif pada jari-jari C. Duduk Stabil
tangan D. Duduk tidak stabil
E. Latihan isometrik pada lengan dan E. Hanya berbaring
ROM bahu
17. Pernyataan yang benar tentang mobilisasi
12. Pernyataan yang salah tentang pasien spinal cord injury tergantung dari :
penatalaksanaan pasien Duchennes A. Kekuatan otot anggota gerak yang
Muscular Dysthropy (Kelainan Otot) adalah : terkena
A. Edukasi aktivitas sehari-hari B. Adanya atropi otot yang dijumpai
B. Latihan peregangan sendi secara C. Kemampuan melakukan aktifitas
pasif sehari-hari
C. Latihan penguatan otot secara D. Level segmen spinal cord yang
berlebihan terkena
D. Brace E. Adanya deficit neurologis
E. Mencegah progresifitas scoliosis
18. Penilaian kualitas pada penderita stroke
13. Terapi modalitas pada penderita Bells memakai :
Palsy dengan menggunakan alat : A. Barrel index
A. Microwave diathermy B. Frenkel index
B. Shortwave diathermy C. Barthel index
C. Ultrasound diathermy D. Score Index
D. Infra red E. William Index
E. Ultra Violet
Sorang pasien umur 50 Tahun datang
14. Pernyataan yang paling benar untuk nilai dengan keluhan anggota gerak sebelah kiri
kekuatan otot sama dengan 4 : terasa berat sejak 1 hari yang lalu. Pada
pemeriksaan fisik didapatkan nilai kekuatan 24. Pernyataan yang salah tentang pasien spinal
otot anggota gerak atas ~ 1 dan nilai kekuatan cord injury adalah :
otot anggota gerak bawah= 3 disertai adanya a. Tindakan mobilisasi dapat dilakukan dilakukan
parese N.VII sentral. Pada pemeriksaan setelah operasi
radiologi didapatkan adanya trombosis pada stabilisasi tulang belakang
subcortex hemisfer kanan di lobus frontalis B. Prognosa pasien tergantung level
segmen spinal cord injury 47
19. Program rehabilitasi yang dianjurkan saat C. Sering dijumpai gangguan bladder
ini pada pasien tersebut adalah : dan bowel
A. Latihan berdiri D. Komplikasi pneumonia dan ulcus
B. Latihan duduk decubitus dapat terjadi
C. Latihan duduk dan berdiri E. Tindakan operasi stabilisasi untuk
D. Latihan keseimbangan memperbaiki level segmen spinal
E. Proper positioning dan latihan cord injury
ROM
25. Yang tidak termasuk dalam latihan
20. Latihan yang diberikan pada pasien tersebut penguatan otot adalah
adalah : A. Nilai kekuatan > 3
A. Latihan ROM aktif anggota gerak atas kiri dan B. Pasien sadar
latihan ROM aktif anggota gerak bawah kiri. C. Tanpa tahanan
B. Latihan ROM aktif anggota gerak atas kiri dan D. Adanya gerakan sendi dan kontraksi otot
latihan ROM pasis anggota gerak bawah kiri E. Adanya kontraksi otot
C. Latihan ROM pasif anggota gerak atas kiri dan
latihan ROM aktif asistif anggota gerak bawah kiri 26. Pada penderita GBS (Guillain Barre
D. Latihan ROM pasif anggota gerak atas kiri dan Syndrome) didapatkan motorik anggota
latihan pasif anggota gerak bawah kiri gerak atas = 2, maka jenis latihan ROM
E. Latihan ROM pasif anggota gerak atas kiri dan yang diberikan adalah :
latihan penguatan anggota gerak bawah kiri. A. Latihan ROM aktif
B. Latihan ROM pasif
21. Mobilisasi pada pasien tersebut dapat C. Latihan ROM aktif assistif
dilakukan: D. Latihan ROM pasif assistif
A. Hari kedua E. Latihan ROM pasif-aktif
B. Hari ketiga
C. Hari keempat 27. Jenis latihan penguatan tanpa adanya
D. Hari kelima pergerakan sendi :
E. Hari keenam A. Latihan penguatan Isokinetik
B. Latihan penguatan Isotonik
22. Latihan jalan pada pasien tersebut C. Latihan penguatan Isometri
memerlukan alat bantu : D. Latihan penguatan Isometrik
A. Cane E. Latihan penguatan Isobarik
B. Tripod
C. Bilateral Cructh 28. Penderita SCI dengan diagnosa C5
D. Walker tetraplegia ASIA (frankle) E, maka
E. Unilateral Cructh penderita mampu:
23. Setelah 3 bulan kemudian pada pasien A. Berjalan dengan walker
mengeluh bahu sebelah kiri kaku dan nyeri B. Berjalan dengan wheel chair
maka latihan yang dianjurkan adalah : C. Berjalan dengan cervikal collar
A. Latihan ROM pasif D. Berjalan dengan bilateral kruk
B. Latihan ROM aktif E. Berjalan tanpa bantuan alat
C. Latihan aktif asistif
D. Latihan isometrik 29. Penebalan saraf pada patient lepra :
E. Gentle Stretching A. N. Auricularis magnus , N.IV, N.V
B. N. IV, N.V, N.VI
C. N. V, N.VI, N.VII
D. N.V, N. Auricularis magnus, N.VII
E. N.VII, N.VIII, N.IX NEUROPUBLICHEALTH
30. Yang tidak termasuk penatalaksanaan 1. agar terhindar dari stroke, seseorang yang
pada pasien HNP (Hernia Nucleosus memiliki resiko stroke dianjurkan untuk rutin
Pulposus) : melakukan pemeriksaan tekanan darah
48
A. William flexion exercise (Screening). Hal ini termasuk:
B. Pemakaian LS Korset a. primary prevention
C. Traksi lumbal b. secondary prevention
D. Proper back mechanic c. tertiary prevention
E. Latihan otot-otot dasar panggul d. common prevention
e. spesific protection

2. penggunaan pelindung mata pada orang yang


bekerja dengan bahan-bahan kimia berbahaya di
sebuah laboratorium adalah contoh dari...
a. general prevention
b. secondary prevention
c. spesific protection
d. tertiary prevention
e. supporting protection

3. siapakah yang paling tepat menjadi kelompok


sasaran program promosi kesehatan yang
bertujuan untuk menurunkan prevalansi infeksi
telinga pada bayi di suatu daerah?
a. dewasa
b. anak-anak sekolah
c. orang-orang yang memiliki bayi
d. kader kesehatan
e. bayi

KASUS
Diketahui jumlah karyawan yang terkena stroke di
suatu perusahaan dalam waktu 5 tahun terakhir
cukup tinggi. Setelah dilakukan survey pada
semua karyawan ditemukan bahwa kebiasaan
merokok merupakan faktor resiko yang paling
banyak teridentifikasi. Para perokok ini sadar
bahwa kebiasaan mereka berbahaya tetapi
mereka tidak tahu bagaimana menghentikan
kebiasaan tersebut.

4. manakah pernyataan berikut yang paling


tepat digunakan sebagai sub-tujuan (sub-
objective)untuk program promosi kesehatan di
perusahaan tersebut?
a. meningkatnya jumlah karyawan yang tahu
hahaya merokok
b. meningkatnya jumlah karyawan sebesar
50% yang tahu bahaya merokok dalam waktu 2
tahun.
c. meningkatnya jumlah karyawan yang
berhenti merokok hingga 20% dalam waktu 1 b. katarak traumatika
tahun c. otitis media kronik
d. meningkatnya jumlah karyawan yang
memanfaatkan Program berhenti merokok d. stroke
sebanyak 20% dalam waktu 1 tahun e. skizofrenia
e. menurunkan jumlah karyawan yang terserang
penyakit stroke sebesar 25% dalam waktu 1 8. anak kedua pak yatemin tiba-tiba panas, 49
tahun. muntah dan telinganya sakit setelah beberapa
hari berenang di sungai. Dari penyakit di bawah
5. metode apakah yang paling tepat digunakan ini, mana yang sangat mungkin terjadi padanya?
dalam kegiatan promosi kesehatan di perusahaan a. meningitis
tersebut berdasarkan kasus di atas? b. katarak traumatika
a. memasang iklan bahaya merokok di c. otitis media kronik
beberapa surat kabar nasional d. stroke
b. menayangkan iklan tentang cara sederhana e. skizofrenia
berhenti merokok di TV nasional
c. memuat artikel populer tentang usaha orang- 9. berdasarkan faktor epidemiologisnya, maka
orang yang berhasil berhenti merokok di penderita skizofrenia mempunyai karakteristik
majalah perusahaan antara lain, kecuali:
d. membuat acara talkshow tentang kampanye a. malnutrisi
anti rokok di pusat perbelanjaan b. rentang usia terjadinya antara 15-34 tahun
e. membagi-bagi t-shirt kepada seluruh c. penyalahgunaan obat
karyawan yang berisi pesan tentang bahaya d. IQ rendah
merokok e. wanita lebih sering daripada pria

KASUS 10. pak yatemin mengeluh ada penurunan


Pak Yatemin (49 thn) dan keluarganya tinggal di penglihatan selama beberapa bulan terakhir.
lingkungan kumuh dengan hygiene sanitasi yang Ketika diperiksa dokter mata, pak yatemin
buruk. Karena tidak mempunyai kamar mandi, didiagnosa menderita katarak traumatika.
maka lingkungan di tempat mereka menggunakan Pernyataan di bawah ini benar tentang penyakit
sungai katarak traumatika, kecuali:
sebagai sarana mandi, cuci, dan memasak. Pak a. laki-laki lebih sering daripada wanita
Yatemin punya 4 anak yang berusia 16,10,4 dan 1 b. orang dengan riwayat diabetes
tahun. c. riwayat trauma mata
Pak Yatemin bekerja sebagai tukang kebun d. orang tua lebih sering daripada dewasa muda
sekolah, sedangkan istrinya tidak bekerja. e. riwayat pernah operasi mata

6. Anak bungsu Pak Yatemin tiba-tiba panas 11. pernyataan di bawah ini benar tentang
selama 5 hari. Anak ini tidak pernah mendapat katarak
imunisasi BCG. Jika dilihat dari faktor resikonya, a. 20 juta orang akan buta karena katarak pada
maka penyakit di bawah ini yang mungkin tahun 2020
akan diderita oleh anak balita Pak Yatemin b. katarak lebih sering terjadi pada pria
adalah: daripada wanita
a. meningitis c. tinggi badan mungkin berhubungan dengan
b. katarak traumatika terjadinya katarak
c. otitis media kronik d. diare mungkin berhubungan dengan katarak
d. stroke e. stroke mungkin berhubungan dengan katarak
e. skizofrenia
7. anak tertua Pak Yatemin dicurigai 12. hipertensi bisa berhubungan dengan katarak
menggunakan obat-obat terlarang. Dari penyakit tipe
di bawah ini, a. nuklear
mana yang sangat mungkin terjadi padanya? b. kortikal
a. meningitis c. sub kapsular
d. kongenital A. Tanda-tanda peningkatan tekanan intracranial
e. traumatika B. Defisit neurologis fokal
C. Epilepsi
D. Stroke like syndrome
NEUROSURGERY E. Gejala-gejalanya bersifat akut

1. Tanda-tanda klinis di bawah ini adalah 7. Hidrosefalus adalah timbunan cairan di dalam :
hidrosefalus infantile : A. Kepala 50
A. Vena kepala prominen, fontanel tegang dan B. Otak
cembung, penurunan fungsi psikomotor C. Sistem ventrikel
B. Kejang, hemiparesis, nistagmus D. Cerebellum
C. Penurunan fungsi psikomotor, kejang, sutura E. Hemisfer otak
merapat
D. Sutura terbuka, hemiparesis, obesitas 8. Meningoencepalitis dapat menyebabkan
E. Kejang, kepala lebih besar dan anak seusianya, hidrosefalus akibat :
sutura merapat A. Sumbatan
B. Perdarahan
2. Sekresi cairan serebrospinal terutama C. Gangguan penyerapan
terjadi pada organ : D. Penekanan
A. Apendym E. Infark
B. Granulatio pacchioni
C. Langsung ke vena 9. Meningokel frontal yang pecah sebaiknya di
D. Pleksus choroideus operasi pada umur :
E. Semua yang tersebut di atas benar A. 1 hari
B. 1 minggu
3. Meningokel nasofrontal yang pecah disertai C. 1 bulan
dengan hidrosefalus, maka tindakannya adalah : D. 4 bulan
A. Segera pasang VP-Shunt terlebih dahulu, E. Segera setelah diagnosis ditegakkan
seminggu kemudia eksisi meningokel
B. Segera pasang VP-Shunt dan eksisi meningokel 10. Secara hipotesis, etiology, meningokel adalah:
dalam satu tahap A. Toxoplasmosis
C. Segera eksisi meningokel dulu, dari 3 hari B. Defisiensi asam folat
kemudian pasang VP-Shunt C. Keracunan Pb
D. Ditunggu sampai keadaan umum optimal D. Defisiensi Zn
E. Operasi segera dilakukan bila tidak ada tanda E. Radiasi sinar ronsen
infeksi
11. Mana diantara pernyataan berikut yang paling
4. Dari etiologi hidrosefalus dibawah ini yang berpengaruh pada beratnya cedera otak
mempunyai prognosis terbaik adalah : sekunder sistemik :
A. Hemoragi subarachnoid A. Cedera axonal diffuse
B. Ensefalitis B. Hematoma epidural
C. Agenesis akuaduktus sylvii C. Hiperventilasi
D. Toksoplasmosis D. Hiponatremia
E. Meduloblastoma E. Usia

5. Dibawah ini adalah sekelompok jenis tumor. 12.Pernyataan dibawah ini merupakan
Sebutkan yang termasuk tumor otak primer : mekanisme dari cedera otak contra coup :
A. Meningioma, CA mama, CA tiroid A. Impact setempat
B. Meduloblastoma, meningioma, astrocytoma B. Cedera akselerasi
C. CA mama, CA tiroid, Ca Paru C. Crush injury
D. Adenoma hipofisis, neuroblastoma, CA mama
D. Cedera akselerasi
E. Adenoma pituitaria, pinealoma, teratoma

6. Yang tidak termasuk tanda-tanda yang dapat 13. Grafik berikut dikenal sebagai grafik doktrin
muncul pada tumor otak adalah : monroe-Kellie:
gejala-gejala yang sering terjadi setelah cedera
kepala kita kenal sebagai :
A. Simptoma pasca kontusio serebri
B. Sindroma pasca komosio serebri
C. Sindroma pasca kontusio serebri
D. Sindroma pasca cedera otak difuse
E. Sindroma pasca cedera axon difuse 51

19. Pada pasien dengan cedera kepala tertutup


disertai hipotensi, maka kemungkinan terbesar
penyebab hipertensi tersebut adalah :
14.Jawaban dibawah ini bukan merupakan
A. Syok hipovolemik
gambaran klinis dari hipertensi intrakranial
B. Cedera medula spinalis
(peningktan tekanan intrakranial) pada cedera
C. Herniasi sentral tahap medulla oblongata
otak akut:
D. Syok neurogenik
A. Bradikardi
E. Syok kardiogenik
B. Hipoventilasi
C. Edema papil
20.Jawaban berikut bukan indikasi CT-Scan kepala
D. Hipertensi
pada trauma kepala:
E. Penurunan kesadaran
A. Skor GCS 14 setelah resusitasi
B. Keadaan mengantuk atau bingung < 2 jam
15. Usaha otak untuk mempertahankan aliran
C. Cephalgia yang terus menerus
darah otak sekalipun terdapat perubahan tekanan
D. Trauma multiple
perfusi otak dikenal sebagai :
E. Hemiparesis
A. Doktrin Monroe Kallie
B. Compliance otak
21. Pernyataan berikut bukan indikasi untuk
C. Autoregulasi
masuk rumah sakit pada cedera kepala :
D. Phenomena Kemohans notch
A. Cephalgia atau muntah dalam 1- 2 jam pasca
E. Brain regulasi
trauma
B. Fraktur tengkorak
16. Serorang laki-laki 40th pengendara sepeda
C. Pasien mabuk
motor dibawa ke unit gawat darurat dengan
D. Amnesia pasca trauma
trauma kepala saja. Petugas ambulan melaporkan
E. Pasien usia < 5 th atau lebih 50 th
bahwa pupilnya anisokor dan respon hanya pada
rangsangan nyeri dengan flexi abnormal lengan,
22.Gejala atau tanda klnis ini berhubungan
membuka mata, dan bicaranya tidak dapat
dengan fraktur basis kranii fossa anterior :
dipahami. Dari scenario ini, silahkan anda
A. Battle signs
tentukan berapa skor GCS nya :
B. Anosmia
A. 5
C. Bloody atorhea
B. 6
D. Parese N. Facialis
C. 7
E. Ketulian
D. 8
E. 9
23. Lucid interval umumnya terjadi pada pasien
dengan :
17. Dari sekenario diatas (16) pasien juga
A. Kontusio serebri
mengalami :
B. Hematima intraserebral
A. Herniasi tonsiler
C. Hematoma subdural
B. Herniasi subfalsin
D. Hematoma epidural
C. Herniasi uncal
E. Komotio serebri
D. Herniasi sentral
E. Herniasi transtentorial
24. Keadaan ini merupakan kasus yang betul-
betul gawat darurat dibidang bedah saraf :
18. Cephalgia, pusing, fotofobi, penurunan
A. Herniasi uncal
konsentrasi, perubahan suasana hati, merupakan
B. Hematoma epidural A. Cedera kepala berat
C. Hematoma subdural B. Herniasi uncal
D. Penurunan deficit neurologis yang cepat C. Penurunan deficit neurologis cepat
E. A, B, C, dan D benar D. Herniasi sentral
E. A, B, C, dan D SALAH
25.Pernyataan berikut bukan merupakan terapi
yang baik untuk peningkatan tekanan intrakranial: 31.Indikasi penjahitan primer pada cedera syaraf52
A. Intubasi ventilasi tepi adalah :
B. Pertahankan MAP 90 mmHg A. Luka kotor
C. Transfer ke CT-Scan dan atau B. Luka Tumpul
D. Elevasi posture kepala 20-30 derajat C. Kurang dari 10 hari
E. Pemberian kortikosteroid D. Hanya satu atau dua saraf yang berdekatan
E. Multitrauma
26. Selain cedera kepala, penyakit berikut
merupakan kasus-kasus bedah saraf : 32.Derajat lesi neurotmesis adalah :
A. Abses otak A. Axon rusak tapi fasiculus utuh
B. Tumor Otak B. Mungkin sembuh spontan
C. Stroke perdarahan C. Terdapat degenerasi wallerian
D. HNP dan canal stenosis D. Pemeriksaan EMG didapatkan denervasi
E. A,B,C, D benar E. kehilangan fungsi dalam beberapa hari

27.Diagnosis standar untuk stroke perdarahan 33.HNP (Hernia Nukleus Pulposus) disebabkan
adalah : oleh :
A. Tanda dan gejala A. Trauma akut
B. Foto polos kepala B. Trauma berulang-ulang
C. CT-Scan kepala C. Kombinasi trauma berulang-ulang dengan
D. MRI kepala proses degeneratif
E. Angiografi otak D. Kombinasi proses degeneratif dengan proses
infeksi
28. Selain pada cedera kepala, pemasangan ICP E. Kombinasi trauma dan proses kegemukan
monitoring telah digunakan secara luas pada
kasus-kasus : 34.Pengobatan HNP dengan manoparesis adalah:
A. Stroke perdarahan A. Segera dilakukan laminektomi
B. Tumor Otak B. Pemberian obat-obatan dan fisioterapi terlebih
C. Infark serebri dahulu selama 2-4 minggu
D. Infeksi otak C. Traksi lumbal
E. A, B, C , dan D benar D. Pemberian neuro roboransia
E. Bila fisioterapi gagal bisa dilakukan laminekomi
29.Pernyataan dibawah ini bukan indikasi
operasi untuk stroke perdarahan : 35. Cauda equina syndrome adalah :
A. Pasien ICH dengan volume otak kurang dari 10 A. Mempunyai prognosa yang jelek
cm3 atau deficit neurologis minimal B. Gejala Klinis : saddle anastesia, kelemahan
B. Pasien hematoma serebelum dengan diameter motor, dan retensi urine
lebih dari 3 cm C. Tidak memerlukan rujukan segera ke bedah
C. Pasien ICH lobar / basal ganglia ukurannya saraf
sedang / besar D. Cauda equina dimulai dari Vetebra thoracal 12
D. Pasien dengan ICH dengan herniasi otak sampai sacrum
E. Pasien ICH dengan efek massa atau edema E. Pemeriksaan MRI menunjukan edema soft
yang bermakna tissue dan perdarahan

30. Jawaban berikut bukan indikasi pemberian 36. Penyakit yang tidak menyebabkan kompresi
mannitol sebelum pemeriksaan CT-Scan kepala pada spinal cord adalah :
pada cedera kepala : A. Myelocele
B. Chiari malformation
C. Spondylolistesis
D. Hyperparathyroidism
E. Diabetes
MKDI PSYCHIATRY 1
37.Pemeriksaan penunjang radiologi yang paling
1. Demensia yang paling sering didapatkan adalah53
bagus untuk kelainan pada kompresi spinal :
:
A. MRI
A. Demensia Multi Infark
B. CT-Scan
B. Demensia Alzhaimer
C. Pain x-ray
C. Demensia paralitika
D. Bone scanning
D. Demensia pada penyakit Parkinson
E. Dinamic view
E. Semua sama
38. Pilih jawaban yang paling benar :
A. Cedera saraf tepi neuropraksia bersifat 2. Waham yang berhungan dengan Delirium :
sementara A. Biasanya menakutkan
B. Gangguan pada neuropraksia axonnya rusak B. Biasanya nihilistic
tapi mielin utuh C. Biasanya tolol dan tanpa efek
C. Gangguan fungsi pada neuropraksia terjadi D. Biasa merupakan waham kebesaran
lebih dari 24 jam E. Tidak satupun yang diatas ini
D. Pada neuropraksia sudah terjadi degenerasi
Wallerian 3. Terapi untuk pasien delirium adalah:
E. Atrophy otot terjadi dalam 4 bulan pada A. Terapi penyebab
neuropraksia B. Terapi simtomatis
C. Penunggu adalah keluarga yang dikenal pasien
39. Yang bukan penyebab dari CTS adalah : D. Pasien di tempatkan di ruang yang tenang,
A. Fraktur penerangan cukup.
B. Diabetes E. Semua yang diatas ini benar
C. Penekanan langsung pada lengan karena
pekerjaan 4. Gangguan kepribadian organic mempunyai ciri
D. Kecelakan lalu lintas klinis, KECUALI :
E. Acromegaly A. Labilitas emosi
B. Tidak impulsif
40. Yang bukan indikasi pembedahan pada C. Paranoid
HNP adalah D. Gangguan prilaku seksual
A. Ada kesamaan antara keluhan E. Tidak dapat bekerja / berfikir lama
penyakit, pemeriksaan fisik dan
pemeriksaan radiologis 5. Gangguan mental organik artinya
B. Terjadi Cauda equina syndrome A. Terdapat kerusakan organic pada otak.
C. Tidak ada respon terhadap terapi Konservatif B. Adanya gangguan fungsi otak
D. Kelemahan motorik yang progresif C. Adanya gangguan neurotransmitter di otak
E. Terjadi perbedaan antara pemeriksaan fisik D. Gangguan mental yang diketahui disebabkan
dan radiologis oleh seatu penyakit badaniah
E. Sekumpulan gejala psikiatrik dengan kesadaran
yang menurun

6.Pada pasien Demensia terdapat gangguan


fungsi luhur yang multipel sebagai berikut,
KECUALI :
A. Daya pikir
B. Daya ingat
C. Daya orientasi
D. Daya pemahaman
E. Kesadaran berkabut
A. Amnesia
7.Yang membedakan psikosa dengan neurosa B. Disorientasi
adalah : C. Fugue
A. Keluhan psikologis D. Trans
B. Keluhan Somatis E. Stupor
C. Kehilangan sense of reality
D. Adanya stressor 14. Yang termasuk dalam kelompok gangguan 54
E. Tidak satupun yang diatas ini. afektif adalah :
A. Episode depresif berat dengan gejala psikotik
8. Semua di bawah ini adalah faktor etiologi dari B. Gangguan skizotipal
gangguan kepribadian, KECUALI faktor : C. Gangguan psikotik akut dan sementara
A. Genetik D. Gangguan skizoafektif tipe manik
B. Kognitif E. Gangguan skizoafektif tipe depresif
C. Konstitusi
D. Sosial 15. Semua gejala dibawah ini termasuk gejala
E. Kultural utama episode depresif KECUALI :
A. Afek depresif
9. Menurut teori psikoanalisa Freud dimana pada B. Konsentrasi dan perhatian berkurang
fase ini anak merasa berkuasa atas dirinya C. Kehilangan minat dan kegembiraan
sehingga ia dapat menahan / mengeluarkan urine D. Berkurangnya energy yang menuju keadaan
dan faecesnya,fase ini dinamakan fase : mudah lelah
A. Falik E. Menurunya aktifitas
B. Oral
C. Toilet training 16.Mekanisme pembelaan ego yang dipakai oleh
D. Laten penderita gangguan obsesif kompulsif adalah :
E. Anal sadistic A. Represi, penyusunan reaksi pelepasan,
pelepasan
10. Pada fase tersebut diatas, usia anak sekitar: B. Represi, salah pindah
A. 0-2 Tahun C. Penyusunan reaksi salah pindah
B. 2-4 Tahun D. Penyusunan reaksi, regresi
C. 3-5 tahun E. Pelepasan salah pindah
D. 6-11 Tahun
E. 11-13 Tahun 17. Mekanisme pembelaan ego yang dipakai
penderita gangguan disosiatif adalah :
11. Yang termasuk teori holistik adalah ajaran : A. Disosiasi
A. Erick Erickson B. Fantasi
B. Harry Stack Sulivan C. Salah pindah
C. Karen Horney D. Pelepasan
D. Erick Fromm E. Isolasi
E. Sigmund Freud
18. Komponen somatik pada depresi adalah
12. Yang biasanya sangat sulit dari langkah- semua yang dibawah ini, KECUALI :
langkah cara penyesuaian diri yang berorientasi A. Insomnia
pada tugas,antara lain adalah : B. Anorexia
A. Mempelajari dan Menentukan Masalah C. Perasaan lelah
B. Menyusun alternatif penyelesaian D. Berdebar-debar
C. Bertindak lalu menilai tindakan E. Tidak bertenaga
D.Memilih alternative yang mempunyai
kemungkinan paling besar akan berhasil 19. Komponen psikologik pada kecemasan adalah
E. Semua pernyataan diatas salah semua yang dibawah ini, KECUALI :
A. Mudah marah
13. Berikut ini yang tidak termasuk dalam B. Gelisah
kelompok gangguan disosiatif adalah : C. Optimis
D. Mimpi yang menakutkan 25. Yang paling sering berhubungan dengan La
E. Sering terkejut Belle Indiference adalah :
A. Reaksi konversi
20. Penderita dengan gangguan konversi, B. Neurosa obsesif kompulsif.
biasanya: C. Kepribadian ganda
A. Mengikuti pola anatomi distribusi susunan D. Sindroma otak organik
saraf somato sensorik dan somato motorik E. Skizofrenia 55
B. Tidak mengikuti pola anatomi distribusi
susunan saraf somato motorik dan sensorik 26. Pada kepribadian premorbid seorang dengan
C. Tidak pernah terdapat pada anak-anak neurosa obsesif kompulsif biasanya terdapat :
D. Banyak terjadi pada orang tua
A. Ketelitian yang besar
E. Tidak pernah terjadi sebelum pubertas
B. Pemborosan uang.
21. Semua dibawah ini merupakan ciri-ciri C. Lancar dalam pergaulan
kepribadian skizoid , KECUALI : D. Sering sombong
A. Menghindari hubungan erat dengan orang E. Semua diatas salah
lain.
B. Tidak mampu mengekpresi agresi 27.Neurosa depresif / gangguan Distimik, sering
C. Kepribadian gand berhubungan dengan
D. Cara berf A. Perubahan yang mendadak dan kritis
ikir yang autistik B. Penumpukan kesukaran yang pelan pelan.
E. Pemalu dan suka menurut pada masa anak- C. Toxin oxogenik.
anak D. Gangguan endokrin
E. Gangguan metabolisme
22. Seorang dengan nerosa-obsesif
kompulsif biasanya : 28. Menutupi kelemahan dengan menonjolkan
A. Intelgensinya rendah sifat yang diinginkan, adalah mekanisme
B. Kaku dalam pergaulan. pembelaan ego :
C. Premorbid sering acuh tak acuh terhadap gejala A. Rasionalisasi
gejalanya B. Sublimasi
D. Pasien sering acuh tak acuh terhadap gelaja C. Kompensasi
gejalanya D. Proyeksi
E. Biasanya shopping doctor E. Simbolisasi

23. Perbedaan antara gangguan psikosomatik dan 29. Seorang yang pernah mengalami stress,
reaksi konversi ialah pada gangguan, maka dapat mengikibatkan kondisi tersebut
psikosomatik : dibawah ini, KECUALI :
A. Mekanisme pembelaan ego tidak bekerja A. Konsentrasi menurun
B. Gejala Gejalanya yang timbul berdasarkan B. Depresi
simbolik C. Produktivitas dan kreativitas meningkat
C. Yang terganggu adalah saraf vegetatif. D. Perasaan pesimis
D. Mekanisme pembelaan ego yang dipakai E. Bukan salah satu diatas
salah pindah
E. Gejala gejalanya lebih hebat 30. Pernyataan dibawah ini benar berhubungan
dengan stress, kecuali :
24. Mengenai reaksi konversi yang benar adalah : A. Disebabkan oleh suatu stressor
A. Tidak ditemukan gangguan organik sebagai B. Selalu mengakibatkan gangguan jiwa
penyebab C. Dapat mengakibatkan sulit tidur
B. Pasien memperhatikan sekali gejala gejalanya D. Berbagai reaksi tubuh terhadap berbagai
C. Kepribadian premorbid yang memounyai sifat tuntutan
pesimis E. Individu merasa ketidaksepadanan stressor dan
D. M.P.E yang dipakai rasionalisasi sumberdayanya
E. Semua yang diatas salah
31. Kepribadian yang berintegrasi baik menurut
Skiner semua yang tersebut di bawah ini benar, neurologi fokal.
KECUALI: E. Gejala ini berupa gangguan ingatan jangka
A. Efisiensi dalam bekerja / studi. pendek.
B. Menerima diri sendiri 37. Yang disebut sebagai gejala psikologis dan
C. Diterima oleh orang lain tingkah laku pada Dimensia (BPSD- Behavional
D. Hidup dengan prestasi tinggi and Psychological Symptomes of Dimentia )
E. Bebas dari konflik dalam diri sendiri dapat berupa, KECUALI: 56
A. Gangguan daya ingat jangka pendek tahap
32. Keadaan gaduh gelisah : awal.
A. Gejala utama psikomotor yang meningkat B. Gangguan isi pikiran berupa ide/waham
B. Timbulnya dapat akut, sub akut,atau kronis paranoid
C. Tampa disertai ketegangan, kecemasan dan C. Gangguan persepsi berupa halusinasi terutama
kebingungan visual.
D. Tampa disertai halusinasi D. Agresivitas oral maupun fisik.
E. Semua benar E. Gangguan aktivitas antara lain
ngluyur/wandering, aktivitas takbertujuan.
33.Latah, KECUALI :
A. Meniru ucapan atau gerakan yang diperagakan 38. Pernyataan yang benar untuk
dihadapanya Pseudodemensia :
B. Diawali pada usia remaja A. Merupakan suatu bentuk gangguan depresi
C. Didahului oleh keadaan cemas atau dengan manifestasi pada keluhan gangguan daya
keterkejutan ingat
D. Biasa pada kaum wanita dengan pendidikan B. Merupakan suatu bentuk demensia yang
rendah irreversible
E. Pernah bermimpi sesuatu yang berkaitan C. Dalam pemeriksaan sering sekali pasien
dengan masalah menunjukan usaha yang keras dalam menjawab
pertanyaan.
34. Gejala yang biasanya terdapat pada semua D. Jawaban A dan C benar.
jenis gangguan kepribadian adalah : E. Jawaban B dan C benar
A. Rumah tangga yang berantakan
B. Predisposisi genetik 39. MMSE (Mini Mental State Examination )
C. Non sosialisasi merupakan suatu pemeriksaan yang seringkali
D. Tahun pertama dalam kehidupan normal dipakai untuk menilai adanya kemungkinan
E. Semua yang diatas salah gangguan kognitif. Pemeriksaan tersebut berupa
pemeriksaan :
35. Pikiran obsesif biasanya mengandung hal hal A. Orientasi,Proses berfikir, psikomotor, dan
yang bersifat: kemauan
A. Irasional B. Orientasi, afek emosi, dan proses berpikir
B. Imoral C. Orientasi, registrasi, persepsi, dan psikomotor
C. Tidak dapat ditahan D. Orientasi, registrasi, atensi, mengingat, dan
D. Tidak menggangu kehidupan sehari hari berbahasa
E. Tidak tahu bahwa dirinya sakit (terganggu) E. Orientasi, berbahasa, dan gangguan persepsi

36. Pertanyaan dibawah ini benar mengenai 40. Dari jenis demensia dibawah ini, yang paling
Demensia Alzheimer,KECUALI : sering dijumpai adalah :
A. Demensia Alzheimer merupakan demensia A. Demensia Paralitika.
yang paling banyak dijumpai. B. Demensia Alzheimer
B. Prevalensi pada wanita lebih banyak dari pada C. Demensia Multi Infark (Vaskular)
pria D. Demensia pada Penyakit Parkinson
C. Perjalanan penyakit umumnya progresif E. Semua sama
dengan kemunduran bertahap, berbercak dan
berfluktuasi.
D. Pada awal umumnya tidak didapatkan tanda
Rumah Sakit dengan keluhan takut, kadang-
kadang seperti melihat keranda mayat di
depannya, kadang-kadang di dalam hati ada
suara "mati kamu, mati kamu", 3 minggu ini
penderita sering tidak masuk kerja dan kadang-
NEUROPSYCHIATRY 2
kadang memegangi istrinya bila rasa takutnya
1. Seorang penderita laki-laki 30 tahun datang muncul, tensi 130/80, nadi/menit, pernafasan 57
berobat ke Rumah Sakit dengan keluhan sulit 20x/menit, suhu 36 derajat celcius. Pengobatan
tidur, malas Bekerja, sering mendengar suara yang paling baik adalah:
orang bicara yang mengomentari dirinya, padahal A. obat anti depresi
waktu itu tidak ada orang. Pada pemeriksaan B. obat anti cemas
intrinsik, neurologik, serta laboratorium tidak C. neuroleptika
didapatkan kelainan. Kemungkinan gangguam D. terapi elektro konvulsi
jiwa yang diderita oleh penderita tersebut adalah: E. psikoterapi
A. depresi
B. gangguan afektif 5. Seorang wanita usia 30 tahun datang ke rumah
C. neurosa sakit dengan keluhan marah-marah tanpa sebab,
D. skizofrenia gelisah, bila ditanya jawabannya inkoheren
E. mania selama 3 hari sebelum ke rumah sakit. Penderita
sering kejang selama 15 tahun, frekuensi kejang
2. Seorang penderita laki-laki 35 tahun, pekerjaan 2-3 kali per bulan, 5 hari sebelum MRS penderita
tani, pendidikan SD tidak tamat datang ke kejang-kejang selama 10 menit, tensi 125/80, nadi
Puskesmas dengan keluhan utama gaduh gelisah, 92/menit, pernafasan 20x/menit, suhu 37 derajat
mondar-Mandir, bicara melantur kedua celcius. Pemeriksaan neurologik apa yang
tangannya diikat, diantar oleh orang banyak. Pada dibutuhkan pada penderita ini?
pemeriksaan lebih lanjut tidak ditemukan adanya A. CT-Scan
kelainan organik, tetapi ditemukan waham dan B. MRI
halusinasi yang cukup menonjol. Pada penderita C. EEG
tersebut obat apa yang paling tepat diberikan? D. SPECT
A. chlorpromazine E. PET
B. estazolam
C. diazepam 6. Demensia yang paling sering didapatkan
D. alprazolam adalah:
E. amitriptilin A. demensia multi infark
B. demensia alzheimer
3. Seorang wanita usia 26 tahun datang ke Rumah C. demensia paralitika
sakit dengan keluhan marah-marah tanpa sebab, D.demensia pada penyakit parkinson
gelisah, bila ditanya jawabannya tidak sesuai E. semua sama
dengan pertanyaan selama 2 hari sebelum ke
Rumah Sakit. Penderita sering kejang selama 11 7. Yang betul mengenai kesadaran pada psikosa
tahun, frekuensi kejang 2-3 kali per bulan, satu fungsional ialah:
minggu sebelum MRS penderita kejang- kejang A. berkabut
selama 15 menit, tensi 110/70, nadi 110/menit, B. menurun
pernafasan 24x/menit, suhu C. meninggi
36,7 derajat celcius. Selain kelompok obat anti D. terganggu secara kwantitatif
epileptika, pada penderita ini diberikan obat apa? E. terganggu secara kwalitatif
A. alprazolam
B. imipramine 8. Waham yang berhubungan dengan delirium:
C. haloperidol A. biasanya menakutkan
D. diazepam B. biasanya nihilistik
E. amitriptilin C. biasanay tolol dan tanpa efek
D. biasanya merupakan waham kebesaran
4. Seorang laki-laki usia 45 tahun datang ke E. tidak satupun yang di atas
yang dibawah ini, keculai:
9. Yang dimaksud "cimeng" dalam bahasa gaul A. demensia
adalah: B. amnesia
A. heroin C. delirium
B. kokain D. halusinasi
C. ganja E. kesadaran yang menurun
D. alkohol 58
E. rohypnol 16.Gejala penyerta pada retardasi mental berat
adalah:
10.Yang dibawah ini semua merupakan gejala- A. iritabilitas
gejala skizofrenia, kecuali B. agresifitas
A. hipokondria C. gerakan-gerakan stereotipik
B. demensia
C. respon emosi yang bizar D. semua benar
D. ganguan proses berpikir E. tidak satupun yang diatas ini
E. kemauan yang ambivalen
17.Ciri-ciri retardasi mental ringan, kecuali:
11.Untuk mendeteksi apakah seseorang A. umumnya baru diketahui sesudah masuk
menggunakan suatu at psikoaktif biasanya sekolah dasar
dilakukan pemeriksaan terhadap: B. umumnya golongan sosial-ekonomi rendah
A. air seni C. IQ 35-49
B. air liur D. kelainan fisik tidak ada
C. darah E. kelainan organik otak tidak ada
D. air mata
E. liquor serebrospinalis 18.Etiologi retardasi mental adalah:
A. malnutrisi
12.Naloxon dapat digunakan untuk menebtukan B. kelainan kromosom
diagnosis apakah seseorang C. tingkat pendidikan orang tua
ketergantungan yang rendah
A. ganja D. semua benar
B. alkohol E. tidak satupun yang diatas ini
C. barbiturat
D. heroin 19.Keberhasilan terapi pada anak autisme
E. benzodiazepin dipengaruhi oleh:
A. berat/ringannya gejala
13.Terapi untuk pasien delirium adalah: B. tingkat kecerdasan
A. terapi penyebab C. usia anak saat terapi dimulai
B. terapi simptomatik D. kemampuan berbicara dan berbahasa
D. heroin E. semua benar
E. benzodiazepin
C. penunggu adalah keluarga yang dikenal pasien 20.Gangguan psikotik yang mirip skizofrenia
D. pasien ditempatkan di ruangan yang tenang, paranoid diakibatkan oleh pemakaian zat apa?
penerangan cukup A. pil koplo
E. semua diatas benar B. sabu
14.Penyulit yang dapat terjadi pada pasien C. bir/alkohol
skizofrenia: D. ganja
A. bunuh diri E. putauw
B. membunuh orang lain
C. menelantarkan diri 21.Gangguan kepribadian organik mempunyai ciri
D. semua benar klinis, kecuali:
E. tidak satupun yang diatas ini A. labilitas emosi
15.Pada psikosa karena gangguan mental organik B. tidak impulsif
yang akut dapat ditemukan semua C. paranoid
D. gangguan perilaku seksual C. impulsifitas
E. tidak dapat bekerja/berfikir lama D. gangguan atensi
E. kesulitan belajar spesifik
22.Penyalahgunaan NAPZA dapat mengakibatkan:
A. ketergantungan 28.Tanda obyektif berupa pilek sampai bersin,
B. depresi lakrimasi, mual, muntah, diare, dilatasi pupil
C. rusaknya organ-organ tubuh merupakan keadaan putus zat apa? 59
D. semua benar
E. tidak satupun yang di atas ini A. alkohol
B. kanabis
23.Gangguan mental organik artinya C. amfetamin
A. terdapat kerusakan organik pada otak D. opioid
B. adanya gangguan fungsi otak E. nikotin
C. adanya gangguan neurotransmitter di otak
D. gangguan mental yang diketahui disebabkan 29.Komplikasi ganguan tingkah laku pada anak
oleh suatu penyakit badaniah dan remaja:
E. sekumpulan gejala psikiatrik dengan kesadaran A. kegagalan sekolah
yang menurun B. ganguan penggunaan zat
C. hamil di luar nikah
24.Pada pasien demensia terdapat ganguan fungsi D. semua benar
luhur yang meliputi sebagai berikut, E. tidak satupun yang di atas ini
KECUALI:
A. daya pikir 30.Yang membedakan psikosa dengan neurosa
B. daya ingat adalah:
C. daya orientasi A. keluhan psikologis
D. daya pemahaman B. keluhan somatis
E. kesadaran berkabut C. kehilangan "sense of reality"
D. adanya stressor
25.Kelainan kongenital (bawaan) tidak didapatkan E. tidak satupun yang di atas ini
pada tingkat retardasi mental di bawah ini:
A. ringan 31.Semua di bawah ini adalah faktor etiologi dari
B. sedang ganguan kepribadian, KECUALI faktor:
C. berat A. genetik
D. sangat berat B. kognitif
E. semua yang di atas ini C. konstitusi
D. sosial
26.Dalam mengahadapi anak dengan ganguan E. kultural
tingkah laku sebaiknya keluarga (ortu) melakukan
apa, KECUALI: 32.Menurut teori psikoanalisa Freud, dimana
A. anak diberi hukuman yang berat pada fase ini anak merasa berkuasa atas dirinya,
B. tetap memberikan kasih sayang, perhatian, sehingga ia dapat menahan/mengeluarkan urine
disiplin sepantasnya dan fesesnya, fase ini dinamakan fase:
C. menciptakan hubungan akrab sehingga A. falik
persoalan anak dapat teratasi B. oral
D. memberikan contoh figur yang kuat, stabil C. toilet training
E. disarankan anak berteman dengan teman gang D. Laten
yang sudah baik tingkah lakunya E. anal sadistik

27.Gambaran klinis anak dengan ADHD (attention 33.Pada fase tersebut diatas, usia anak sekitar:D.
deficit and hyperactivity disorder), laten
KECUALI A. 0-2 tahun
A. hiperaktifitas B. 2-4 tahun
B. emosi stabil C. 3-5 tahun
D. 6-11 tahun 40.Mekanisme pembelaan ego yang dipakai oleh
E. 11-13 tahun penderita ganguan disosiatif adalah:
A. disosiasi
34.Yang termasuk teori holistik adalah ajaran: B. fantasi
A. Erick Erickson C. salah pindah
B. Harry Stack Sulivan D. pelepasan
C. Karen Horney E. isolasi 60
D. Erick Fromm
E. Sigmund Freud 41.Komponen somatikpada depresi adalah semua
yang di bawah ini, KECUALI:
35.Yang iasanya sangat sulit dari langkah-langkah A. insomnia
cara penyesuaian diri yang berorientasi pada B. anorexia
tugas, antara lain adalah: C. perasaan lelah
A. mempelajari dan menetukan masalah D. berdebar-debar
B. menyusun alternatif penyelesaian E. tidak bertenaga
C. bertindak laku menilai tindakan
D. memilih alternatif yang mempunyai 42.Komponen psikologik pada kecemasan adalah
kemungkinan paling besar akan berhasil semua yang dibawah ini, KECUALI:
E. semua pernyataan di atas salah A. mudah marah
B. gelisah
36.Berikut ini yang tidak termasuk dalam C. optimis
kelompok gangguan disosiatif adalah: D. mimpi yang menakutkan
A. amnesia E. sering terkejut
B. disorientasi
C. fugue 43.Penderita dengan gangguan konversi,
D. trans biasanya:
E. stupor A. mengikuti pola anatomi distribusi susunan
saraf somato sensorik dan somato motorik
37.Yang termasuk dalam kelompok gangguan B. tidak mengikuti pola anatomi distribusi
afektif adalah: susunan saraf somato motorik dan sensorik
A. episode depresif berat dengan gejala psikotik C. tidak pernah terdapat pada anak-anak
B. gangguan skizotipal D. banyak terjadi pada orang tua
C. gangguan psikotik akut dan sementara E. tidak pernah terjadi sebelum pubertas
D. gangguan skizoafektif tipe manik
E. gangguan skizoafektif tipe depresif 44.Semua di bawah ini merupakan ciri-ciri
kepribadian skizoid, KECUALI:
38.Semua gejala di bawah ini termasuk gejala A. menghindari hubunga erat dengan orang lain
utama episode depresif, kecuali B. tidak mampu mengekspresi agresi
A. afek depresif C. kepribadian ganda
B. konsentrasi dan perhatian berkurang D. cara berpikir yang autistik
C. kehilangan minat dan kegembiraan E. pemalu dan suka menurut pada masa kanak-
D. berkurangnya energi yang menuju keadaan kanak
mudah lelah
E. menurunnya aktifitas 45.Seseorang dengan neurosa obsesif kompulsif
biasanya:
39.Mekanisme pembelaan ego yang dipakai oleh A. intelegensinya rendah
penderita ganguan obsesif kompulsif adalah: B. kaku dalam pergaulan
A. represi, penyusunan reaksi, pelepasan C. premorbid sering lelah dan lalai
B. represi, salah pindah D. pasien sering acuh tak acuh terhadap gejala-
C. penyusunan reaksi, salah pindah gejalanya
D. penyususnan reaksi, regresi, pelepasan E. biasanya "shopping doctor"
E. pelepasan, salah pindah
46.Perbedaan antara gangguan psikosomatik dan
reaksi konversi ialah pada gangguan B. Mempunyai efek anti cemas.
psikosomatik: C. Disebut juga Tranqullizer minor
A. mekanisme pembelaan ego tidak bekerja D. Mempunyai efek anti agitasi.
B. gejala-gejala yang timbul berdasarkan simbolik E. Contohnya flufenazin
C. yang terganggua adalah saraf vegetatif 53. Seorang penderita yang suka menggosok
D. mekanisme pembelaan ego yang dipakai gosokkan alat kelaminnya pada pantat yang
adalah "salah pindah" berpakaian lengkap, adalah penderita gangguan :61
E. gejala-gejalanya lebih hebat A. Fetihisme.
B. Frotteurisme.
47.Mengenai reaksi konversi yang benar adalah: C. Veyourisme
A. tidak ditemukan gangguan organik sebagai D. Masokisme.
penyebab E. Nekrofilia
B. pasien memperhatikan sekali gejala-gejalanya
C. kepribadian premorbid mempunyai sifat 54. Yang tersebut di bawah ini termasuk golongan
pesimis anti depresan selective Serotonin Reuptake
D. MPE yang dipakai "rasionalisasi" Inhibitor (SSRI), Kecuali.
E. semua yang diatas salah A. Sertralin.
B.Paroxetine.
48.Yang paling sering berhubungan dengan "La C. Mirtazapine
Belle Indiference" adalah: D. Fluoxentine.
A. reaksi konversi E. Fluvoxamine
B. neurosa "obsesif-kompulsif"
C. kepribadian ganda 55. Yang tersebut dibawah ini termasuk anti
D. sindroma otak organik Psikosis Atipikal, KECUALI :
E. skizofrenia A. Sulpiride.
B. Clozapine.
49.Pada kepribadian premorbid seorang dengan C. Risperidone
neurosa obsesif kompulsif biasanya terdapat: D. Fluphenazine.
A. ketelitian yang besar E. Olanzapine
B. pemborosan uang
C. lancar dalam pergaulan 56. Menutupi kelemahan dengan menonjolkan
D. sering sombong sifat yang diinginkan, adalah mekanisme
E. semua di atas salah pembelaan ego :
A. Rasionalisasi.
50.Neurosa depresif/gangguan distimik, sering B. Sublimasi.
berhubungan dengan: C. Kompensasi
A. perubahan yang mendadak dan kritis D. Proyeksi.
B. penumpukan kesukaran yang pelan-pelan E. Simbolisasi.
C. toxin oxogenik
D. gangguan endokrin 57. Seseorang yang pernah mengalami stress,
E. gangguan metabolisme maka dapat mengakibatkan kondisi tersebut
dibawah ini, KECUALI :
51. Yang benar tentang neuroleptika dosis efektif A. Konsentrasi menurun.
kecil adalah : B. Depresi
A. Obat yang efek sampingnya kecil. C. Produktivitas dan kreativitas meningkat
B. Dapat untuk mengobati epilepsi. D. Perasaan pesimis.
C. Cocok untuk anak kecil E. Bukan salah satu diatas
D. Mempunyai efek anti emetik.
E. Contohnya trihexifenidil 58. Pernyataan dibawah ini benar berhubungan
dengan stress, KECUALI .
52. Pernyataan dibawah ini benar untuk A. Disebabkan oleh suatu stressor.
neuroleptika, KECUALI : B. Selalu mengakibatkan gangguan jiwa.
A. Termasuk golongan psikotropika. C. Dapat mengakibatkan sulit tidur.
D. Berbagai reaksi tubuh terhadap berbagai D. Gangguan pada hipothalamus.
tuntutan. E. Semua diatas salah
E. Individu merasa ketidak sepadanan antara
stresor dan sumberdayanya. 65. Untuk memeriksa seorang terdakwa, seorang
59. Kepribadian yang berintergrasi baik menurut ahli psikiatri sebagai saksi ahli boleh melakukan
Skinner, semua yang tersebut semua hal di bawah ini, KECUALI.:
dibawah ini benar. KECUALI A. Membaca proses verbal dari polisi. 62
A. Efisiensi dalam bekerja / studi. B. Menggunakan sugesti atau persuasi.
B. Menerima diri sendiri. C. Meminta terdakwa dimasukkan ke rumah
C. Diterima oleh orang lain sakit.
D. hidup dengan prestasi tinggi D. Melakukan narko analisa dengan persetujuan
E. Bebas dari konflik dalam diri sendiri terdakwa.
E. Semua diatas benar.
60. Seorang tidak bertanggung jawab secara
kriminal jika tindak kejahatannya merupakan hasil 66. Yang termasuk suportif psikoterapi :
penyakit jiwa atau defek mental. Pernyataan ini A. Katarsia.
sesuai dengan : B. Persuasi.
A. Mc. Naughton Rule. C. Terapi kerja
B.Durham Rule. D. Sugesti.
C. Irresitable impuls E. Semua diatas benar
D. Hukum salah benar Edward
E. Drumond rule 67. Yang tersebut dibawah ini termasuk
neuroleptika, KECUALI :
61. Seorang laki-laki untuk mendapatkan A. Trifluoperazin.
rangsangan seksual harus berpakaian wanita B. Perfenazin.
gangguan ini disebut: C. Pimozide
A. Protteurisme. D. Isokarboxacid.
B. Masokisme. E. Thioridazin
C. Ekshibisionisme
D. Fetihisme transversik. 68. Semua termasuk anti depresan trisiklik,
E. Vayourisme KECUALI :
A. Imipramin.
62. Semua hal yang dikatakan atau dilakukan B. Amitriptilin.
seseorang untuk mengungkapkan bahwa dirinya C. Perazin
laki- laki / wanita , disebut : D. Protiptilin.
A. Identitas jenis kelamin. E. Tofranil
B. Orientasi jenis kelamin.
C. Peran jenis kelamin 69. Gejala yang biasanya terdapat pada semua
D. Identitas seksual. jenis gangguan kepribadian adalah :
E. Faktor psikoseksual A. Rumah tangga yang berantakan.
B. Predisposisi genetik.
63. Kontra indikasi mutlak untuk ECT / TEK adalah C. Non sosialisasi.
: D. Tahun pertama dalam kehidupan normal.
A. Kehamilan. E. Semua yang diatas salah
B. Osteoporosis.
C. Tumor otak 70. Pikiran obsesif biasanya mengandung hal - hal
D. TBC Paru. yang bersifat :
E. Depresi A. Irasional.
B. Imoral.
64. Sebagian besar deviasi seksual terjadi karena. C. Tidak dapat ditahan
A. Gangguan hormonal. D. Tidak mengganggu kehidupan sehari hari.
B. Kekecewaan dalam rumah tangga. E. Tidak tahu bahwa dirinya sakit ( terganggu )
C. Penerangan seksual yang salah
71. Pernyataan dibawah ini benar mengenai
Demensia Alzheimer, KECUALI : 77. Parafilian :
A. Demensia Alzheimer merupakan demensia A. Biasanya tidak membahayakan bagi orang
yang paling banyak dijumpai. dengan gangguan tersebut
B. Prevalensi pada wanita lebih banyak dari pada B. Ditemukan seimbang perbandingan diantara
pada pria. pria dan wanita.
C. Perjalanan penyakit umumnya progresif C. Berdasarkan contoh psikoanalisis klasik, 63
dengan kemunduran bertahap, berbercak dan dikaitkan dengan kegagalan untuk melengkapi
berfluktuasi. pada proses penyesuaian heteroseksual.
D. Pada awal umumnya tidak didapatkan tanda D. Pada usia dini serangan dihubungkan dengan
neurologi fokal. prognosis yang bagus.
E. Gejala ini berupa gangguan ingatan jangka E. Seperti pedophilia biasanya meliputi penetrasi
pendek vaginal atau anal pada korban.

72. Dari jenis demensia dibawah ini, yang paling 78. Prevalensi pada disfungsi seksual diantara
sering dijumpai adalah : pasangan suami istri diperkirakan sebanyak :
A. Demensia Paralitika. A. 5 persen.
B. Demensia Alzheimer. B. 10 persen.
C. Demensia Multi infark ( Vaskular ). C. 20 persen.
D. Demensia pada penyakit Parkinson. D. 30 persen.
E. Semua sama E. Lebih dari 40 persen.

73. Penyakit penyakit yang bisa menyebabkan 79. Manakah dibawah ini yang berhubungan
gejala gaduh gelisah,KECUALI : dengan pria dengan gangguan erectile dikaitkan
A. Delirium. dengan kondisi medis ?
B. Skizofrenia paranoid. A. Gondok.
C. Skizofrenia katatonik gaduh gelisah B. Atherosklerosis.
D. Skizofrenia simplek. C. Sindrom klinefelter.
E. Gangguan bipolar tipe mania. D. Sklerosis berlipat / ,, Multiple sclerosis ,,
E. Semua benar.
74. Yang termasuk kedaruratan psikiatri, KECUALI
: 80. Pruduct rule berkaitan dengan.
A. Gaduh gelisah. A. Upacara keberuntungan ( testimonial previlege
B. Penelantaran diri. ).
C. Gejala lepas zat B. Pengakuan tiba tiba ( involuntary admission )
D. Tentamen suicide. C. Tanggung jawab kriminal.
E. Kecemasan D. Kemampuan untuk bertahan dari masalah.
E. Semua jawaban diatas benar.
75. Kondisi yang sering menyebabkan tentamen
suicide : 81. Masing masing pertanyaan tentang seksual
A. Skizofrenia simplek. masochisme dibawah ini benar, Kecuali.:
B. Bipolar tipe mania A. Yang paling umum ditemukan pada seseorang
C. Gangguan cemas menyeluruh dengan seksual masochisme tidak bisa
D. Gangguan somatisasi. mengambil bagian sebaliknya dari seksual sadis
E. Gangguan Depresi berat dengan penyiksaan dan kesenangan.
B. Segi yang paling penting terdapat pada
76. Gangguan erectile pada pria merupakan kesenangan seksual yang diciptakan dari
A. Terkadang tergantung pada keadaan penderitaan / kesakitan pada seorang itu sendiri.
B. Keluhan utama pada kurang dari 25% semua C. Fantasi fantasi seksual masochistic telah
pria terjangkit gangguan seksual. muncul pada masa kanak-kanak.
C. Umumnya terjadi saat penuaan pada pria. D. Biasanya merupakan penyakit / gangguan
D. Penyebab organic kronis.
E. Semua benar. E. Mutilas diri, jika ada, biasanya berulang
ulang. halusinasi
C. memiliki arti yang nyata dalam praktek klinis
82. Seorang mahasiswa kedokteran mulai dan percobaan terakir
D. tidak digunakan dalam DSM IV
mengantuk di dalam kelas dan terbangun kaget E. diuntuk menggambarkan orang yang
karena merasa namanya dipanggil. Pada mengalami phobia.
kenyataannya, tidak demikian. Hal ini merupakan
satu contoh dari : 64
A. Halusinasi hipnagogik. 88.Semua istilah dibawah ini digolongkan dalam
B. Halusinasi hipnopompik. paraphilia kecuali :
C. Ilusi. A. Fetishism
D. Sinestesia. B. Homosexuality.
E. Disasosiasi / menarik diri C. Exhibitionism
D. Sexual sadism
83. Seorang pria 27 tahun, mendatangi ruang E. Transvestic fetishism.
gawat darurat mengeluhkan tentang pikirannya
yang dikontrol oleh CIA. Pikiran semacam itu 89. Ejakulasi dini dihubungkan dengan
merupakan. A. Stress.
A. Dereistik. B. Dengan kondisi kultur negatif.
B. Sihir. C. Seringnya terjadi pada pria berpendidikan
C. Dipersonalisasi. tinggi dari pada berpendidikan rendah.
D. Obsesi. D. Semata mata karena faktor medis umum
E. Mengigau. E. Semua benar.

84. Seorang pria 26 tahun, yakin bahwa kelompok 90. Pendekatan terapi kognitif termasuk :
Mafia dan saudara laki lakinya meletakkan A. Menimbulkan pikiran yang berkerja sendiri.
pikiran mengerikan kedalam kepalanya, hal ini B. Menguji pikiran yang bekerja sendiri.
merupakan contoh dari. C. Mengenali dugaan yang mendasari
A. Penyiaran pikiran. maladaptive.
B. Waham acuan. D. Menguji kebenaran dugaan maladaptive.
C. Waham nihilistis. E. Semua benar
D. Insersi pikiran.
E. Pseudologika fantastika. 91. Perilaku yang merupakan tanda utama dari
perasaan gemetar selama terapi kejut listrik ( ECT)
85. Gangguan perceptual termasuk yang terdiri dari :
disebutkan dibawah ini, Kecuali A. Bergeraknya jempol kaki
A. Halusinasi. B. Bergeraknya kari jari tangan.
B. Pengalaman hipnagogik. C. Bintik - bintik pada kulit.
C. Ekolalia. D. Apnea.
D. Depersonalization. E. Perluasan plantar ( plantar extension ) pada
E. Derealization. kaki.

86. Meminta seorang pasien untuk mengartikan 92. Manakah dari hal hal dibawah ini yang
sebuah pepatah digunakan sebagai cara untuk merupakan sebab utama adanya klaim
menilai malpraktek pada psikiater dilakukan oleh pasien ?
A. Penilaian. / juaqment A. Percobaan bunuh diri.
B. Kontrol impuls. B. Penggunaan sistem pertahanan yang tidak
C. Pikiran abstrak. tepat ( improper use of restrain ).
D. Pemahaman. C. Kegagalan dalam menyembuhkan penyakit
E. Kecerdasan jiwa.
D. Kepuasan seksual.
87. Istilah psychotic: E. Ketergantungan obat.
A.dipakai untuk distorsi mayor dan mior dari
kenyataan 93. Dalam pemeriksaan status mental
B. digunakan dalam kehadiran waham maupun A. Suasana hati pasien bisa diuraikan secara
normal, tumpul, menyempit dan datar.
B. Halusinasi dan ilusi adalah gangguan dalam
proses pikiran pasien.
C. Meminta pasien untuk mengeja kata ,, dunia ,,
secara terbalik dirancang untuk mengukur FARMAKO-PSIKIATRI
kemampuan visuospasial.
1. Mekanisme kerja benzodiazepine adalah 65
D. Tingkat paling tinggi dalam pemahaman adalah
a. mengaktifkan reseptor GABA di korda
pemahaman emosi nyata.
spinalis
E. Mengalang halangi merupakan sikap yang
b. antagonis terhadap reseptor glysin di korda
diperkirakan oleh pasien untuk menghentikan
spinalis
wawancara.
c. menghambat reseptor glutamate di otak
d. meningkatkan frekuensi pembukaan kanal
94. Tanda pertama penyakit otak awal adalah
ion Cl jika berinteraksidengan reseptor GABA
kerusakan pada :
e. menghambat GABA amino transferase yang
A. Ingatan seketika.
menyebabkan peningkatan kadar GABA
B. Ingatan baru ( diterima )
C. Ingatan jangka panjang.
2. Tipe interaksi obat yang paling sering terjadi
D. Ingatan samar.
pada pasien yang menggunakan obat-obat
E. Salah semua.
golongan sedatif-hipnotik adalah
95. Bertanya pada pasien tentang apa yang akan
a. menambah depresi SSP
dilakukan seandainya mendapat surat dari orang
b. bekerja sebagai antagonis sedative atau
lain yang berada diatntara mereka, merupakan
hipnotik
contoh dari tes.
c. berkompetensi pada ikatan plasma protein
A. Kecerdasan.
d. menginduksi enzim metabolisme obat di
B. Pikiran abstrak
hepar
C. Pemahaman diri / ,, insight ,,
e. menghambat enzim metabolisme di hepar
D. Penilaian / ,,judgment,,
E. Pengenalan / Cognition
3. Yang mana dari obat tersebut di bawah ini
digunakan untuk keadaan withdrawal alcohol?
Penggunaan kronisnya menyebabkan peningkatan
metabolisme warfarin dan phenytoin.
a. chloriazepoxide
b. meprobamate
c. phenobarbital
d. triazolam
e. zolpidem

4. Obat hipnotik yang memfasilitasi efek GABA,


tapi tidak berefek antikonvulsan atau relaksasi
oto, dan mempunyai efek minimal terhadap pola
tidur adalah:
a. Buspiron
b. Diazepam
c. Flurazepam
d. Phenobabital
e. Zolpidem

5. peningkatan sensitivitas pasien tua


terhadap dosis tunggal triazolam & obat
sedatif-hipnotik lainnya adalah karena:
a. perubahan fungsi otak pada proses penuaan
b. penurunan fungsi ginjal
c. peningkatan aliran darah ke otak
d. penurunan metabolisme hepar a. phenytoin
e. perubahan pada ikatan protein plasma b. haloperidol
c. flurazepam
6. flumazenil (Anexate*) adalah: d. Phenobarbital
a. obat neuroleptik e. chlorpromazine
b. obat antidepresi
c. antagonis benzodiazepine 13. mekanisme kerja antipsikotik konvesional 66
d. obat halusinogenik yaitu:
e. obat ansiolitik a. menghambat reseptor GABA
b. menstimulasi reseptor serotonin
7. phenobarbital lebih mudah menimbulkan c. menghambat reseptor dopamin.
toleransi disebabkan karena : d. menstimulasi reseptor adrenergik
a. desensitisasi reseptor GABA e. menghambat kanal ion Na
b. peningkatan ekskresi via ginjal
c. peningkatan metabolisme di hepar 14. efek samping chlorpormazin adalah
d. hambatan absorpsi di usus a. hipertensi
e. belum diketahui penyebabnya b. muntah
c. miosis
8. obat sedatif-hipnotik yang mempunyai d. tidak bisa tidur
efek antikonvulsan dan relaksasi otot adalah: e. mulut kering
a. diazepam
b. phenobarbital 15. mekanisme kerja golongan phenothiazine
c. buspirone tidak spesifik. Mekanisme kerjanya adalah
d. zolpidem sbb, kecuali:
e. zaleplon a. menekan pusat muntah
b. antagonis reseptor dopamin
9. mekanisme buspirone sebagai obat anti c. antagonis reseptor muskarinik
cemas adalah: d. antagonis reseptor GABA
a. agonis reseptor GABA e. antagonis reseptor histamine
b. menghambat MAO
c. agonis reseptor histamine H-3 16. migraine menstrual (karena menstruasi) dapat
d. partial agonis reseptor 5HT-1a dicegah menggunakan NSIDs, karena NSIDs:
e. antagonis reseptor glutamate a. menstimulasi pelepasan serotonin
b. menghambat produksi bradikinin
10. obat tidur yang manakah yang dipilih pada c. menghambat inflamasi neurogenik
pasien dengan keluhan sering terbangun selama d. menghambat prostaglandin yang
tidur adalah : menginitiasi fase vasodilatasi dari migraine
a. phenobarbital e. menghambat reuptake serotonin
b. secobarbital
c. diazepam 17. midrin dapat digunakan apabila analgesik
d. thiobarbital simple tidak berespon adekuat. Midrin adalah:
e. lorazepam a. kombinasi acetaminophen, isometheptene
mucate, (a sympathomimetic amine), dan
11. pernyataan mana yang paling benar tentang dichloralphenazone (a chloral hydrat derivative)
mekanisme phenothiazin? b. agonis serotonin yang selektif
a. mengaktifkan reseptor muskarinik c. agonis dopamin yang selektif
b. sebagai antiemetic (antimuntah) d. 5HT2 perifer antagonis dan 5HT2 sentral
c. menurunkan kadar prolaktin serum agonis
d. meningkatkan ambang rangsang e. ergot alkaloid semisitetik
kejang
e. meningkatkan tekanan darah 18. Disamping mempunyai efek antiemetik,
chlorpromazine juga dapat digunakan untuk
12. gynecomasti merupakan efek samping dari: migraine, karena obat tersebtu :
a. menstimulasi reseptor serotonin adalah:
b. memblok reseptor dopamin a. ulcus pepticum
c. menghambat inflamasi neurogenik b. diabetes mellitus
c. penyakit jantung
d. menghambat prostaglandin yang menginitiasi
d. asthma
fase vasodilatasi dari migraine e. glaucoma
e. menghambat reuptake serotonin
26. obat berikut ini digunakan untuk 67
19. Pernyataan berikut benar untuk methysergide, mengatasi ansietas:
kecuali: a. cocaine
a. adalah agonist 5HT1B/1D yang selektif b. methaqualone
b. dapat merangsang iritasi lambung c. lysergic acid diethylamide (LSD)
d. ecstacy
c. dapat menyebabkan rebound headache
e. morphine
d. mengandung tartazapine yang dapat
merangsang terjadinya alergi 27. tanda dan gejala intoksikasi akut sedative-
e. mempunyai efek antiemetic hipnotik adalah sebagai berikut, kecuali:
a. pupils are midriasis
20. mekanisme kerja sumatripan adalah b. tendon reflexes are decrease
a. selective 5HT1D/1B agonis
c. blook pressure decrease
b. 5HT1 perifer antagonis dan 5HT2 sentral
agonis d. respiration are depressed
c. dopamine reseptor antagonist e. nystagmus, ataxia, slurred speech
d. NE antagonist
e. penghambat sintesa prostaglandin 28. kekuatan efek kokain dan analog kokain
sangat berhubungan dengan efektifitas dalam
21. obat di bawah ini yang dapat digunakan mengeblok transporter yang akan meningkatkan:
untuk prevensi migraine adalah:
a. konsentrasi dopamine at critical brain sies
a. propanolol
b. verapamil b. mengeblok reuptake nrepinefrin (NE)
c. imipramine c. mengeblok reuptake serotonin (5-HT)
d. valproic acid d. menurunkan metabolit neurotransmitter 3-
e. salbutamol methoxy-4 hydroxyphenethyleneglycol(MOPEG or
22. reseptor serotonin (eksitatori) yang berperan MHPG) and 5- hydroxyindoleacetic acid (5-H1AA)
pada patogenesa migraine adalah e. semua di atas benar
a. 5 HT 1D
b. 5 HT 1B 29. Bila seseorang mengalami toleransi terhadap
c. 5 HT 2 suatu obat maka memerlukan dosis yang lebih
d. 5 HT 3 besar dari dosis normal untuk obat kedua agar
e. 5 HT1A berefek (cross tolerance). Keadaan ini terjadi
antara:
23. disamping memblok reseptor NE, a. barbiturat dan kokain
propanolol juga dapat: b. barbiturat dan ecstacy
a. menstimulasi 5HT1D reseptor c. benzodiazepin dan efedrin
b. menstimulasi 5HT1B reseptor d. ampetamin dan kokain
c. memblok 5HT2 reseptor e. semua di atas benar
d. menstimulasi 5HT1A reseptor
e. memblok 5HT3 reseptor 30. neuron bukan satu-satunya sel di CNS. Mereka
didukung oleh berbagai supportive cell seperti
24. gabapentin efektif digunakan untuk profilaksis
sel-sel berikut, kecuali:
migraine. Efek samping gabapentin salah satunya
a. microglia
adalah:
b. macrophage
a. mual
b. mengantuk c. asrocyte
c. peningkatan berat badan d. macroglia
d. dispepsia e. oligo dendroglia
e. konstipasi
31. untuk masuk ke dalam neuron obat harus
25. kontraindikasi penggunaan methysergide
melewati sawar darah otak (BBB/ blood brain bertanggung jawab terhadap
barrier). Untuk berfungsi BBB terdiri dari, kecuali: transport mekromolekul
a. the asctrocyte and feet b. electrically conductive membranes
b. basal lamina (BL) c. tempat sintesis dan menyimpan transmitter
c. endothelial cell with tight junction d. tempat uptake transmitter yang aktif di ujung
d. smooth muscle cell of blood vessel saraf atau glia
e. pericyte cell e. semua di atas benar 68

32. nasib neurotransmitter asetilkolin di celah 38. pilihan utama pada epilepsi petitmal
sinap saraf adalah: a. diazepam
a. reuptake ke neuron diblok oleh kokain b. phenobarbital
b. sebagian besar di metabolisme setelah c. valproic acid
reuptake d. ethosuximide
c. hanya menempati reseptor reuptake e. carbamazepine
muskarinik neuron
d. dimetabolisme oleh asetilkolinesterase 39. Awang usia 30 thn, pekerjaan petani, baru
e. bukan ssalah satu di atas didiagnosa menderita epilepsi grand mal, pilihan
obat utamanya dengan program pengobatan
33. Neurotransmitter berikut dikelompokkan ke jangka pendek dan mengingat kemungkinan side
dalam peptide neurotransmitte, kecuali: effect yang terjadi adalah:
a. tyroid stimulating hormone a. diazepam
b. somatostatine b. phenobarbital
c. vasopressin c. valproic acid
d. angiotensin II d. carbamazepine
e. taurine e. phenytoin

34. neurotransmitter berikut ini menstimulasi 40. epilepsi grandmal dapat berlangsung seumur
neuron, kecuali: hidup. Pengobatannya juga dapat berlangsung
a. glutamate seumur hidup. Pada pertama kali diberikan,
b. histamine saudara merencanakan memberi obat untuk
c. prostaglandin penderita selama :
d. aspartate a. 1 day
e. taurine b. 1 week
c. 2 weeks
35. lebih dari separuh kandungan katekolamin d. 1 month
di otak adalah: e. 2 months
a. norepinefrin
b. dopamin 41. penderita yang mengkonsumsi phenytoin
c. serotonin. jangka panjang side effect yang harus saudara
d. histamin amati adalah:
e. epinefrin a. fungsi hepar
b. hemoglobin
36. neurotransmitter aminoacid, exitatory, c. kadar phenytoin dalam darah
adalah: d. struktur ginggiva
a. dopamin e. semua tersebut harus diamati
b. GABA
c. glutamate 42. mekanisme kerja anti convulsant berupa
d. serotonin calcium channel blockade dimilike oleh:
e. glycine a. ethosuximide
b. phenobarbital
37. tempat yang sensitif terhadap obat di otak c. valproic acid
adalah: d. carbamazepine
a. microtubules dan molecular motors e. diazepam
c. GABA ergic agonist
43. patofisiologi post herpetic neuralgia mirip d. cholinergic agonist
dengan terjadinya focus epilepsi dan penyebaran e. noradrenergic agonist
action potensialnya. Perbedaannya pengobatan
pada penderita post herpetic neuralgia pada 49. pemberian antihistamin dapat mengurangi
umumnya jangka pendek (selama 1-2 minggu). gejala parkinson, walau tidak efektif. Mekanisme
Oleh karena itu obat antiepilepsi yang digunakan kerja antihistamin tersebut terkait dengan 69
untuk post herpetic neuralgia adalah: aktifitas neuron:
a. ethosuximide a. 5 HT meningkat
b. valproic acid b. NA menurun
c. phenytoin c. GABA meningkat
d. carbamazepine d. acethyl choline menurun
e. diazepam e. dopamin menurun

44. anak usia 2 thn, berat badan 10 kg, menderita 50. bromocriptine adalah partial dopaminergic
febris 39 C diserai kejang. Pada keadaan demikian agonist. Oleh karena itu efek samping yang terkait
sikap saudara adalah: dengan konsumsi bromocriptine adalah:
a. keadaan ini adalah emergency a. palpitasi
b. neuron adalah cell yang paling peka terhadap b. mengantuk
anoxia c. penglihatan kabur
c.anoxia neuron menyebabkan degenerasi neuron d. mual-mual
yang irreversible e. diare
d. diberikan diazepam secara i.v
e. semua benar
45. ibu hamil 3 bulan menderita epilepsi. Pilihan
obat yang aman untuk fetusnya adalah
a. phenytoin
b. ethosuximide
c. carbamazepine
d. valproic acid
e. tidak ada yang aman

46. Komunikasi kepada penderita yang


menkonsumsi obat antiepilepsi adalah:
a. jangan menghentikan obat secara tiba-tiba
b. jaga higiena mulut
c. jangan mengkonsumsi alkohol
d. periksa secara periodik ke dokter
e. semua tersebut diinformasikan ke penderita

47. patofisiologi parkinson adalah terjadinya


aktifitas neuron yang menyebabkan:
a. 5 HT meningkat
b. NA menurun
c. GABA meningkat
d. acethyl choline menurun
e. dopamin menurun

48. Berdasarkan patofisiologi terjadinya


parkinson, maka pengobatannya adalah
pemberian:
a. serotoninergic agonist
b. dopaminergic agonist
D. Kelumpuhan n.XII kiri.
E. Reflek tendon menurun.
7. Gejala dan tanda lesi myelum setinggi
torakal 10 ( transection syndrome) adalah :
GABUNGAN - 1 A. Tetraplegia.
B. Seluruh modalitas sensoris terganggu
1. Ciri lesi UMN adalah : 70
bilateral dibawah lesi.
A. Tonus menurun C. Proprioseptif ( getar dan posisi ) normal.
B. Atrofi dan fasikulasi otot. D. Tonus menurun.
C. Pola kelemahan fokal/ local. E. Reflek tendon menurun.
D. Reflek tendon menurun.
E. Respon Ekstensor plantar : positif 8. Gejala dan tanda lesi radiks Sakral 1 (S1)
atau radikulopati adalah :
2. Keadaan yang tersebut dibawah ini A. Nyeri sepanjang distribusi akar saraf
termasuk pola kelumpuhan UMN : Lumbal 5
A. Penyakit motor neuron ( sel-sel kornu B. Rasa kebas (numbness) pada dorsum
anterior) pedis.
B. Radikulopati (lesi pada radiks) C. Paresis fleksi plantar pedis
C. Pleksopati (lesi pada pleksus) D. Paresis ekstensi plantar pedis
D. Mononeuropati E. Reflek tendon Achilles meningkat.
E. Mielopati.
9. Gejala dan tanda lesi n.medianus kiri adalah
3. Keadaan yang tersebut dibawah ini sbb
termasuk pola kelumpuhan LMN : A. Kelumpuhan semua jari tangan kiri.
A. Hemiparesis B. Rasa kebas (numbness) pada semua jari.
B. Tetrapareses dan tonus meningkat C. Reflek tendon normal
C. Paraparesis dan reflek tendon D. Tes Tinel dan Phallen positif.
meningkat. E. Atropi otot-otot lengan bawah kiri.
D. Monoparesis spastik
E. Penyakit Motor neuron (lesi kornu 10. Gejala dan tanda kelainan pada
anterior) neuromuscular junction adalah sbb:
A. Kelumpuhan wajah seringkali terjadi.
4. Dibawah ini adalah tanda lesi LMN : B. Kelumpuhan otot-otot paha.
A. Tonus meningkat. C. Kelumpuhan otot okuler merupakan
B. Pola fokal dari kelumpuhan dan atrofi. kasus paling sering.
C. Tidak ditemukan atrofi otot. D. Kelumpuhan otot-otot thenar.
D. Reflek tendon meningkat. E. Kelumpuhan otot lidah.
E. Respon Ekstensor plantar : positif.
11. Berikut ini bukan merupakan reseptor
5. Gejala dan tanda lesi kortikal hemisfer kiri aferen sensorik :
adalah : A. Meissner untuk raba.
A. Hemiparesis kiri UMN. B. Merkel for tactile
B. Afasia atau gangguan berbahasa. C. Golgi untuk dingin
C. Kelumpuhan n.fasialis kiri. D. Paccine untuk tekan
D. Kelumpuhan n.XII kiri. E. Free nerve ending untuk nyeri
E. Respon fleksor plantar positif.
12. Traktus dibawah ini mentransmisikan
sensasi nyeri :
6. Gejala dan tanda lesi PONS kiri (Braistem) A. Spinothalamicus lateral
adalah: B. Spinocerebellaris
A. Hemiparesis kiri UMN. C. Funiculus dorsalis
B. Afasia atau gangguan berbahasa. D. Corticospinalis lateralis
C. Kelumpuhan n.fasialis kiri. E. Reticulospinal
D. Abses otak.
13. Traktus dibawah ini mentransmisikan E. Tumor otak.
sensasi tekan dan getar :
A. Spinocerebellaris 20. Penyebab coma dengan herniasi uncal tipe
B. Spinothalamicus lateral supratentorial adalah:
C. Spinothalamicus anterior A. Meningitis
D. Corticospinal lateral B. Perdarahan Subarachnoid 71
E. Corticospinal anterior C. Syok Septic
D. Uremic renal failure
14. Berikut ini adalah salah satu pola sensorik E. Intracerebral hemorrhage
perifer (polyneuropathy):
A. Glove and stocking 21. Penyebab coma dengan herniasi sentral tipe
B. Hemiparestesi supratentorial adalah :
C. Onion ring A. Hidrosefalus akut
D. Anestesi level spinal cord B. Diabetic ketoacidosis
E. Thalamic pain C. Septic shock
D. Uremia
15. Berikut ini termasuk ciri lesi funikulus E. Liver failure
posterior/ proprioceptive :
A. Sensasi dingin menurun 22. Central Neurogenic Hiperventilation
B. Sensasi getar dan posisi menurun. merupakan tanda herniasi pada tingkat.
C. Tes suhu menurun. A. Diencephalon
D. Tes rasa nyeri menurun. B. Mesencephalon
E. Tes rasa raba menurun. C. Pons
D. Medulla oblongata
16. Berikut ini TIDAK BENAR tentang lesi E. Medulla spinalis
talamik:
A. Kelumpuhan 23. Hyperthermia merupakan tanda herniasi
B. Nyeri sentral / Central pain pada tingkat .
C. Alodinia A. Diencephalon
D. Nyeri thalamic B. Midbrain
E. Hemihiperpatia C. Pons
D. Medulla oblongata
17. Pusat of persepsi somatosensorik adalah : E. Medulla spinalis
A. Lobus Frontalis
B. Lobus Temporalis 24. Patofisiologi sinkop dengan reflek vasovagal
C. Lobus Occipitalis adalah :
D. Lobus Parietalis A. Dehidrasi
E. Brain stem B. Heat stroke
C. Stimulasi Glomus caroticus
18. Keadaan yang menyebabkan gangguan sistim D. Septic shock
retikuler/ brainstem sehingga terjadi penurunan E. Kelemahan umum.
kesadaran adalah :
A. Hypoxemia 25. Pupil anisocoria, midriasis ipsilateral
B. Hemorrhagic shock merupakan tanda herniasi pada tingkat...
C. Epidural bleeding A. Diencephalon
D. Uremia B. Midbrain
E. Hypoglikemia C. Pons
D. Medulla oblongata
19. Penyebab coma kortikal bihemisfer adalah : E. Medulla spinalis
A. Gagal hepar.
B. Intracerebral hemorrhage 26. Penyebab vertigo perifer adalah :
C. Perdarahan Epidural A. Labyrintitis
B. Perforasi membran timpani C. Vokal nodul
C. Gangguan fungsi tuba D. Gastritis
D. Serumen obsturans E. Tonsilitis akut
E. Benda asing liang telinga

38. Salah satu penyebab nyeri telinga karena 48. Penyebab suara parau adalah :
kelainan telinga tengah adalah : A. Stomatitis 72
A. Sakit gigi B. Sinusitis.
B. Mastoiditis C. Vokal nodul
C. Otitis Eksternal D. Tumor tonsil
D. Frosbite E. Parese nervus hypoglosus
E. Bells palsy
49. Penyebab Stridor inspiratoir adalah :
42. Otore yang disebabkan oleh kelainan pada A. Vokal nodul
liang rongga telinga tengah adalah : B. Sinusitis.
A. Otitis Eksternal C. Vocal cord paralisa
B. Myringitis bullosa D. Stomatitis
C. Chronic otitis media dengan perforasi E. Abses peritonsilar
membran timpani
D. Keratosis obsturans 50. Penyebab nyeri tenggorok adalah :
E. Benda asing liang telinga A. Polip kavun nasi
B. Vocal cord paralisa
43. Penyebab buntu hidung bilateral adalah : C. Sinusitis
A. Hipertrofi adenoid D. Gangguan fungsi tuba Eustachius
B. Benda asing E. Faringitis akut
C. Fraktur os nasal
D. Septum deviasi
E. Neoplasma

44. Penyebab rinore :


A. Fraktur os nasal
B. Choncha hipertropi
C. Septum deviasi ( mild )
D. Benda asing rongga hidung
E. Sakit gigi

45. Penyebab epistaksis adalah :


A. Sinusitis.
B. Septum deviasi ( mild )
C. Choncha hipertropi
D. Polip kavum nasi
E. Sakit gigi

46. Penyebabnya disfagia adalah :


A. Sakit gigi
B. Neuromuskular dysfunction
C. Stomatitis
D. Gastritis
E. Gangguan fungsi tuba Eustachius

47. Penyebab odinofagia adalah :


A. Neuromuskular dysfunction
B. Gangguan fungsi tuba Eustachius
6. Kriteria brain death, kecuali ..
A. Koma yang tidak diketahui penyebabnya
B. Respon motorik negatif
C. Respon pupil negatif
GABUNGAN - 2 D. Respon kornea negatif
E. Respon kornea pada 1 sisi positif
1. Yang merupakan pusat keadaran kecuali: 73
a. ARAS 7. Kondisi penyebab coma
b. Diensefalon A. Vegetative state
c. Mesensefalon B. Lock in syndrome
d. Medula spinalis C. Histeria
e. Cerebrum D. Katatonia
E. Hipoglikemia
Struktur anatomi yang tidak berperan dalam
kesadaran adalah 8. Yang bukan merupakan herniasi system syaraf
a. Korteks serebri pusat
b. - a. Herniasi subtentorial
c. ARAS b. Herniasi central
d. Medula spinalis lumbar c. Herniasi subfacial
e. Thalamus d. Herniasi infratentorial
e. Nucleus Prolapsus Herniation
2. Yang termasuk koma bihemisferik:
a. - 9. Lelaki 60th hemiparese dextra. hemi estesia
b. CVS ICH dextra. CN VII parese dextra tipe UMN. CN XII
c. Meningitis parese dextra tipe UMN. Lesinya di
d. - A. subcortex kiri
e. Brain Tumor B. subcortex kanan
C. pons kiri
3. Tanda dari metabolik encepalopati, kecuali: D. pons kanan
a. Pupil kecil E. medulla oblongata
b. Pupil anisokor
c. Respon patologis (+) pada kedua sisi 10. Ciri2 lesi LMN:
d. - A. Tonus meningkat
e. - B. Focal pattern of weakness and atrofi
C. Tidak didapatkan atrofi otot
4. Pria,55thn. Hilang kesadaran ketika D. Reflex tendon cepat
olahraga disertai muntah-muntah. E. Respon Plantar Extensor Positif
Hemiparesis kanan. Uncontroled
hypertension, 230/120. Pupil anisokor. 11. Ciri2 lesi UMN:
Pernafasan, Central Neurologic A. Tonus menurun
Hyperventilation. Disebabkan: B. Otot mengalami atrofi dan fasikulasi
a. Meningitis C. Focal pattern of weakness
b. Hiponatremia D. Reflex tendon berkurang
c Ensephalitis E. Respon plantar extensor positif
d. CVA ICH
e. Tumor otak 12. Wanita 25 tahun dengan keluhan mata
sering menutup. Kejadian ini hilang timbul. Pagi
5. Penyebab transient conciousness hari normal,makin siang makin menutup seperti
a. syncope mengantuk. Tidak ada gangguan saraf kranial dan
b. hypoglicemi perifer. Kemungkinan lesi terdapat pada:
c. seizure tipe absent A. Batang otak
d. histeris B. Medula spinalis
e. absess cerebri C. Radix
D. Saraf tepi c. Traktus corticospinalis
E. Neuromuscular junction d. Traktus corticobulbaris
e. Funikulus dorsalis
13. Etiologi no.12
a. Stroke 21. Bila ada gangguan suhu dan nyeri setinggi
b. Tumor umbilicus sampai kaki, proprioseptif normal,
c. Guillan Barre tak ada kelemahan. Gangguan pada: 74
d. Myestenia Gravis a. Funiculus prosterior
e. Neuropathy b. Radix T10
c. Segmen T10
14. Seorang mengeluh mengalami kelumpuhan. d. Segmen T11
ditemukan hemiparesis dextra, parese N.6 dan e. Radix T11
N.7 sinistra. kemungkinan lesi pada :
a. cortex. 22. Pria 62 tahun dengan keluhan tubuh
b. subcortex. mengalami hemiplegi. Lesi terletak pada:
c. mesenchepalon. a. N. VI
d. pons. b. N. VII
e. med.oblongata c. Thalamus
d. Midbrain
15. Termasuk dalam syndrom cauda equina e. Cerebellum
adalah
A. Paraplegia 23. Penyebab syncope melalui vasovagal reflex
B. Parahipestesia sebagai berikut:
C. Overlow incontinence a. Dehidrasi berat
D. Reflex balbinsky positif b. Heat stroke
E. Klonus Achilles c. Pukulan Glomus Caroticus
d. Arrytmia Jantung
16. Jalur berikut yang merupakan jalur e. Serangan epilepsi
motorik?
A. T. Spinothalamicus Anterior 24. Di bawah ini yang benar tentang
B. T. Spinthalamicus Latateralis syringomyelia adalah kecuali....
C. T. Cortikocelebral lateralis a. Disasosiasi motorik
D. T. Cortikocelebral posterior b. Nyeri terganggu
E. T. Spinocelebralis c. Suhu terganggu
d. Lesi di canalis sentralis
17. Tanda lesi pda motor neuron motorik.. e. Proprioseptik normal
a. Campuran UMN dan LMN
b. Parahipestesi
c. Tidak ada proprioseptif 25. berikut termasuk jaras untuk sensoris
d. Romberg positif adalah....
e. Hipertrofi otot a. spinothalamicus lateral
b. spinothalamicus anterior
18. Lesi setinggi medulla spinalis T10 c. korticospinal lateral
(transection syndrme) : d. kortikospinal anterior
a. Hemiplegia e. retikulospinal
b. Paraplegia
c. Monoplegia
d. Hemiplegia alternant 26. BagianKornea yang paling kuat
e. Hemihipestesia a. Epitel
b. Stroma
19. Jaras ascending nyeri dan suhu c. Bowman
a. Traktus spinocerebralis d. Descement membrane
b. Traktus spinothalamicus lateralis e. Endothel
a. Tes ishihara
27. Dibawah ini merupakan traktus uvea b. Tes amsler grid
a. Iris, ciliary body, sklera c. Tes pelli robson
b. Ciliary body, choroid, iris d. Tes snellen
c. Iris, sklera, choroid e. Tes perimetri
d. Cilliary body, choroid, retina
e. Sklera, choroid, retina 38. Buta mata merah disebut ??? 75

29. Tipe injection pd penderita uveitis anterior


a. Pericorneal injection
b. Conjuctival injection 40. Yang termasuk media refraksi adalah,
c. Skleral kecuali
d. Episkleral a. Kornea
e. Palpebral b. Aquaeus humor
c. Lensa
30. Yang termasuk media refrakta adalah.. d. Vitreus body
A. Iris e. Fovea
B. Pupil
C. Lensa 42. Yang benar soal lensa kecuali
D. Badan koroid a. Otot siliaris u/ akomodasi
E. Fovea b. Di ikat oleh iris
c. Avaskular
31. Organ Refrakta yang paling tebal: d. Colorless
a. Kornea e. Desturgensi
b. Lensa
c. Aquaeous Humor 43. Tes penglihatan diperiksa dengan
d. Vitreus Humor a. Visual axis
e. b. Snellen chart
c. Ishihara
32. Struktur mata yang berperan penting dalam d. Amsler gride
proses akomodasi adalah e. Photoreceptor
a. Kornea,......,........
b. Iris, cilliary body, choroid 44. Dibawah ini merupakan disturbance of
c. Zonula zinii, cilliary muscle, lensa visual kecuali:
d. .........,............., fovea a. Halos
e. .............................. b. Distortion
c. Visual field defect
33. Struktur mata yang berfungsi dalam d. Visual pathway
penglihatan warna adalah e. Night blindness
a. Rod
b. Cone 45. Yang menyebabkan kornea transparan
c. Sel bipolar A. avascular
d. Sel ganglion B. desturgensi
e. Sel amacrine C. dehidrasi relatif
D. uniform
34. Sel yg bekerja u adaptasi gelap terang E. neovascularisasi
a. Cone
b. Rod 46. Pemberi Nutrisi kornea:
c. Ganglion sel a. air mata, n.ophtalmicus
d.Sel bipolar b. -
e. Sel amakrin c. -
d. -
36. Tes ketajaman penglihatan ? e. vaskularisasi limbus, aquous humour, n.
ophtalmicus

47. Ciri2 red eye konjunctival injection..


A. Fornix to limbus, bright red
b. fornix to limbus, merah gelap GABUNGAN 3
c. limbus to fornix, merah terang
d. - 1. Berikut ini adalah penyebab tuli 76
e. - sensorineural :
A. Otitis Eksternal.
48. Penyebab mata merah: B. Cerumen
C. Nyringitis Bullosa.
a. konjungtivitis akut
D. Noise Induce.
b. Uveitis anterior E. Keratosis Obsturans.
c. Glukoma akut
d. Infeksi kornea 2. Penyebab tinitus oleh karena kelainan
e. Semua benar telinga dalam :
A. Cerumen.
50. Struktur di retina yang berfungsi sebagai inner B. Otosklerosis.
barrier C. Menieres disease.
D. Perforasi membran timpani.
a. Photoreceptor layer
E. Gangguan fungsi tuba Eustachius.
b. Retinal pigment epithelium
c. Retina vessels 3. Penyebab vertigo perifer adalah:
d. Macula A. Labyrintitis.
e. Ganglion cell layer B. Perforasi membrane timpani.
C. Gangguan fungsi tuba.
D. Serumen Obsturans.
E. Benda asing liang telinga.

4. Salah satu penyebab nyeri telinga


karena kelainan telinga tengah adalah :
A. Sakit Gigi.
B. Mastoiditis.
C. Otitis Eksternal.
D. Frosbite.
E. Bells palsy.

5. Otore yang disebabkan oleh kelainan pada


liang rongga telinga tengah adalah :
A. Otitis Eksternal.
B. Myringitis Bullosa.
C. Chronic Otitis Media dengan perforasi
membran timpani
D. Keratosis Obsturans.
E. Benda asing liang telinga

6. Penyebab buntu hidung bilateral


adalah :
A. Hipertrofi adenoid.
B. Benda asing.
C. Frakture os nasal.
D. Septum deviasi.
E. Neoplasma.

7. Penyebab rinore :
A. Fraktur os nasal.
B. Choncha Hipertropi.
C. Septum Deviasi (mild)
D. Benda asing rongga hidung.
E. Sakit Gigi. tengah adalah :
A. Spinocerebelaris anterior.
8. Penyebab epistaksis adalah : B. Spinotalamikus lateral.
A. Sinusitis. C. Spinotalamikus anterior.
B. Septum deviasi (mild) D. Kortikospinal Lateral.
C. Choncha hipertropi. E. Kortikospinal anterior
D. Polip Kavum nasi.
E. Sakit Gigi. 77
17. Di bawah ini termasuk gangguan
9. Penyebab disfagia adalah : sensoris tipe central / UMN adalah :
A. Glove and stocking.
A. Sakit Gigi.
B. Hemiparestesi.
B. Neuromuskular dysfunction.
C. Onion Ring.
C. Stomatitis.
D. Anestesi setinggi level dermatome.
D. Gastritis.
E. Talamik pain.
E. Gangguan Fungsi tuba Eustachius.
18. Di bawah ini bukan termasuk sindroma
10. Penyebab odinofagia adalah :
lesi funikulus posterior / propioseptik :
A. Neuromuskular dysfunction.
A. Decrease sensation of position.
B. Gangguan fungsi tuba Eustachius.
B. Romberg negatif.
C. Vokal Nodul.
C. Decrease sensation of vibration.
D. Gastritis.
D. No Weakness.
E. Tonsilitis Akut. E. Normal sensation of Pain.
11. Penyebab suara parau adalah : 19. Di bawah ini bukan gejala akibat
A. Stomatitis. rusaknya kortek lobus parietalis :
B. Sinusitis. A. Hemiparestesia.
C. Vokal Nodul. B. Hemianestesia.
D. Tumor Tonsil. C. Astereognosia.
E. Parese nervus hypoglosus. D. Ataksia.
E. Agraphestesia.
12. Penyebab Stridor inspiratoir adalah :
A. Vokal Nodul. 20. *Di bawah ini bukan pola gangguan
B. Sinusitis. lesi talamik :
C. Vocal Cord paralisa. A. Hemianestesia.
D. Stomatitis. B. Hiperpatia.
E. Abses Peritonsilar. C. Alodinia.
D. Degree hand = foot.
13. Penyebab nyeri tenggorok adalah : E. Ipsilateral lesi.
A. Polip kavum nasi.
B. Vocal Cord Paralisa. 21. Di bawah ini termasuk gangguan
C. Sinusitis. Brown Sequard :
D. Gangguan fungsi tuba Eustachius. A. Anestesi setinggi level.
E. Faringitis Akut. B. Ipsilateral weakness.
C. Ipsilateral protopatik disturbance
14. Berikut ini termasuk reseptor sensorik D. Konteralateral Disturbance
afferent : E. Ipsilateral reduced vibration.
A. Meissner for touch.
B. Merkel for tactile. 22. Di bawah ini yang benar tentang
C. Golgi for cold. syringomyelia adalah :
D. Paccine for pressure. A. Disosiasi motorik.
E. Free nerve ending for pain. B. Nyeri terganggu.
C. Suhu terganggu.
15. *Di bawah ini termasuk jaras desending untuk D. Lesi di canalis sentralis.
sensoris adalah : E. Propioseptik normal
A. Spinotalamikus lateral.
B. Spinocerebelaris.
C. Funiculus dorsalis.
D. Kortikospinalis lateral. 23. Bila terdapat gangguan propioseptif,
E. Retikulospinal nyeri suhu normal, tidak ada weakness
16. Traktus di bawah ini tidak menyilang garis letak lesi adalah :
A. Kortek lobus parietalis. B. Midbrain
B. Talamus. C. Pons.
C. Cornu anterior spinal cord. D. Medulla Oblongata.
D. Funiculus dorsalis. E. Medulla Spinalis.
E. Radik.
30. Penyebab Syncope melalui Vasovagal
24. Salah satu penyebab Coma dibawah ini akibat reflex sebagai berikut :
Gangguan Retikular system diencefalon / A. Dehidrasi Berat. 78
batang otak, adalah: B. Heat Stroke.
A. Meningitis C. Pukulan Glomus Caroticus.
B. Subarachnoid Hemorrhage. D. Arrytmia Jantung.
C. Epidural Bleeding. E. Serangan Epilepsi.
D. Uremic Coma.
E. Hypoglicemic Coma. 31. Pada Uncal Herniation dengan mass lesion
supratentorial timbulnya Pupil anisocor dengan
25. Salah satu penyebab Coma di bawah ini akibat midriasis ipsilateral yaitu Herniasi tahap berikut :
gangguan cortical cerebral bihemisfer : A. Diencefalon.
A. Meningitis B. Midbrain.
B. Intracerebral Hemorrahage. C. Pons.
C. Epidural Bleeding. D. Medulla Oblongata.
D. Absces Otak. E. Medula Spinalis.
E. Knockout Coma pada petinju.
32. Gejala dan tanda konjungtivitis, selain mata
26. Salah satu penyebab Coma dengan Herniation merah, adalah :
tipe Uncal pada mass lesion supratentorial, A. Papil dan Folikel.
adalah : B. Sekret
A. Meningitis. C. Perdarahan subkonjungtiva.
B. Subarachnoid Hemorrhage. D. Pernyataan A dan B benar.
C. Knockout Coma pada petinju. E. Pernyataan A, B, dan C benar
D. Uremic Coma.
E. Interacerebral Hemorrhage. 33. Pernyataan yang benartentang konjungtiva :
A. Lapisan tebal, membran mucus
27. Salah satu penyebab Coma dengan Herniation transparan.
tipe Central pada mass lesion supratentorial B. Terdiri dari konjungtiva bulbi, konjungtiva
adalah : fornix, konjungtiva palpebra, dan tarsus.
A. Meningitis. C. Vaskularisasi berasal dari arteri siliaris
B. Hydrocephalus Acute. anterior dan arteri palpetra.
C. Knockout Coma pada petinju. D. Persarafan berasal dari divisi
D. Uremic Coma. mandibula Nervus V (Trigeminus) dan Nervus
E. Intracerebral Hemorrhage VII (Fascialis)
E. Konjungtiva bulbi berikatan erat
28. Pada Uncal Herniation dengan mass lesion dengan kapsul tenon dan sklera.
Supratentorial, timbulnya perubahan tipe
34. Penyebab mata merah :
pernafasan Central Neurogenic Hiperventilation A. Konjungtivitis Akut.
yaitu Herniasi tahap berikut : B. Uveitis anterior.
A. Diencefalon. C. Glaukoma Akut.
B. Mesencefalon. D. Infeksi Kornea.
C. Pons. E. Pernyataan A, B, C, dan D benar.
D. Medulla Oblongata.
E. Medulla Spinalis 35. Tanda-tanda pada mata merah berupa :
A. Injeksi konjungtiva.
B. Injeksi Perikorneal.
C. Sekret.
29. Pada Uncal Herniation dengan mass D. Pernyataan A dan B benar.
lesion Supratentorial timbul Hyperthermia E. Pernyataan A, B dan C Benar.
yaitu Herniasi tahap
berikut : 36. Dilatasi perilimbal (circumcorneal) adalah :
A. Diencefalon. A. Hiperemia perilimbal, yang makin
berkurang ke arah formix. kanan.
B. Dilatasi lokal pada pembuluh darah D. Parese N.XII kanan.
episklera. E. Reflex tendon cepat.
C. Hiperemia akibat rupture dari pembuluh
darah. 43. Gajala dan tanda lesi pada level 10th thorocal
D. Hiperemia di formix dan berkurang di spinal cord (transcetion syndrome) adalah sebagai
limbus.
berikut, kecuali : 79
E. Pernyataan B dan C benar A. Paraplegia of the legs.
B. Bilateral impairement of all sensory
37. Lesi Upper Motor Neuron (UMN) memberikan modalities bellow the level.
gambaran sebagai berikut : C. Proprioseption (Vibration, Position sense)
A. Tonus menurun. intact.
B. Otot mengalami atrofi dan D. Tonus meningkat.
fasikulasi. E. Reflex tendon cepat.
C. Focal pattern of Weakness.
D. Reflex tendon berkurang. 44. Lapisan retina yang berperan sebagai
E. Respon plantar extensor positif Outer Barrier adalah:
A. Nerve fiber layer.
38. Pola UMN weakness adalah sebagai berikut di B. Ganglion Cell Layer.
bawah ini, kecuali : C. Outer nuclear layer.
A. Hemispareses. D. Retinal pigment Epithelium.
B. Tetrapareses. E. External limiting membrane.
C. Parapareses.
D. Monopareses. 45. Tanda hyperemia pada inflamasi kornea
E. Penyakit motor neuron (anterior adalah :
here all) A. Episderal Injection.
B. Subconjungtival Bleeding.
39. Lesi lower motor neuron (LMN) memberikan C. Ciliary injection.
gambaran sebagai berikut : D. Conjungtival Injection.
A. Tonus meningkat . E. Scleral Injection.
B. Focal pattern of weakness dan atrofi.
C. Tidak didapatkan atrofi otot. 46. Struktur pada mata yang berperan dalam
D. Reflex tendon cepat. proses akomodasi adalah :
E. Respon Plantar Extensor Positif. A. Kornea.
B. Humor Aqueous.
40. Pola LMN weakness adalah sebagai berikut di C. Lensa.
bawah ini, kecuali : D. Retina
A. Penyakit motor neuron (anterior E. Nervus Opticus
horn cell).
B. Radiculopath. (root lesion) 47. Bagian dari mata di bawah ini bukan
C. Plexopathy (plexus lesion) merupakan media refraksi okuler :
D. Mononeuropathy. A. Kornea
E. Myopathy. B. Vitreous.
C. Lensa.
41. Gejala dan tanda dari lesi pada hemisphere D. Retina.
cortical kiri adalah sebagai berikut, kecuali : E. Humor aqueous.
A. Hemiparese UMN bagian kiri.
B. Bicara normal. 48. Membran Hyaloid vitreous melekat
C. Kelumpuhan wajah sebelah kiri pada beberapa struktur di bawah ini,
(N. VII paresis) kecuali :
D. Parese N XII UMN bagian kiri. A. Kapsul posterior lensa.
E. Respons extensor plantar B. Serabut zonola zinii.
C. Papil Nervus Opticus.
42. Gejala dan tanda lesi pada pons kiri D. Retina.
(Brainstem) adalah sebagai berikut, E. Kornea.
kecuali :
A. Hemiparese Kanan. 49. Struktur di retina yang berfungsi
B. Disarthria (Bicara abnormal) sebagai inner barrier adalah :
C. Parese Nervus Fasialis (N. VII) A. Photoreceptor layer.
B. Retinal Pigment epithelium.
C. Retinal Vassels.
D. Macula.
E. Ganglion Cell Layer.
GABUNGAN 4
50. Struktur pada mata yang berfungsi
mengatur jumlah cahaya yang masuk
ke dalam mata adalah : 1. Tanda dan gejala intoksikasi akut sedative80
A. Kornea. adalah sebagai berikut, kecuali :
B. Lensa a. Pupil midriasis
C. Pupil b. Tendon refleks menurun
D. Retina. c. Blood pressure meningkat
E. Nervus Opticus d. Respiration depressed
e. Nystagmus, ataxia, slurred speech

2. Neuron bukan satu-satunya sel di CNS


didukung oelh berbagai supportive cell seperti sel
berikut, kecuali :
a. Microglia
b. Macrophage
c. Astrocyte
d. Macroglia
e. Oligodendroglia

3. Kekuatan efek kokain dan analog kokain


sangat berhubungan dengan efektivitasnya
dalam mengeblok transporter yang akan
meningkatkan :
a. Konsentrasi dopamin at critical binding site
b. Mengeblok reuptake norepinephrine (NE)
c. Mengeblok reuptake serotonin (5-HT)
d. Menurunkan metabolit neurotransmitter 3-
methoxy-4-hydroxyphenetyleneglycol
(MOPEG atau MHPG) dan 5 -tic acid (5-H1AA)
hydroxyindoleacetic acid (5
e. Semua di atas benar

4. Untuk masuk ke dalam neuron obat harus


melewati sawar darah otak (BBB/Blood Brain
Barrier). Untuk berfungsi BBB terdiri dari, kecuali:
a. The astrocyte and feet
b. Basal lamina (BL)
c. Endothelial cell with tight junction
d. Smooth muscle cell of blood vessel
e. Pericyte cell

5. Nasib neurotransmitter acetilkolin di celah


sinap saraf
adalah :
a. Reuptake ke neuron diblok oleh kokain
b. Sebagian besar di metabolisme setelah
reuptake
c. Hanya menempati reseptor muskarinik
neuron
d. Dimetabolisme oelh asetilkolinesterase
e. Bukan salah satu diatas menderita epilepsi grandmal, pilihan obat
utamanya dengan program pengobatan jangka
6. Daftar neurotransmitter berikut pendek dan mengingat kemungkinan side effect
dikelompokkan ke dalam peptide yang terjadi adalah :
neurotransmitter, kecuali : a. Diazepam
a. Tyroid stimulating hormone b. Phenobarbital
b. Angiotensin II c. Valproic acid 81
c. Vasopressin d. Carbamazepine
d. Somatostatine e. Phenytoin
e. Taurine
13. Penderita yang mengkonsumsi phen
7. Neurotransmitter berikut ini menstimulasi panjang side effect yang harus Saudara amati
neuron, kecuali : adalah :
a. Glutamate a. Fungsi hepar
b. Histamine b. Hemoglobin
c. Prostaglandin c. Kadar phenytoin dalam darah
d. Aspartate d. Struktur ginggiva
e. Taurine e. Semua faktor tersebut harus diamati

8. Lebih dari separuh kandungan katekolamin 14. Anak usia 2 tahun, berat badan 10 kg,
di otak adalah : menderita febris 39C disertai kejang. Pada
a. Serotonin sikap saudara adalah :
b. Norepinephrine a. Keadaan ini adalah emergency
c. Dopamin b. Neuron adalah cell yang paling peka
d. Epinephrine terhadap anoxia
e. Histamin c. Anoxia neuron menyebabkan degenerasi
neuron yang irreversible
9. Neurotransmitter aminoacid d. Diberikan diazepam secara i./v.
a. Dopamin e. Semua benar
b. Gaba
c. Glutamat 15. Komunikasi teradap penderita yang
d. Serotonin mengkonsumsi obat antiepilepsi adalah :
e. Epinephrine a. Jangan menghentikan obat secara tiba
b. Jaga hygiena mulut
10. Tempat yang sensitif terhadap obat di otak c. Jangan menkonsumsi alkohol
adalah : d. Periksa secara periodik ke dokter
a. Microtubules dan molecular motors yang e. Semua diatas dikonfirmasikan ke penderita
bertanggungjawab terhadap transport molekul
b. Electrically conductive membranes 16. Obat hipnotik yang memfasilitasi efek
c. Tempat sintesis dan m GABA, tidak berefek antikonvulsan atau
transmitter relaksasi otot dan punya efek minimal terhadap
d. Tempat uptake transmitter yang aktif di pola tidur adalah :
ujung saraf atau glia a. Buspiron
e. Semua di atas benar b. Diazepam
c. Flurazepam
11. Pilihan utama pada epilepsi petitmal : d. Phenobarbital
a. Diazepam e. Zolpidem
b. Phenobarbital
c. Valproic acid 17. Peningkatan sensitivitas pada pasien tua
d. Ethosuximide terhadap dosis tunggal triazolam & obat
e. Carbamazepine sedative lainnya adalah karena :
a. Perubahan fungsi otak
12. Awang, usia 30 tahun, pekerjaan petani b. Penurunan fungsi ginjal
c. Peningkatan aliran darah ke otak b. Antagonis reseptor muskarinik
d. Penurunan metabolisme hepar c. Antagonis reseptor GABA
e. Perubahan pada ikatan protein plasma d. Antagonis reseptor Dopamin
e. Antagonis reseptor Histamin
18. Flumazenil (Anexate) adalah :
a. Obat neuroleptik 25. Migraine menstrual (karena menstruasi)
b. Obat antidepresi 82
dapat dicegah menggunakan NSIDs, karena NSIDs:
c. Antagonis benzodiazepi a. Menstimulasi pelepasan serotonin
d. Obat halusinogenik b. Menghambat produksi bradikinin
e. Obat anxyolitic c. Menghambat inflamasi neurogenik
d. Menghambat prostaglandin yang
19. Obat sedativ hipnotik yang mempunyai menginisiasi fase vasodilatas
efek antikonvulsan dan relaxan otot adalah : e. Menghambat reuptake serotonin
a. Diazepam
b. Phenobarbital 26. Midrin dapat digunakan apabila analgesik
c. Buspirone simpel tidak berespon adekuat. Midrin adalah :
d. Zolpidem a. Kombinasi acetaminophen, isometheptene
e. Zolepilon mucate (a sympathomimetic amine) dan
didoraphenazone (a chloral hydrate
20. Pernyataan yang paling benar mengenai derivative)
mekanisme phenotiazine adalah : b. Agonis serotonin yang selektif
a. Mengaktifkan reseptor muskarinik c. Agonis dopamin yang selektif
b. Sebagai antiemetik (antimuntah) d. 5HT2 perifer antagonis dan 5HT2 sentral
c. Menurunkan kadar prolaktin serum agonis
d. Meningkatkan ambang rangsang kejang e. Ergot alkaloid semisintetik
e. Meningkatkan tekanan darah
27. Disamping mempunyai efek antiemetik,
21. Mekanisme kerja antipsikotik konvensional chlorpromazine juga dapat digunakan untuk
yaitu : migraine, karena obat tersebut :
a. Menghambat reseptor GABA a. Menstimulasi reseptor serotonin
b. Menstimulasi reseptor serotonin b. Mengeblok reseptor dopamin
c. Menghambat reseptor dopamin c. Menghambat inflamasi neurogenik
d. Menstimulasi reseptor adrenergik d. Menghambat prostaglandin yang
e. Menghambat kanal ion Na menginisiasi fase vasodilatasi dari migraine
e. Menghambat reuptake serotonin
22. Reseptor serotonin (eksitatori) yang
berperan pada patogenesa migrain adalah : 28. Pernyataan berikut benar untuk
a. 5 HT 1D methylsergide, kecuali :
b. 5 HT 1B a. Adalah agonist 5HT1B/1D yang selektif
c. 5 HT 2 b. Dapat merangsang iritasi lambung
d. 5 HT 3 c. Dapat menyebabkan rebound headache
e. 5 HT 1A d. Mengandung tartazapine yang dapat
merangsang terjadinya alergi
23. Efek samping chlorpromazine adalah : e. Mempunyai efek antiemetik
a. Hipertensi
b. Muntah 29. Mekanisme kerja sumatriptan adalah :
c. Miosis a. Selective 5HT1/1D agonis
d. Insomnia (tidak bisa tidur) b. 5HT1 perifer antagonis dan 5HT2 sentarl
e. Mulut kering agonis
c. Dopamin releasing receptor agonist
24. Mekanisme kerja phenotiazine tidak spesifik. d. NE antagonist
Mekanisme kerja adalah sbb, kecuali : e. Penghambat sintesa prostaglandin
a. Menekan pusat muntah
30. Obat dibawah ini dapat digunakan untuk e. Medulloblastoma
prevensi migraine adalah :
a. Propanolol 37. Psammomma bodies didapatkan pada :
b. Verapamil a. Astrocytoma
c. Imipraimine b. Oligodendroglioma
d. Valproic acid c. Pilocytic astrocytoma
e. Salbutamol d. Meningioma 83
e. Medulloblastoma
Patologi Anatomi
31. Neoplasma yang sering terjadi di otak adalah: 38. Meningioma dengan klasifikasi WHO grade III
a. Tumor metastase adalah :
b. Tumor primer a. Synctitial menigioma
c. Tumor pembuluh darah b. Fibroblastik mengingioma
d. Neurofibromatosa c. Anaplastic meningioma
e. Osteo sarcoma d. Transitional meningioma
e. Psammommatous meningioma
32. Neoplasma CNS pada anak-anak sering
terjadi di daerah : 39. Verocay bodies didapatkan pada :
a. Hemisphere cerebri a. Meningioma
b. Fossa posterior b. Schwannoma
c. Corda spinalis c. Neurofibroma
d. Lobus frontalis d. Astrocytoma
e. Lobus parietalis e. Medulloblastoma

33. Tumor-tumor yang berasal dari sel glia adalah 40. Tumor jaringan saraf subkutan multipel :
: a. Meningioma
a. Astrocytoma b. Neurofibromatosa
b. Oligodendroglioma c. Schwannoma
c. Ependymoma d. Dermatofibroma
d. Glioblastoma e. Fibroushistiocytoma
e. Semua benar
41. Agar terhindar dari serangan stroke,
34. Gambaran nekrosis dengan kelompok sel seseorang yang memiliki resiko stroke
dianjurkan untuk rutin melakukan pemeriksaan
pseudopalisade di sekitarnya adalah : tekanan darah (screening).
a. Meningioma Hal ini termasuk :
b. Pilocytic astrocytoma a. Primary pevention
c. Ependymoma b. Secondary prevention
d. Glioblastoma multiforme c. Tertiary prevention
e. Oligodendroglioma d. Common prevention
e. Specific protection
35. Pilocytic astrocytoma tegolong klasifikasi
WHO grade 42. Faktor risiko stroke yang tidak dapat
a. I dikontrol adalah :
b. II a. Kebiasaan merokok
c. III b. Tinggi kolesterol
d. IV c. Serangan stroke / TIA sebelumnya
d. Obesitas
36. Berikut ini yang termasuk neoplasma jinak e. Atrial fibrillation
adalah :
a. Astrocytoma 43. Tekanan darah tinggi merupakan yang
b. Oligodendroglioma paling dialami penderita stroke. Hal apakah
c. Malignant peripheral nevesheath tumor yang paling penting dilakukan untuk mengelola
d. Fibroblastik meningioma tekanan darah
tinggi? melakukan ...
a. Melakukan general check up a. Impact evaluation
b. Meminum obat-obatan b. Process evaluation
c. Mengkonsumsi jamu/herbal c. Outcome evaluation
d. Modifikasi gaya hidup d. Formative evaluation
e. Mengendalikan berat badan e. Predictive evaluation
84
44. Kampanye FAST (Face Arm Speech Time) Diketahui jumlah karyawan yang terkena
tidak bisa diterapkan di Indonesia karena masih stroke di suatu perusahaan dalam kurun waktu
belum tersedia sistem informasi yang 5 tahun terakhir cukup tinggi. Setelah
terintegrasi untuk merespon kondisi emergensi dilakukan survey pada semua karyawan
seperti 911 (AS) atau 999 (Inggris). Evaluasi ditemukan bahwa kebiasaan merokok
seperti apa yang digunakan untuk keadaan merupakan faktor risiko yang paling banyak
tersebut? teridentifikasi. Para perokok ini sadar bahwa
a. Impact evaluation kebiasaan mereka berbahaya tetapi mereka
b. Process evaluation tidak tahu bagaimana menghentikan kebiasaan
c. Outcome evaluation tersebut
d. Formative evaluation
e. Predictive evaluation 49. Manakah pernyataan berikut yang
paling tepat sebagai sub-tujuan (sub program
45. Membaca literatur dapat membantu kesehatan di perusahaan tersebut?
mengidentifikasi kelebihan dan kekurangan yang a. Meningkatnya jumlah karyawan yang tahu
kita buat. Aktivitas ini merupakan contoh dari ... bahaya merokok
a. Impact evaluation b. Meningkatnya jumlah karyawan sebesar
b. Process evaluation 50% yang tahu bahaya merokok dalam
c. Outcome evaluation kurun waktu 2 tahun
d. Formative evaluation c. Meningkatnya jumlah
e. Predictive evaluation karyawan yang
berhenti merokok hingga 20% dalam waktu
46. Mahasiswa kedokteran melakukan talkshow 1 tahun
di pusat perbelanjaan. Ternyata jumlah peserta d. Meningkatnya jumlah
melebihi target. Keberhasilan pencapaian target karyawan yang
peserta termasuk ke dalam ... memanfaatkan Pogram berhenti merokok
a. Impact evaluation sebanyak 20% dalam waktu 1 tahun
b. Process evaluation e. Menurunkan jumlah karyawan yang
c. Outcome evaluation terserang penyakit s
d. Formative evaluation dalam waktu 1 tahun
e. Predictive evaluation
50. Metode apakah yang paling tepat
47. Setelah dilakukan program pencegahan digunakan dalam kegiatan promosi kesehatan
stroke, selama 5 tahun di kota telah terjadi di perusahaan tersebut berdasarkan kasus di
penurunan sebanyak 20%. Tahap evaluasi apa ini? atas?
a. Predictive evaluation a. Memasang iklan bahaya merokok di
b. Impact evaluation beberapa surat kabar nasional
c. Formative evaluation b. Menayangkan iklan tentang car berhenti
d. Outcome evaluation merokok di TV nasional
e. Process evaluation c. Memuat artikel populer tentang usaha
orang-orang yang berhasil berhenti merokok di
48. Hasil evaluasi suatu program kesehatan majalah perusahaan
mendapatkan 20% dari tim penyelenggara tidak d. Membuat acara talkshow tentang
menjalankan tugas kewajibannya dengan baik e. Mebagi-bagi T-shirt kepada seluruh karyawan
sehingga hanya 80% dari program yang bisa yang berisi pesan tentang bahaya merokok
terlaksana. Hal ini bisa diketahui setelah
51. Parasit yang dapat menyebabkan e. Echinococcus granulosus
encephalitis yang memiliki DH tikus adalah ...
a. ... 58. Primary emebic meningoencephalitis
b. Taenia saginata penyebabnya
c. Taenia solium adalah ...
d. Angiostrongylus cantonensis a. Naegleria fowleri
e. Echinococcus granulosus b. Entamoeba hystolitica 85
c. Angiostrongylus canthonensis
52. Parasit di bawah ini belum pernah penyebab d. Toxoplasma gondii
meningo-encephalitis : e. Echinococcus granulosus
a. Naegleria fowleri 59. Mekanisme dasar terjadinya komplikasi pada
b. Taenia solium cerebral
c. Taenia saginata malaria adalah ...
d. Angiostrongylus cantonensis a. Hipoglikemia
e. Echinococcus granulosus b. Endocytosis
c. Pengeluaran radikal bebas
53. Parasit di bawah ini yang portal of entry d. Rossetting
lewat nasopharynx dan N. Ophtalmicus adalah ... e. Tissue anoxia
a. Acanthamoeba
b. Taenia solium 60. Proses di bawah ini yang menyebabkan
c. Taenia saginata cytoadherence pada endotel :
d. Angiostrongylus cantonensis a. Hipoglikemia
e. Echinococcus granulosus b. Endocytosis
c. Pengeluaran radikal bebas
54. Terapi untuk sistiserkosis otak adalah ... d. Rossetting
a. Arthesunat e. Tissue anoxia
b. Mebendazole
c. Metronidazole mikrobiologi
d. Sulfadoxin-pyrimethamine 61. Bakteri yang sering menyebabkan infeksi
e. Primakuin pada sistem
saraf adalah ...
55. Drug of choice untuk cerebral malaria adalah a. Streptococcus pneumoniae
... b. Herpes simplex
a. Arthesunat c. Cryptococcus neoformans
b. Mebendazole d. Clostridium botulinum
c. Metronidazole e. Bukan salah satu diatas
d. Sulfadoxin-pyrimethamine
e. Primakuin 62. Mengenai infeksi Mycobacterium leprae
berikut ini
56. Terapi untuk toxoplasmosis adalah ... benar, kecuali :
a. Arthesunat a. Bakteri patogen intraselular
b. Mebendazole b. Masa inkubasi panjang
c. Metronidazole c. Sulit dikembangkan pada media buatan
d. Sulfadoxin-pyrimethamine d. Menyerang kulit dan saraf
e. Primakuin e. Diagnosis pasti dengan lepromin test

63. Mikroba berikut ini menghasilkan toxin


yang
57. Penyebab hydrocephalus adalah infeksi ... menyebabkan kelumpuhan :
a. Naegleria fowleri
b. Taenia solium
c. Angiostrongylus canthonensis
d. Toxoplasma gondii a. Clostridium tetani
b. Polio virus
c. Rabies virus
d. Clostridium botulinum
e. Listeris monocytogenes

64. Transmisi infeksi poliovirus terutama terjadi


melalui : 86
a. Fecal-oral route
b. Transfusi darah
c. Parenteral
d. Infeksi kongenital
e. Airbone droplets

65. Pemeriksaan mikrobiologi meng


untuk melihat penyebab infeksi CNS :
a. Haemophillus influenzae
b. Listeris monocytogenes
c. Cryptococcus neoformans
d. Poliovirus
e. Clostridium tetani
72. Neuroimaging modality for brain trauma :
66. Obat-obat anaestesi mempengaruhi sistem a. Foto polos
saraf pusat dengan mekanisme : b. MRI
a. Menurunkan cerebral metabolic rate c. CT scan
b. Menurunkan cerebral blood flow d. USG
c. Meningkatkan cerebral metabolic rate e. DSA
d. Meningkatkan aktivitas depolarisasi neuron
korteks 73. The advantages of Ultrasonography / USG is :
e. Meningkatkan excitatory neurotransmitter a. Non invasive, non radiation, cheap, easy
b. Non invasive, non radiation, multiplanar
67. Komponen otak yang paling banyak c. Non invasive, minimal radiation, axial
menerima cerebral blood flow adalah : imaging
a. Kortikal d. Minimal radiation, cheap, easy
b. Subkortikal e. Safe, easy, minimal inv
c. Basal ganglia
d. Mesencephalon 74. Keuntungan MRI adalah :
e. Cerebellum a. Minimal radiation, cheap, easy
b. Non radiation, multiplanar imaging, non
68. Bila terjadi peningkatan tekanan invasive
intracranial karena edema cerebri, kompensasi c. Non invasive, fast imaging, cheap
awal yang dilakukan adalah mengurangi volume: d. Axial imaging, easy, non radiation
a. Vaskular e. Non radiation, axial planar, safe
b. Arteri
c. Vena 75. Neuroimaging modality for mielitis
d. Liquor cerebrospinal a. Foto polos
e. Plexus choroideus b. MRI
c. CT scan
69. Regulasi cerebral blood flow bertujuan d. USG
untuk: e. DSA
a. Mempertahankan cerebral perfussion
pressure
b. Mempertahankan TIK
c. Mempertahankan suplai oksigen
d. A dan C benar
e. A dan B benar

70. Obat berikut yang paling banyak menurunkan


CMR :
a. Propofol
b. Ethomidate
c. Thiopental
d. Diazepam
e. Midazolam

Radiologi
71. Tujuan imaging pada acute stroke patient
adalah :
a. Indentify cerebral edema
b. Identify stenosis intracranial arteries
c. Identify mass effect
d. Identify herniation
e. Identify supra/infratentorial lession

Anda mungkin juga menyukai